Department of Health and Ageing – Emergency Triage
CHAPTER 9: PREGNANCY AND
TRIAGE
Statement of purpose
The purposes of this chapter are to:
• Provide an outline of the physiological adaptations that occur in pregnancy; and
• Discuss the factors that influence the triage code allocation for pregnant women.
Learning outcomes
After completing this chapter, participants will be able to state the
main physiological changes that occur in pregnancy and explain
how these adaptations will influence the allocation of a triage code.
Participants will also be able to identify common
and life-threatening complications that present to triage and discuss
how urgency is determined for these conditions.
Learning objectives 72
• Outline the physiological changes in pregnancy that
may modify triage decision-making.
• Describe the relevant questions to ask about a woman’s obstetric history.
• Discuss common non-obstetric conditions that may adversely
impact on a pregnant woman and the unborn child.
• Explain the maternal factors that may alert the Triage Nurse
that urgent foetal assessment is required.
• Discuss significant obstetric complications of pregnancy that
impact on the pregnant woman and the unborn child.
Key points
• All women of child-bearing age should be considered to be
pregnant until proven otherwise.
• An assessment of urgency must be made on the basis of both
the woman and the foetus.
• An elevated BP is an ominous sign: the higher the BP the more
urgent the review.
• Pregnant women are at an increased risk of a number of
conditions, including cerebral haemorrhage, cerebral thrombosis,
severe pneumonia, atrial arrhythmias, venous thrombosis and
embolus, spontaneous arterial dissection, cholelithiasis and
pyelonephritis, than non-pregnant women of child-bearing age.
• Presentations may include concerns about normal
Content
Triage and the pregnant patient
A pregnant woman presenting to an ED raises a number of unique
challenges to the Triage Nurse.
• The Triage Nurse needs to be aware of the normal physiological
and anatomical adaptations of pregnancy because these will
influence assessment.
• Triaging should consider the wellbeing of both the mother and
the foetus and potential threats to either.
• The pregnant woman may present with any disease.
The presentation of some diseases is modified by pregnancy and
some diseases only occur in pregnancy.
Pregnancy and the primary survey
Airway
Any pregnant women presenting to the ED with a potentially
compromised airway needs urgent medical attention. Pregnant
73 women are often difficult intubations due to patient size, patient
positioning and different induction agent requirements due to
cardiovascular physiological changes.
Breathing
Progesterone is thought to be responsible for altering the sensitivity
of the respiratory centre and increasing the drive to breathe.119
• Pregnant women commonly experience increased nasal and
airway vascularisation and mucosal oedema. This presents as an
increase in complaints about nasal congestion.
• About one-third of women with asthma suffer a deterioration of
their illness during pregnancy.120
Circulation
Pregnancy is described as a hyperdynamic state and physiological
changes occur as early as 6–8 weeks gestation. Progesterone causes
widespread vasodilatation and oestrogen is thought to contribute to a
40–50 per cent increase of blood volume.The diastolic blood pressure
falls on average 6–17 mmHg, with BP lowest during the second
trimester. Cardiac output (CO) increases by 30–50 per cent.
At 20 weeks gestation, the weight of the uterus compresses the
inferior vena cava if the woman is lying on her back.The subsequent
reduction in placental flow is enough to compromise foetal wellbeing
and the drop in venous return reduces maternal CO and BP.
Unspecified changes occur to blood vessels that predispose pregnant
women to spontaneous arterial dissections.121
Department of Health and Ageing – Emergency Triage
The splenic artery, subclavian artery and aorta, for example, have an
increased tendency to spontaneous dissection, even in women with no
previous medical history.
Domestic violence is more common during pregnancy and is
associated with an increase in obstetric complications for the mother
and adverse neonatal outcomes.123
Important points to note:
• Pregnant women often describe palpitations during pregnancy,
which is usually due to the hyperdynamic flow.
• The high volume and dynamic blood flow is thought to contribute
to the increased likelihood of cerebral haemorrhage (especially
sub-arachnoid haemorrhage
(SAH)) in pregnancy.
• It is not uncommon for pregnant women to experience a
sudden and serious deterioration of their condition therefore
pregnant women showing signs of haemodynamic de-
compensation require urgent medical assessment.124
• All pregnant women >20 weeks gestation should have a left lateral
tilt (wedge under their right hip, or whole bed tilted if wedge is
contraindicated) if they are lying down.
• Pulmonary embolus is relatively common during pregnancy due
to the changes in the coagulation system associated with
pregnancy.
• In the setting of trauma, all usual trauma criteria should be
considered. Additional considerations include trauma to the
uterus, placenta or foetus, particularly in the third trimester when 74
the foetus is viable.The maternal vital signs may remain stable
even when loss of one-third of blood volume may have
occurred.125
• ‘The best initial treatment for the foetus is the optimum resuscitation of the
mother.’125
Common conditions that present to ED
according to gestational age
Problems occurring prior to 20 weeks
Pregnant women frequently present to the ED with vaginal
bleeding. Common causes include the various types of miscarriage
(i.e. threatened, inevitable, complete, incomplete and septic).
• Knowledge of the volume and colour of per vaginal (PV) loss will
assist the Triage Nurse with categorising the urgency of the case.
• Bright red blood loss is usually indicative of active bleeding,
while brownish red blood loss is usually old.
• Many women may also complain of associated abdominal pain that
may be likened to severe period pain.
• Shoulder tip pain can be indicative of a bleeding ectopic pregnancy.
• The first and foremost diagnosis to exclude in the female of child-
bearing age, including those who have undergone sterilisation
procedures presenting with vaginal bleeding, is an ectopic
pregnancy.126
Abdominal pain is the most common symptom in ruptured ectopic
pregnancy.127 Non-ruptured ectopic pregnancies generally present
with bleeding (brown being the most common) due to low
progesterone and consequent shedding of the decidua.
Regardless of the diagnosis, vital signs that deviate from normal and
severe pain (such as torsion or ruptured cysts) warrant prompt
medical assessment.
Problems occurring from 20 weeks onwards
Pregnant women from 20 weeks gestation may present with the
following obstetric conditions:
• Antepartum haemorrhage
• Preeclampsia (including eclampsia)
• Pre-term rupture of the membranes and labour.
Hypertension (>140/90) is a particularly important sign to alert
the Triage Nurse to a more serious problem.The presence of the
associated symptoms of severe preeclampsia warrants urgent
medical assessment.These include:
• Headache
• Visual disturbances
• Epigastric pain
75 • Right upper quadrant (RUQ) pain
• Non-dependent oedema.
These women are at risk of fitting and placental abruption, and the
foetus has a higher risk of placental insufficiency.
There is a correlation between the degree of hypertension and
complications such as cerebral haemorrhage.
• Antepartum haemorrhage is defined as >15 mL of blood loss from
the vagina from 20 weeks gestation.
• Common causes include placenta praevia and placental abruption.
• In placenta praevia, blood loss is usually visible PV and is not
usually accompanied by pain.
• In placental abruption, the primary symptom is abdominal
pain.The associated blood loss may be concealed between the
placenta and uterus. Haemodynamic changes are only seen with
big bleeds, smaller bleeds may be difficult to detect or more
easily detected with an abnormal cardiotocograph (CTG).The
main signs
and symptoms are haemodynamic changes associated with
hypovolaemic shock and abdominal pain.
Department of Health and Ageing – Emergency Triage
Postnatal women may present with the following:
• Secondary postpartum haemorrhage ± puerperal sepsis
• Mastitis
• Wound infection
• Eclampsia
• Postpartum cardiomyopathy
• Postnatal depression.
Urgent threats to foetal wellbeing
• Changes in oxygen saturations in the mother are of direct
relevance to foetal wellbeing. A small reduction in maternal
oxygenation can severely impact on
foetal oxygenation because of the left shift in the oxyhaemoglobin
dissociation curve associated with foetal haemoglobin.129 Consider
oxygen saturation at triage on all pregnant women.
• Major alterations in blood pressure (whether high or low) are not
well tolerated by the foetus.
• Active vaginal bleeding at any gestation presents a risk to the foetus.
• Abdominal pain during pregnancy may represent a pathological
process threatening the foetus.
• Pregnant women normally feel foetal movement from 18–20 weeks
gestation. A regular pattern of foetal movement is a reassuring
sign of foetal wellbeing. Absent or diminished foetal movements 76
require prompt assessment.
Teaching resources
Further reading
• Angelini DJ. Obstetric triage revisited: Update on non-obstetric
surgical conditions in pregnancy. Journal of Midwifery & Womens
Health 2003;48(2):111–18.
• Beischer NA, MacKay EV, Colditz PB. Obstetrics and the
Newborn: An Illustrated Textbook. 3rd ed. London: Bailliere
Tindall; 1997
• Coppola M, Della-Giustina D. Emergency Gynaecological
Presentations. Emergency Medicine Clinics of North America
2003; Aug: 21.3
• DeLashaw MR,Vizioli TL, et al. Headache and seizure in a young
woman postpartum. Journal of Emergency Medicine
2005;29(3):289–93.
• Fuschino W. Physiologic changes of pregnancy: impact on critical
care. Critical Care Clinics of North America 1992;4(4):691–701.
• Murray H, Baakdah H. et al. Diagnosis and treatment of ectopic
pregnancy. Canadian Medical Association Journal
77 2005;173(8):905–12.
Teaching strategies
Multiple-choice questions
Select the correct response by circling one answer only.
1. The normal blood pressure changes that occur during pregnancy, develop:
(a) at about 20 weeks gestation
(b) from about 6–8 weeks gestation
(c) during the third trimester
(d) during the second trimester.
2. A 26-year-old woman in her third trimester presents to the ED
with a severe headache.What are the primary pieces of
information you need to make a triage decision?
(a) past medical history, presenting history
(b) conscious state and blood pressure
(c) what medications the woman has taken
(d) her gestation and presence of foetal movements
Short-answer questions
1. A 42-year-old woman presents with no foetal movements. Discuss
the information you need to determine a triage category.
2. A 32-year-old woman presents with PV bleeding. She says that it is
possible that she is pregnant, though she’s not sure. Describe how
you would assess this woman.
3. A 39-year-old woman who is 27 weeks pregnant presents to the ED
with chest pain. What are the potential causes of this symptom,
and how may they be distinguished?
Discussion points
After completing the prescribed reading consider the following
questions. Discuss your answers with a peer or your educator.
1. What are the key questions you would ask about the pregnancy,
when a pregnant woman presented to triage?
2. Acutely unwell pregnant women may remain haemodynamically
stable until a sudden deterioration in condition takes place.Why
may there be a delay in changes to vital signs in pregnant women? 78
CHAPTER 10: MEDICO–
LEGAL ISSUES
Statement of purpose
The purpose of this chapter is to outline the legal responsibilities
associated with the professional practice of triage.
Learning outcomes
After completing this chapter, participants will be able to apply
medico–legal concepts to triage practice.
Learning objectives
• Discuss the role of education and supervised practice in relation to triaging; and
• Describe the medico–legal responsibilities of the nurse
performing the triage role including:
– Informed consent 79
– Duty of
care – Negligence
– Documentation
– Confidentiality
– Preservation of forensic evidence.
Key points
• Nurses performing the role of triage must have appropriate
education and supervised practice prior to practicing
independent triage.
• Documentation must be accurate and contemporaneous.
• There should be clear understanding of duty of care.
• Nurses must appreciate the importance of re-triaging.
• Policies and protocols should be readily accessible for the nurse
performing the triage role.
Content
Role of the Triage Nurse
A nurse performing triage must have an appropriate level of
knowledge and skills to perform the role. Nurses have a legal and
professional duty to perform the role of Triage Nurse utilising a
systematic approach.
Emergency Nurses, as professionals, are accountable for their
practice.The accountability comes from the utilisation of available
protocols, the completion of the correct documentation, and
adherence to standards and quality guidelines. Protocols ideally help
in the maintenance of a consistently high standard of care at the
institution and can be utilised if necessary to provide evidence of the
clinical practice encouraged at the health care facility.
The physiological discriminators and Australasian Triage Scale (ATS)
are examples of the guidelines that are available for the nurse to
utilise. It is not assumed that following protocols blindly will protect
the nurse from any legal liability.With this in mind, consideration
should also be given to the autonomy of the role, with use made of
the Triage Nurses’ independent judgement for each triage episode,
80 and the ability to utilise his or her expertise to individualise the
assessment of the patient.
Protocols should be viewed as the minimum standard of care
required to be delivered. Position statements that describe the roles
and responsibilities of the Triage Nurse including the minimum
practice standard have been produced by the professional bodies.
All nurses should know some basic legal principles, which include
consent, the elements of negligence, definition and sources of the
standards of care, and how policies and guidelines can influence
practice.There is an expectation that the nurse performing the role of
the Triage Nurse will have had adequate experience, training and
supervision to perform the role.The employing institution also has a
responsibility to ensure that the staff are adequately prepared to
perform the role.
Consent
The five elements of consent are as follows:
1. Consent must be given voluntarily.
2. A person must have the legal capacity to give consent.
3. Consent should be informed.
4. Consent must be specific.
Department of Health and Ageing – Emergency Triage
5. Consent must cover what is actually done.
Department of Health and Ageing – Emergency Triage
The absence of any one element renders the consent invalid.
Consent may be given in several ways:
• Implied consent: Implied consent is the most
straightforward.With implied consent, by virtue of the patient
presenting at the triage area to be assessed does
not necessarily imply consent, but consent is often implied by the
patient’s behaviour. This implied consent becomes less defined if
the patient is confused or unable to communicate for any other
reason.
• Verbal consent:This form of consent is more valid than implied
consent. For example, if the Triage Nurse states that he or she
is going to ask the patient a couple of questions, and the
patient agrees to this, this implies verbal consent.
• Written consent:This form of consent is not something that is
necessarily obtained by the Triage Nurse during his or her
assessment, however there should be awareness of the local
policies and procedures regarding obtaining of
written consent.
Duty of care
By engaging with a patient as they present to the ED, the Triage
Nurse enters into a health professional–patient relationship.The
nurse shares the responsibility of the hospital to ensure that
patients who present to the ED are offered an appropriate
assessment of their treatment needs. 81
A ‘duty’ is an obligation that is recognised by law, and the nurse’s
duty to a patient is to provide the same level or degree of care that
would be employed by a nurse practising under similar or the same
circumstances.The Triage Nurse then has an obligation to try to
protect the patient from any foreseeable harm or injury ensuring a
reasonable standard of care.This reasonable standard of care may be
informed by policies such as the Minimum Standards for Triage and
other documents such as the Australian Nursing and Midwifery
Council (ANMC) competencies.
Scales such as the ATS are also utilised to guide decision-making,
remembering that the ATS are guidelines for care.
There are certain circumstances when the Triage Nurse may be
forced to rapidly detain a patient because, if they leave they pose a
risk of harming themselves or others in the community. Such action is
covered by legislation (which is different in different jurisdictions) and
may be initiated under the principle of necessity under common law.
It is important that such circumstances are immediately referred to
the senior clinician on duty.
The proportion of patients who do not wait for medical treatment in
EDs may be up to 20 per cent of presentations. This is regarded as
representing a failure to access the health system. Patients may
choose to leave the hospital without being seen by the medical staff
in the ED, and if the patient is competent the Triage Nurse cannot
restrain them. However, the Triage Nurse has a responsibility to
warn the patient of the consequences of such a decision, and
appropriate documentation recording this decision should be
completed by the patient and witnessed.
However, patients who have cognitive impairment from drug use,
alcohol use or mental illness are at risk from adverse events in such
situations.The Triage Nurse must therefore consider their duty of care
in such cases.
The Triage Nurse must be aware of his or her responsibilities with
these patients and abide by any local policies or protocols.
Negligence
Negligence laws vary between states and have recently undergone
significant changes. Nurses have a responsibility to behave in a
reasonable manner. If there is any breach from this responsible
approach which results in some type of injury to another, this breach
constitutes negligence.
For negligence to be proven it requires the establishment of all of the following
elements:
• Duty to meet the standard of care
• Breach of the duty to meet the standard of care
• Breach of that duty which causes foreseeable harm
• Causing actual harm and injury
• Causing loss.
82 Documentation requirements
Communication with and by the staff leads to increased information
shared and clear advice given. Medical records are a method of
communication for health care team members and are a
contemporaneous record of events.They must be accurate, clear and
succinct. It is also expected that the records will be easily accessible and
able to be understood.
Documentation of each interaction between the nurse performing
triage and the patient and/or significant others are another area of
accountability for practice.The Australasian College for Emergency
Medicine (ACEM) is clear in its guidelines about the minimum
information that is required to be recorded for any triage episode.
Documentation standards that are required by ACEM are:
• Date and time of triage assessment
• Name of the Triage Nurse
• Chief complaint/presenting problem
• Limited relevant history
• Relevant assessment findings
• Initial triage category allocated
• Re-triage category with time and reason
• Assessment and treatment area allocated
• Diagnostic, first aid or treatment initiated at triage.
Any change in the patient’s condition should be documented clearly.
This documentation should include the time of the re-triage, the
reason for the re-triage and who was responsible for the
performance of the re-triage. (See ‘The Challenge of Triage’ on page
33 of Chapter 4.)
The Triage Nurse should be aware of the management systems in
place at the individual institutions to facilitate this documentation.
Similarly, if it is the practice of the institution to transfer the
care of patients to other health care providers such as general
practitioners, accurate and concise
documentation of any treatment administered and any recommended
course of action should be made.
Some patients choose to leave prior to medical assessment. If such a
patient advises the Triage Nurse they are not waiting, the Triage
Nurse should document this decision, as well as any advice given to
the patient, including possible adverse outcomes.
Confidentiality
Health professionals must maintain any information that has been provided
in-confidence to them. It is also expected that the patient is in receipt
of privacy from health professionals. Safeguards are in place to
protect patient’s information.These include health legislation at both
federal and state level.
The Triage Nurse also has a responsibility to ensure the patient’s
privacy is respected both during the triage assessment and while the
patient waits in the waiting room.The hospital policy regarding 83
patient’s privacy and rights should also be readily accessible to the
Triage Nurse.
A health care professional is obliged to treat the patient’s medical
information as private and confidential. However, in certain
circumstances there is a legal requirement to override a patient’s
privacy and confidentiality; for example, children at risk.
Otherwise, a breach of a patient’s privacy constitutes a breach of the duty of care.
Mandatory reporting responsibilities
If there is any suspicion that a child or children may be in need of
care or may be being maltreated, the nurse has a legal responsibility
to report it to the relevant authorities and refer to their jurisdiction.
Although this reporting may not occur from the triage desk, the
nurse needs to be aware of the legal requirements and of the
procedures and documentation requirements of the hospital, in
order to fulfil these obligations.
Preservation of forensic evidence
Nurses performing the triage role must be familiar with the hospital’s
procedures for dealing with the preservation of forensic evidence
involving a patient who is a possible victim of crime (e.g. rape or
assault).These procedures should include liaison with police officers
as appropriate, with the patient’s consent.
Teaching resources
Further reading
• Australasian College for Emergency Medicine. Guidelines for
implementation of the Australasian Triage Scale in Emergency
Departments. ACEM [Online] 2005
[cited 2007 Feb 2]. Available from:
URL:
http://www.acem.org.au/media/policies_and_guidelines/G24_Impleme
ntation ATS.pdf 2
• Australasian College for Emergency Medicine. Policy on the
Australasian Triage Scale. ACEM [Online] 2006 [cited 2007 Feb 2].
Available from:
URL:
http://www.acem.org.au/media/policies_and_guidelines/P06_Aust_
Triage_ Scale_-_Nov_2000.pdf1
• Australian Nursing and Midwifery Council. National Competency
Standards for the Registered Nurse. ANMC [Online] 2005 [cited
2007 Mar 9]. Available from:
URL: http://www.anmc.org.au/docs/Competency_standards_RN.pdf
• Australian Nursing and Midwifery Council. National Competency
Standards for the Enrolled Nurse. ANMC [Online] 2005 [cited 2007
84 Mar 9]. Available from:
URL: http://www.anmc.org.au/docs/Publications/Competency%20standards
%20EN.pdf
• Australian Nursing and Midwifery Council. Code of
Professional Conduct for Nurses in Australia. ANMC [Online]
2005 [cited 2007 Mar 9]. Available from:
URL: http://www.anmc.org.au/docs/Publications/ANMC%20Professional
%20Conduct.pdf
• Bromfield L, Higgins D. National comparison of child protection
systems. Child Abuse Prevention Issues 2005 Autumn;22.
• College of Emergency Nursing Australasia.Availabe from: URL:
http://www.cena.org.au
• Wand T. Duty of care in the emergency department. International
Journal of Mental Health Nursing 2004; 13(2): 135–9
This paper explores issues that relate to the management of
deliberate self-harm in the ED from a New South Wales perspective.
Tim Wand is a nurse practitioner in the ED at the Royal Prince
Alfred Hospital.
Teaching strategies
Discussion points
1. Join with a colleague and role-play patient presentations to
triage. Attempt to be as varied in your complaints as
possible.Your colleague is to perform an accurate and succinct
triage assessment, including the documentation of the
assessment
and the utilisation of the ATS. The documentation should take into
consideration the objective assessment.
2. Identify what the requirements are of the health care facility for
the documentation of patients who choose not to wait for
treatment or who leave at their own risk.
3. Be familiar with the triage area including any extra resources such
as protocols that may be available.
4. Observe an experienced nurse performing the triage role,
including review of documentation.
5. Identify the policies/protocols for the individual health
care facility that demonstrates the standard of care.
85
6. Use the following patient scenarios as a basis for discussion in tutorial groups.
Patient scenarios
1. John Oliver was a 25-year-old man who was a regular night
clubber and had spent the night before out with friend until the
early hours of the morning. Later
that day he attended the local ED and was complaining of a
headache. He was accompanied by a friend who waited with him in
the waiting room. He was assessed as an ATS category 4 and was
seated in the waiting room accompanied by his friend. After two
hours, the friend spoke to the Triage Nurse about his concerns for
his friend, but John made the decision not to wait to be seen by a
doctor and went home. The next day he was found to have died at
home.
Suggested areas for discussion include:
• The identification of the role/responsibility of the Triage Nurse to
stop John from leaving the ED;
• The documentation of any interactions with the patient or his friend; and
• The procedure of documentation at the hospital.
2. A five-year-old girl is brought in by ambulance with moderately
severe asthma. She is accompanied by her 14-year-old brother
who is translating for her. Her mother is at home, being unable to
attend as she is 38 weeks pregnant (and is looking after a three
and seven year old).What issues does this presentation illustrate?
Why?
3. A 15-year-old girl presents to triage in the presence of police,
smelling of alcohol, stating that she was raped in the toilets of
the local club. Outline your responsibilities as a Triage Nurse and
describe local practices to manage this case.
4. A six-year-old presents with his mother with a deep laceration
to his arm. His mother states that he fell over in the park.The
wound is not actively bleeding, and you assign him an ATS
category 4. You place him in the waiting room and 30 minutes
later they are gone.You phone the mobile number they have
given
and find that it is disconnected.What do you do next?
5. A woman aged 40 is brought in by ambulance, having called it
herself. She reports feeling suicidal, and states that she has the
necessary tablets and that she wrote
a suicidal note. But she says she changed her mind and wants help.
She identifies multiple recent stressors.
(a) How would you go about triaging this person?
(b) Someone else has triaged the woman as an ATS category 3 and
placed her in the waiting room.You go to call the patient but she
has left.
6. Information about a patient is requested of the Triage Nurse by:
86 • A police officer
• A child protection worker
• The media
• The patient’s employer
• The driver of the other car involved in the accident
• A concerned bystander or witness to the accident
• The patient’s parent
• A relative of the patient
• Another member of the hospital staff.
(a) What are your responsibilities?
(b) How would you deal with this situation?
7. Identify the risks associated with the note ‘DNW’ (did not wait)
being the only documentation by a Triage Nurse of a patient
who did not wait to be seen for medical treatment.
CHAPTER 11: CONSOLIDATION
Statement of purpose
The purposes of this chapter are to:
• Apply the principles learnt in Chapters 1–10 to a set of 63 triage scenarios;
• Use the tools contained within the ETEK to assist decision-making; and
• Assess your own level of decision making consistency by
comparing your performance with the expected triage
category for this scenario set.
Learning outcomes
After completing this chapter, participants will have consolidated the
principles learnt in Chapters 1–10, and will be familiar with the
application of the ATS guidelines to actual occasions of triage.
Learning objectives
Choose the most appropriate ATS category for each of the 63 triage scenarios.
87
Teaching resources
Australasian College for Emergency Medicine. Guidelines for
Implementation of the Australasian Triage Scale in EDs. ACEM [Online]
2005 [cited 2007 Feb 2]. Available from: URL:
http://www.acem.org.au/media/policies_and_guidelines/G24_Implementa
tion_ATS.pdf
Mental Health Triage Tool – Table 5.1, page 43.
Paediatric Triage Tool – Table 8.2, page 68.
Teaching strategies
This activity will take approximately two hours, of which 30 minutes
should be spent discussing answers and obtaining feedback on
performance for the scenario set.
For each triage scenario, select the ATS category you think is most
appropriate by ticking the box; chose one option only. Make notes in
the comments section to justify your decisions.When you have
finished, compare your answers with the answer guide (see Appendix
E).
Discuss any disagreements with your triage instructor.
Triage scenarios
1. Ebony is a four-month-old girl who is brought to the ED by her
mother at 4.00 pm. Her mother states that the child has had
difficulty breathing for two days and has been worse overnight.The
child has been coughing and feeding poorly. Her fluid intake has
been approximately half that of a normal day and she has had a
decrease in the number of wet nappies. She has a moist-sounding
cough and no audible wheeze. She is tachypneic with a respiratory
rate of 60 breaths per minute. Examination of her chest shows
mild use of accessory muscles. On auscultation she has an
expiratory wheeze. Her skin is pink and she has moist mucous
membranes.
1 2 3 4 5
Comments:
2. Laura is a 10-year-old girl who presents to the ED at 11.00 pm with
her older sibling saying that she has had abdominal pain for the
past few hours. She indicates that the pain is across the centre of
her stomach and paracetamol has not helped. She complains of
88 nausea and says that she has vomited once since the onset of
pain.
When asked, she states that she has had normal bowel motions. She is able to
give
her own history while leaning over onto the desk, holding her stomach. Her skin is
8
pink and she is not short of breath.
1 2 3 4 5
Comments:
3. Graham is a 55-year-old male who presents to the ED accompanied
by his partner. He states that he has been ‘bleeding from the back
passage’ since the previous night. He is very anxious about the
bleeding and reports that it was ‘bright red’ in colour and ‘filled the
toilet bowl’ on two occasions. His blood pressure is 155/100; his
heart rate is 102 beats per minute; his respiratory rate is 20
breaths per minute.
1 2 3 4 5
Comments:
4. Louisa is a 24-year-old female who presents to the ED with her
friend after ‘fainting’ in the toilet at home. She is complaining of
left-sided abdominal pains, which she has had ‘on and off’ for
several months. She previously attended the ED two weeks
ago for the same problem. An abdominal ultrasound was
performed at that time but identified no abnormalities. She rates
her pain as ‘six out of ten’. Her heart rate is 82 beats per minute
and her respiratory rate is 18 breaths per minute. Her skin is cool
and dry. She looks pale and uncomfortable.
1 2 3 4 5
Comments:
5. A mother presents to the ED at 9.20 pm with her nine-week-old
son, Christopher, stating that he has had a fever since 4.00 pm
that afternoon. She gave him paracetamol at 5.00 pm. She says
that he normally vomits after feeds but has vomited once this
evening between feeds. Christopher is breast-fed; he has fed less
frequently this evening. Christopher’s mother also informs you
that he had his first immunisation two days ago. He is in his
mother’s arms and is crying. He appears slightly pale. His hands
are warm but his feet are cold. Capillary refill is about two 89
seconds and he has moist mucous membranes and normal skin
turgor. His anterior
9 fontanelle is not bulging.
1 2 3 4 5
Comments:
6. Kimberley is 32 years of age. She was sent to the ED following an
accident at work. She was carrying a pot of hot oil and slipped,
spilling it on her upper legs. She immediately removed her
clothing and stood under a cool shower for 15 minutes.
On arrival in the ED she is in considerable pain (‘nine out of
ten’).You estimate that she has approximately eight per cent burns
to her anterior thighs. Her heart rate is 110 beats per minute and
her respiratory rate is 24 breaths per minute.
1 2 3 4 5
Comments:
7. Michaela is a three-week-old infant who is brought to the ED at
9.30 pm by her parents. She has been referred by her local doctor.
Her parents state that Michaela has been feeding poorly for
several days and that her weight gain has been poor. The infant
seems lethargic.The parents have not noticed a fever.The infant is
sleeping in her mother’s arms and her skin is pale. Her peripheries
are cool and her eyes slightly sunken. Painful stimulus is required
to wake the child, who then wriggles and cries vigorously.
1 2 3 4 5
Comments:
8. Toby is an 18-month-old boy who presents to triage at 6.00 pm with his parents.
They state that he has been ‘unwell’ for two days; he started vomiting 48 hours
ago, developed diarrhoea yesterday and has had seven loose stools
today. He has had episodes of ‘crying and drawing up his legs’. He
is drinking small amounts. He appears lethargic and uninterested in
his surroundings. He is pale and his capillary refill is approximately
three to four seconds.
1 2 3 4 5
90
Comments:
9
9. Edward is a 36-year-old male with a past history of alcoholism. He
presents to triage at 5.30 pm. He has a referral letter from the
nearby drug and alcohol service and
an escort.The referral letter states that the patient has ‘suicidal
ideation and homicidal thoughts’. The letter requests a psychiatric
assessment and states that the patient is ‘possibly experiencing
alcohol withdrawal’. He states that his last drink was at 9.00 am.
1 2 3 4 5
Comments:
10. Rae, a 24-year-old university student, comes to the ED with a friend. She has a
four-hour history of generalised abdominal pain now localised to the
right iliac fossa. She has vomited twice and had one episode of
diarrhoea about two hours ago. Her heart rate is 92 beats per
minute and her temperature is 38.2°C.
1 2 3 4 5
Comments:
11. A father presents to the ED at 8.00 pm with his three-and-a-half-
year-old daughter, Savannah, stating that she has had a sore
throat for ‘a day or two’. It started with a runny nose and a fever,
and then yesterday she began complaining of a sore throat. She
has no cough or stridor, she demonstrates no shortness of breath
and her skin is pink and warm.
1 2 3 4 5
Comments:
12. Baz, 34 years old, was installing a ceiling fan with the assistance of a friend in
his own home. He received a 240 volt charge to his right hand,
and was thrown back against the roof. His friend immediately
switched the power off and called an ambulance. Baz had a brief
period of loss of consciousness, but was alert when
the ambulance crew arrived. His heart rate is 80 beats per minute and irregular;
his respirations are 20 breaths per minute. He has a five centimetre blackened
area to his right hand. No exit wound is seen.
1 2 3 4 5
Comments: 91
1
13. Hannah is a 41-year-old woman who presents via ambulance
with an altered conscious state following collapse. She is 30
weeks pregnant (G3P1) and is normally well. She was out
shopping with a friend when she suddenly collapsed.
Ambulance officers report a fluctuating conscious state. At the
scene she tolerated an oropharyngeal airway but spat it out en
route. She is in a lateral position on
the ambulance trolley with supplemental oxygen via a mask. Her
respiratory rate is 10 breaths per minute. Her SpO2 is 93 per cent;
her heart rate is 130 beats per minute. Her skin is pale, cool and
moist. Her blood pressure is 190/110. Her Glasgow Coma Score is
10 out of 15. Her temperature is 36.3°C.
1 2 3 4 5
Comments:
14. Mr J is a 74-year-old man who is brought to the ED by ambulance
at 5.10 am. He has acute shortness of breath and a history of left
ventricular failure. His heart rate is 112 beats per minute and
irregular, his blood pressure is 180/100 and his respiratory rate is
30 breaths per minute, with accessory muscle use. His SpO2
is 89 per cent, but the pulse oximetry display is giving a poor trace. Oxygen is
being
administered at 100 per cent via bag-valve-mask. Mr J is trying to
remove the mask and is very agitated.
1 2 3 4 5
Comments:
15. Bo is a 16-month-old boy who presents to triage at 11.00 am with
his mother. She states that he has had ‘a cold for over a week’
which ‘has not improved’. Since last night he has had a fever and a
cough and has seemed ‘more congested’. He was restless over
night, is tired today and is drinking less than usual. He is resting
against his mother and doesn’t protest when examined. No cough,
stridor or grunting is heard. He is tachypneic and demonstrates
mildly increased work of breathing. His skin is flushed and warm.
92 His capillary refill is less than two seconds and his mucous
membranes are moist.
9
1 2 3 4 5
Comments:
16. Luka is a nine-year-old boy who presents to triage with his
father at 3.00 pm. He has an injured elbow as a result of a fall
playing football. He is distressed and is clutching his arm, which
is in a sling. He tells you that his pain is ‘ ten out of ten’.
His left elbow is markedly swollen and deformed. He has a strong
radial pulse, and sensation distal to the injury is intact. He is pale,
slightly diaphoretic and tachycardic.
1 2 3 4 5
Comments:
17. Albert, 62 years old, often attends your ED.Today he says he is
constipated. His bowels have not opened for ‘at least two weeks’.
He says he has pain and feels bloated.When you ask him to score
his pain he is not sure what to say and just answers ‘it’s really
bad’. His vital signs are within normal limits and his skin is warm
and dry.
1 2 3 4 5
Comments:
18. Sebastian is a 16-year-old boy who is brought to the ED by a
passer-by, who found him crying and banging his head against the footpath in a
small laneway. After bringing Sebastian to the triage the accompanying adult
leaves the ED. Sebastian has superficial lacerations to both wrists, and is
dishevelled and unkempt. He is upset about having being brought to the ED, and
is saying,‘just leave me alone – why don’t you just piss off’. He admits trying to
hurt himself, and says that he will do so again as soon as he can.
1 2 3 4 5
Comments: 93
3
19. Anne-Marie is a 22-year-old female who is brought to the ED by
her flatmates, who are concerned about her bizarre behaviour. She
had been talking to herself for several days, turning the television
off and on because it is sending her messages, yelling out at night
and not sleeping. Her flat mates are concerned that she will come
to some harm without help.
1 2 3 4 5
Comments:
20. Mohammed is a 24-year-old Somali man who is brought to the
ED by police. He is crying and lying on the floor, rocking. He
smells of alcohol, and police say he is a
refugee who has recently been released from a detention centre. He
has committed no crime, but was apprehended ‘directing traffic’ in
the middle of a busy city highway.
1 2 3 4 5
Comments:
21. Damien is a 36-year-old male who is brought to the ED by his
friend. He has had a recent marriage break-up, which involved a
lengthy custody and property court case. He has had symptoms
of depression for several weeks, including low mood,
ruminations, poor sleep and appetite, feelings of hopelessness
and agitation. Since
receiving the outcome of the Family Court hearing three days ago,
Damien has been using the amphetamine ‘ice’, and is now
‘obsessed with plotting revenge’ on his former spouse. He has been
awake for more than 48 hours, and presents as angry, rambling in
speech, volatile and disordered in his thinking.
1 2 3 4 5
Comments:
22. Chloe is a 15-year-old girl who is brought to the ED from a friend’s
house after taking an overdose.The circumstances are unclear,
however, she admits to having taken 12 paracetamol tablets and
‘some ‘other things’, including alcohol. She is known to the ED,
having presented 12 months ago following an episode of self-harm.
She is cooperative, coherent and not drowsy. Her breath smells of
alcohol.
94
1 2 3 4 5
9
Comments:
23. Leonie is a 29-year-old woman who presents to triage with her
mother. She has had three days of abdominal pain and vomiting.
She tells you she is 32 weeks pregnant (G2P1) and is an insulin-
dependent diabetic. Her main reason for coming
to the ED is that she couldn’t get an appointment with her
obstetrician and the pain is ‘worrying’ her. She appears a little
short of breath and her respiratory rate is 28 breaths per minute.
Her SpO2 is 98 per cent. Her heart rate is 128 and her skin is pale,
warm and dry. She is alert and oriented and her Glasgow Coma
Score is 15 out
of 15. Her temperature is 37.2°C (tympanic).
1 2 3 4 5
Comments:
24. Paul is a 47-year-old male. He has a painful left shoulder, and
received treatment in the ED for the same problem two days
ago.There is no history of injury, but Paul tells you that his
shoulder is stiff and keeps ‘seizing up’. He tells you that he was
prescribed some pain killers that worked initially, but that the pain
is back and is ‘much worse now’. He is crying in pain. His left
hand is pale and cool; a week radial pulse is noted. His right hand
is pink and warm.
1 2 3 4 5
Comments:
25. Gillian is a 26-year-old woman who presents via ambulance
with palpitations. She is 34 weeks pregnant (G1P0) and is
normally well. She tells you that she was out
shopping when her palpitations started. She does not have any associated chest
pain
or shortness of breath. Her respiratory rate is 20 breaths per minute. Her SpO2 is 98
per cent. Her heart rate is 108 beats per minute and her blood pressure is 120/80.
Her skin is pale, warm and dry. Her Glasgow Coma Score is 15 out of 15.
1 2 3 4 5
5 Comments: 95
26. Mal is a 28-year-old male who presents to triage saying that he
has been bitten by ‘some sort of insect’. He was clearing rubble
from a building site about two hours ago when he felt a sudden
burning sensation in his right hand. He said ‘I flicked something
off but I didn’t see what it was’. Over a period of two hours his
right arm has become increasingly painful and he is sweating. He
is complaining of a frontal headache. He is alert and oriented to
time, place and person. His heart rate is 98 beats per minute and
his respiratory rate is 22 breaths per minute.
1 2 3 4 5
Comments:
27. Thuy, a 44-year-old woman, presents to the ED with back pain.
She has had the problem on and off for many years.This current
episode was brought on after lifting a light shopping bag from her
car four hours ago. She has taken Nurofen with little improvement.
Currently she has no general practitioner so she ‘didn’t know
where else to go when the pain happened’. Her vital signs are
within normal limits and she is not sure how to rate her pain but
says it is ‘very bad’.
1 2 3 4 5
Comments:
28. Patty is a 53-year-old female who presents to triage complaining
of right-sided abdominal pain. She states that the pain has been
constant for two days now. She has not had any nausea or
vomiting. She tells you that the pain is worse when she is sitting
still. She states that she has had this pain before and that her
doctor thought it might be gall stones. Prior to coming to the ED
she took two paracetamol with minimal effect. She rates the pain
as ‘five out of ten’. Her blood pressure is 145/84, her heart rate is
96 beats per minute and her respiratory rate is 18 breaths per
96 minute. Her temperature is 36.4°C.
1 2 3 4 5 9
Comments:
29. Emil is a five-year-old boy with a seven-day history of diarrhoea
and vomiting. He presents to the ED with his mother at 9.30 pm.
He has been unable to keep food or fluids down today. He is pale,
lethargic and drowsy. His heart rate is 124 beats per minute and
his respiratory rate is 20 breaths per minute.
1 2 3 4 5
Comments:
30. Catherine is a four-year-old girl who is brought to the ED at 4.30 pm with a
12-hour onset of being unwell. In the past four hours she has
developed a petechial rash on her abdomen. She also has a runny
nose and a fever (her temperature is 37.8°C per axilla). She has
been tolerating sips of oral fluid but now seems drowsy.
1 2 3 4 5
Comments:
31. Lee is a 20-year-old female who presents to the ED with her
mother. Her mother reports that she has had paranoid
hallucinations and that since yesterday she has not taken any
fluids. She states that her reason for not drinking is that she
believes that there are ‘spiders and poison around’.
1 2 3 4 5
Comments:
32. Candy, a three-month-old female, presents to the ED with her
mother. She has been referred by the maternal child and health
nurse. According to her mother, the infant has been ‘crying a lot’
and has ‘bad colic’.The baby was born prematurely at 36 weeks,
and was delivered by emergency caesarean section due to
preeclampsia.
Since birth, the baby has gained weight and her mother says that apart from the
colic she ‘is doing OK’.When you examine the baby you note green/yellow bruising
and red welts on her upper arms.
1 2 3 4 5
Comments:
7 97
33. Nathan is a 45-year-old man who presents to the ED with his wife
and child. He asks to see a psychiatrist because he has been
having problems managing his anxiety about his work situation,
and he doesn’t know how to get a referral. He reports that he once
saw a psychiatrist, four years ago, and that it helped him sort out
his troubles, but that he can not remember the doctor’s name. He
is on no medication and has no active thoughts of harming himself;
he says that he ‘just needs to sort out his anxiety’.
1 2 3 4 5
Comments:
34. Brian is a 39-year-old male who walks to the triage desk. He says
he fell in his driveway and now has left shoulder pain. On
examination his shoulder is very swollen and painful on
movement. His arm is already in a sling. His left hand is warm and
a radial pulse is present.
1 2 3 4 5
Comments:
35. Bianca is 24 years old. She has a history of a perianal abscess,
which underwent drainage two days ago. She continues to have
pain (‘six out of ten’) and was seen by her local doctor today. She
has taken Panadeine Forte with no relief and is also on oral
antibiotics.
1 2 3 4 5
Comments:
36. Craig is an 18-year-old male patient who presents saying he
feels ‘suicidal’ and requesting admission. He makes a verbal
threat to ‘cut up’ if he is not admitted.
1 2 3 4 5
Comments:
37. Karen, a 36-year-old female, presents to triage accompanied by
a social worker. She has come from the plaster clinic. She has
98 increasing pain in her left foot from a
fractured right fibula which she sustained yesterday morning. A lower-leg plaster
was 9
applied in the ED last evening.The social worker tells you that the
patient has a history of depression and has said that she wants to
‘end it all’.
1 2 3 4 5
Comments:
38. Ida is a 66-year-old female who presents to the ED alone. She
states that she is on Aropax and is having ‘suicidal’ ideation. She
tells you that she has two possible plans to harm herself. She says
she is having an anxiety attack and reports poor sleeping and
eating patterns for the past two weeks.
1 2 3 4 5
Comments:
39. A 52-year-old male presents to triage. He has a history of schizophrenia. He
is currently on medication for his condition but can not recall the
name of the medication, or the name of his case manager. He says
that he has been having suicidal thoughts and that ‘there are
voices’ urging him to ‘step in front of a train’.
1 2 3 4 5
Comments:
40. Rohan, a 50-year-old male, has been brought to the ED by the
district nurse. The nurse states that he has a history of alcohol
abuse and that he is feeling ‘suicidal’. She notes also that over the
past week he has been neglecting his general care. The patient
has a history of an intracerebral bleed (two years ago) and he is
deaf.
1 2 3 4 5
Comments:
9 41. While playing volley ball, Gary, 47, hurt his left wrist. He 99
has a good range of movement but reports pain when asked
to rotate his left hand.
1 2 3 4 5
Comments:
42. Janine is a 56-year-old woman who presents to the ED with her
partner at 2.30 am. She has pain in the epigastric region which has
been increasing since yesterday.The pain radiates to her lower
abdomen and she says that she has been vomiting clear fluid
tonight. Her bowels last opened two days ago. She is on Oridus and
has a history of hypertension.
1 2 3 4 5
Comments:
43. Mr D, 84, has a chronic leg ulcer.The district nurse has sent him
to the ED because she believes the wound is infected. Mr D has a
history of hypertension and ischemic heart disease. He lives with
his daughter, who normally helps him out with his daily living, but
she has gone to Queensland for a holiday.The wound is covered
when you see him, but the bandage is soiled with what appears to
be haemo-serous ooze. His temperature is 35.9°C and his vital
signs are within normal limits.
1 2 3 4 5
Comments:
44. Nic, a 38-year-old arborist, has cut his left arm with a chain saw.
He was brought to the ED by a workmate. He has a deep
laceration of about ten centimetres to the inner aspect of his
arm.The wound was bleeding ‘quite a bit’, but the blood loss has
been controlled with a firm bandage. He tells you that the wound
is ‘not that painful’, but he looks pale and is sweating. His heart
rate is 84 beats per minute and his respiratory rate is 20 breaths
per minute. His workmate reports that the dressing was changed
once, half an hour ago, because it was soaked with blood.
100
1 2 3 4 5
1
Comments:
45. Liam is a 23-year-old male who presents to triage after being seen
by a locum doctor. He is backpacking around Australia and has
been staying in a boarding house near the hospital. His partner has
brought him to the ED. He has a six-hour history of fever and
lethargy. He has been vomiting, and complains of a headache.The
doctor gave him intramuscular Maxalon, with some effect. His
temperature is 38.4°C, and his partner points out a fine petechial
looking rash on his torso. He is drowsy but oriented to time, place
and person.
1 2 3 4 5
Comments:
46. Ashley, a 23-year-old university student, fell off her bicycle two
days ago and was seen in another ED. She is complaining of
stiffness and pain to her left wrist. Her left hand is swollen but she
has full range of movement; her left hand is pink and warm.
1 2 3 4 5
Comments:
47. Remo is a 43-year-old male who presents with a two-week history
of right renal stones. He now has pain, which he describes as
‘colicky’ in nature. He rates the pain as ‘four out of ten’. He has
had no pain relief today.
1 2 3 4 5
Comments:
48. Angie is a 27-year-old woman who presents via ambulance
following a high-impact motor vehicle accident. She is 38 weeks
pregnant (G2P1) and is normally well. She was a passenger in a car
that collided head-on with another vehicle in an 80 kph zone.The
ambulance officers report significant damage to both vehicles.
Angie was wearing a seatbelt and the passenger airbag was
deployed. She has good recall of events but complains of a painful
chest and abdomen and has visible seatbelt marks. She also has a
facial abrasion and lacerations to both her knees. She has a
cervical collar on; oxygen is at 10 litres per minute via mask and
500mls crystalloid fluid in
progress intravenously. Her respiratory rate is 28 breaths per
minute, her SpO2 is 93 per cent and her heart rate is 134 beats per
minute. Her skin is pale, cool and dry. Her Glasgow Coma Score is
15 out of 15. Her blood pressure is100/R. Her pain is ‘six out of
ten’. She has no PV loss. 101
01
1 2 3 4 5
Comments:
49. Norm is a 60-year-old man who arrives at triage at 9.20 am. He is
ambulating using a walking stick.When asked what is wrong he
points to his abdomen and chest and says,‘This is as tight as billy-
o. I got stirred up yesterday – I had a barney with a bloke up home,
and then the tightness got worse, like a vice’. On examination you
find that his heart rate is within normal limits and is regular. His
skin is warm and dry. He is not short of breath. His SpO2 is 95 per
cent on room air.
1 2 3 4 5
Comments:
50. Ann is a 16-year-old female who walks to triage with her mother.
She reports that she injured her left wrist while playing volley ball.
On examination you note good range of movement but she still has
some pain. She says the pain is ‘three out of ten’.
1 2 3 4 5
Comments:
51. Mr A is a 54-year-old man who has been sent to the ED by his
local doctor. He is unsteady on his feet and requires the
assistance of his son to walk. His referral letter reads:
Dear Doctor,
Please assess this man who was recently admitted to your
hospital with left renal calculi. He has been complaining of
dizziness and headache for several days. No focal weakness,
visual disturbance or confusion. Seen for same 2/7 ago no
improvement with Stemitil. PMx, IHD, NIDDM, renal calculi,
hypertension. Blood pressure: 215/130. Please assess.
Via translation through his son, Mr A tells you that he is ‘very dizzy’,
feels ‘weak all over’, has pain in his back and his abdomen and has
102 vomited twice today.
1 2 3 4 5
Comments:
52. Jake, 46 years of age, presents to triage with his carer. He is
crying because he has abdominal pain and has a recent history of
a small bowel obstruction (six months ago). Jake has an intellectual
disability, and lives in a community residential unit with three
other adults and supervisory staff. His carer says that he is
‘normally able to attend to his activities of daily living under
supervision’, and that he ‘usually tolerates a lot of pain before he
will let staff know he is unwell’. In fact, his carer says that ‘last time
he was hospitalised he had been ill for quite a while before staff
actually realised that there was a problem with his health’. His
heart rate is 120 beats per minute and his respirations are 26
breaths per minute. His skin is pale, cool and moist.
1 2 3 4 5
Comments:
53. Jane is a 17-year-old girl who was sent to the ED by her local
doctor. On her way home from school her boyfriend noticed that
she had become drowsy,‘she kept asking where she was, and
appeared disoriented’. She was seen by her local doctor who told
her to ‘go straight to the ED’. He did not provide her with a letter
of referral. Her Glasgow Coma Score is 14 out of 15.
1 2 3 4 5
Comments:
54. Jonny, 34 years of age, has an abscess under his tooth. He presents to the ED at
1.30 am. He is in pain (‘six out of ten’) despite having taken
Panadeine and Nurofen regularly. He has an appointment with his
dentist tomorrow, but has not been able to sleep because of the
pain. He is afebrile.
1 2 3 4 5
Comments:
103
55. Rose is a 47-year-old female who presents to triage with a
letter from her local doctor. She makes no eye contact when
you speak to her. The letter reads:
Dear Doctor,
Please assess Rose, a 47-year-old woman who lives alone.
She has a history of cholelithiasis and schizophrenia. She has
some burns on her inner thigh which require your attention.
On questioning, Rose tells you that her burns occurred two days
ago, and that they are red and itchy.When you ask her how she
sustained the burns she says she isn’t sure.
1 2 3 4 5
Comments:
56. Adrian is a 13-year-old boy who presents to the ED via ambulance
at 10.00 am.The ambulance officer states that he was hit by a car
with a bull bar, and was thrown several metres. He is complaining
of pain in his neck and legs. He has a cervical collar in place. He
looks pale. He is tachycardic and tachypneic. He answers questions
appropriately and is able to move all limbs on request.
1 2 3 4 5
Comments:
57. Edna is 93 years old. She has been transferred to the ED via
ambulance from a nearby aged care facility. For the past two
weeks she has not been eating much, and yesterday was only
taking small amounts of oral fluids .Today she was found to be
semi conscious and with a fever (39.8°C). She has a history of
ischemic heart disease, heart block and hypertension. She has a
dual-chamber pace maker. She also has a three-year history of
dementia. On arrival her respiratory rate is 28 breaths per minute;
her heart rate is 68 beats per minute. Her skin is hot and moist,
and her Glasgow Coma Score is 9 out of 15.
1 2 3 4 5
Comments:
58. Rodney is 43 years old. He was escorted to the ED by police,
having been apprehended climbing out of a window of an
abandoned warehouse.While trying to escape he cut his right hand
on some broken glass. He has a deep, six centimetre laceration to
the palm of his right hand.There has been minimal blood loss, but
he says he can not feel his right index or second finger at all.
1 2 3 4 5
104 1
Comments:
59. Mr G is a 53-year-old male who walks to the triage desk
unassisted. He is short of breath. He states that he was recently a
patient of this hospital. He has cancer of the liver and had a
peritoneal tap 10 days ago for acites. He also tells you that he
‘needs draining again’. His respiratory rate is 24 breaths per
minute and his heart rate is 92 beats per minute.
1 2 3 4 5
Comments:
60. Heidi, a 17-year-old female, presents to the ED complaining of a sore throat. She
has a hoarse voice and her friend states that she also has muscular
pain to her neck, shoulders and back. She has been unwell for a
few days, but has come to the ED today because she is having
trouble swallowing.You ask her to open her mouth
and note that her breath is foul-smelling. Her tonsils appear to be
covered in pus. Her temperature is 39.4°C.
1 2 3 4 5
Comments:
61. Over the past four weeks Gregory, 56, has attended your ED 14
times.Today he says he has a problem with a tattoo that was
applied ‘by a mate’ two weeks ago.The
wound looks red and is oozing pus. Gregory has a history of alcohol
and intravenous drug use, hepatitis C and type 2 diabetes. His vital
signs are within normal limits.
1 2 3 4 5
Comments:
05 105
62. Larry, 62, stubbed his right big toe on the corner of a
fireplace.The nail has lifted right off and the toe is now covered
with a blood-soaked tea-towel. Larry walks into the ED assisted by
his son. He tells you that he takes Warfarin, so ‘thought it best to
come to hospital rather than see the local doc’. His son tells you
that the tea-towel has not been changed since the injury, but that
there was ‘blood all over the floor’.
1 2 3 4 5
Comments:
63. Carole, 48 years, is brought to the ED by her husband. She is
vomiting and has severe epigastric pain. She ate at a local
restaurant and tells you she thinks that she has ‘food poisoning’.
Her heart rate is 98 beats per minute; her respiratory rate is 26
breaths per minute. Her skin is pale and moist to touch. She says
that the pain comes and goes: she rates it as ‘eight out of ten’ at
the worst point and ‘two out of ten’ at the lowest point’. She has
vomited semi-digested food more than six times in the past hour.
Now the vomit is clear fluid.
1 2 3 4 5
Comments:
Department of Health and Ageing – Emergency Triage
CHAPTER 12: SELF-TEST
Statement of purpose
The purposes of this chapter are to:
• Apply the principles learned in Chapters 1–11 to a set of 92 paper-based scenarios;
and
• Test participants own level of decision-making
consistency by comparing performance with the expected
triage category for this scenario set.
Learning outcomes
After completing this chapter, participants will be able to apply the
principles learnt in Chapters 1–11 to a set of triage scenarios and
demonstrate consistency of triage using the ATS guidelines for the
scenarios in the self-test.
Learning objectives
Choose the most appropriate ATS category for each of the 92 triage scenarios.
106
Teaching resources
Australasian College for Emergency Medicine. Guidelines for
Implementation of the Australasian Triage Scale in EDs. ACEM
[Online] 2005 [cited 2007 Feb 2].
Available from:
URL:
http://www.acem.org.au/media/policies_and_guidelines/G24_Implementa
tion_ATS.pdf Mental Health Triage Tool – Table 5.1, page 43.
Paediatric Triage Tool – Table 8.2, page 68.
Teaching strategies
For each triage scenario, select the ATS category you think is most
appropriate by ticking the box; chose one option only. Make notes in
the comments section to justify your decisions.When you have
finished, compare your answers with the answer guide (see Appendix
E).
Discuss any disagreements with your triage instructor.
Triage scenarios
1. Albert, 44 years, was mowing the lawn on Sunday morning when a
foreign body flicked up out of the mower and into his eye. His wife
drove him to the hospital. On presentation at triage, his eye is
tearing quite a bit, and he still has the sensation that ‘something is
there’. He says he has ‘no pain’ but the eye is ‘uncomfortable’. A
quick check of vision reveals that he has no problems with visual
acuity.
1 2 3 4 5
Comments:
2. Shane is a 30-year-old male who presents to the ED complaining of
having a frontal headache.The patient states he has been thinking
of harming himself and wants to ‘get help’. If he can not sort out
his problems, he says he will ‘go and jump off a bridge’.
1 2 3 4 5
Comments:
107
3. Violet is a 91-year-old female of non-English speaking background.
She is brought to the ED in her son’s car after seeing her local
doctor.You are called to assist her
to get out of the car. Her son tells you she is ‘very sick’.You note that she is able
to transfer to a wheelchair with minimal assistance. According to
her son,Violet fell three days earlier and has bruised her right hip.
She is able to walk, though the hip is very painful. She is not
distressed when seated. Her blood pressure is 150/90, her heart
rate is 88 beats per minute, and her respiratory rate is 20 breaths
per minute. You are unable to ascertain her exact level of pain,
though she tells you she is ‘alright’.
1 2 3 4 5
Comments:
4. Glen, 52 years, presents to the ED with ‘bleeding haemorrhoids’.
He has had this problem ‘on and off for the past few months’, but
now it is ‘getting worse’. He says he has considerable pain when
he opens his bowels and bleeds ‘quite a bit’ (about half a cup at a
time for the last two days). He states that he needs to be seen by
a doctor ‘as soon as possible’ as he considers his problem is ‘an
emergency’.
1 2 3 4 5
Comments:
5. Rebecca is a 17-year-old female who is brought to the ED by her
friends following an all-night party, where she took two tablets of
ecstasy. She can not stop crying, and says she wants to die. She
has had two previous minor overdoses in the past year.
1 2 3 4 5
Comments:
6. Charlie is a 15-month-old boy who presents to the ED via
ambulance at 2 am following an episode of ‘shaking and jerking’
with a loss of consciousness.The episode lasted approximately
four minutes.The ambulance officers state that he stopped fitting
when they arrived but he had been ‘very drowsy’ during
transport.
His mother states that Charlie has never had an episode like this before. During
the night, he had a fever and a runny nose. He has been sleeping
poorly and is a little irritable.The child is flushed and his skin is very
warm. He is tachypneic, but has no use of accessory muscles or
retraction. He is crying and clinging to his mother.
1 2 3 4 5
Comments: 108
7. Dianne is a 67-year-old lady who was out shopping with her
daughter when she slipped and fell on her outstretched hand
injuring the left wrist. She is not distressed by the pain and rates it
as ‘three out of ten’. Her wrist is tender, but not deformed. Radial
pulse is present at 72 beats per minute.
1 2 3 4 5
Comments:
8. Kate is 18 years. She attends triage at 12.30 pm with a work
colleague. Her hand is wrapped in a tea towel and she appears
pale and anxious. She tells you she has cut her hand with a
carving knife. On examination you see a four centimetre
laceration across her left palm.Tendons are on view and the
wound is bleeding slowly. Kate tells you she is feeling quite
nauseous and her pain is ‘seven out of ten’. Movement and
sensation to her fingers are intact.
1 2 3 4 5
Comments:
9. Denise is a 34-year-old female who is transferred to the ED on a
40°C day via ambulance. According to bystanders, she was
attending an outdoor barbeque and ‘collapsed in the
heat’.Witnesses helped her to an upright position, and she was
then observed having a ‘fit’ that lasted approximately two minutes.
She was not incontinent and regained consciousness when she was
placed in the supine position. Paramedics attended and inserted an
intravenous cannula. Her Glasgow Coma Score is 15 out of 15;
heart rate 112 beats per minute and respiratory rate 22 breaths
per minute. Her skin is hot and moist.
1 2 3 4 5
Comments:
10. Justin is a 22-year-old male who comes to the ED concerned
about a mole on his back. He says that his girlfriend advised him to
see a doctor and he is ‘worried that it might be a melanoma’.The
mole is large and irregular in shape; he says it is sometimes itchy.
1 2 3 4 5
109 Comments:
11. A young man is brought in to the ED by ambulance after having
been dragged unconscious and not breathing from the sea in Far
North Australia. He arrives with full CPR in progress and you note
that he has red welts across his chest.
1 2 3 4 5
Comments:
12. Fred, an 84-year-old man, presents to triage complaining of
palpitations and central chest pains. He has a history of ischemic
heart disease, coronary artery by-pass grafts and atrial fibrillation.
He takes his anti-arrhythmic medications regularly and normally
manages well at home.Today his skin is pale, cool and moist, and
his heart rate is 142 beats per minute and irregular.
1 2 3 4 5
Comments:
13. A man states that his three-week-old baby grandson, Kyle, is ‘not
breathing properly’. The baby is wrapped in a bunny rug held by his
grandfather. On closer examination, you note the baby’s eyes are
open and his face is white/grey. Respiratory effort is poor.
1 2 3 4 5
Comments:
14. Robyn is a 38-year-old woman with a history of asthma. She has
required two admissions to the intensive care unit for her asthma
in the past 18 months. She presents to triage at 8.30 pm following
a 22-hour history of wheeze and shortness of breath. She has been
self-administering Ventolin at home but has had a minimal
response despite the use of three nebulisers in the past hour. On
arrival to triage, her respiratory rate is 26 breaths per minute; she
is speaking three-word sentences and has an audible wheeze.
1 2 3 4 5
Comments:
110
15. Caroline is a 45-year-old female who presents to triage
complaining of a ‘cold’ for the past four days. In the past two
days, she has pain in her right upper quadrant. The pain is now
increasing and she describes right thoracic ‘back pain’. Caroline
states that she has no diarrhoea, vomiting or urinary symptoms
but has had ‘difficulty breathing since yesterday’. Her skin is
pale, hot and moist, and she has normal respiratory effort.
Caroline says she has a fever and her heart rate is
112 beats per minute. Her respiratory rate is 26 breaths per
minute and she says that her pain is currently ‘seven out of
ten’.The pain is worse on deep inspiration and movement.
1 2 3 4 5
Comments:
16. Neil is a 74-year-old male who presents to triage following trauma
to his left arm after slipping on a wet floor. He describes tenderness
at his wrist, elbow and shoulder. He rates his pain as ‘three out of
ten’. No obvious deformity of the wrist is noted, but he has a
decreased range of movement. His heart rate is 92 beats per
minute.
1 2 3 4 5
Comments:
17. Harry is a 48-year-old man who regularly attends your ED for
various complications associated with his poorly controlled type 2
diabetes. He has no GP, lives in a special accommodation house
but frequently sleeps out in a local park.Today he presents to triage
complaining of a two-hour history of intermittent left-sided chest
pain that is ‘heavy’ in nature. On further questioning you establish
that his pain came on at
rest and radiates down both arms. His heart rate is 66 beats per
minute, respirations are 20 breaths per minute, and skin is cool and
dry.
1 2 3 4 5
Comments:
18. Mr Wallace, 57 years, works for an energy company reading
gas meters. On his rounds today he was attacked by a dog and
bitten on the upper left leg. On inspection you note six to seven
square centimetres of skin loss.The wound is irregular, fat tissue
is exposed and it looks dirty.There is a small amount of blood
loss. Mr Wallace says the injury is ‘a bit painful’ but he is not
overtly distressed.
1 2 3 4 5
111 Comments:
19. Kira is a seven-year-old girl who presents with a school teacher
having fallen from play equipment. Her mother is on her way to the
ED. She fell onto her right arm and has been complaining of pain
around her wrist. She ‘did not hit her head’ and does not complain
of pain anywhere else. Her arm has been placed in a sling but she
has not received any analgesia. Kira is tearful but states that her
arm is only ‘a little bit sore’.There is a small amount of swelling
around her distal forearm; there is no deformity and no
neurovascular impairment. She demonstrates tenderness over her
distal radius and has a limited range of movement of her wrist. She
has no other signs of injury.
1 2 3 4 5
Comments:
20.A mother presents at midday with her nine-month-old son,
Connor. She describes three days of fever, poor oral intake and a
‘runny nose’. On the first evening of
the episode he vomited once and had two loose stools overnight, but this has
not reoccurred. Connor had 180 ml of fluid this morning (his usual
intake is about 320 ml) and he had a normal number of wet
nappies. He was previously well and
he is fully immunised. He has signs of a runny nose, no cough is
heard and he has no signs of increased work of breathing. His skin
is slightly pale but warm and his mucus membranes are not dry. He
is quiet but looking around at his surroundings.
1 2 3 4 5
Comments:
21. Alanna is a ten-week-old infant who presents with her parents.
She has a two-day history of increasing lethargy and poor feeding.
Her mother indicates that she has become unsettled and less keen
to feed over the past two days. She developed a fever yesterday
and had to be woken for feeds overnight, which is unusual. She
was born at term, has had her first immunisation and has no other
health problems. She does not demonstrate increased work of
breathing but is slightly tachypneic. Her skin is pale and her legs 112
are mottled, a little cool and demonstrate a capillary refill of three
to four seconds. She is lethargic but responds to painful stimuli.
1 2 3 4 5
Comments:
22. A 76-year-old woman, Rita, is brought to the ED by her daughter
who found her wandering in a ‘dazed state’ outside her house.The
patient presents as agitated and confused, is picking at imaginary
things on her cardigan and is unable to give an account of herself.
1 2 3 4 5
Comments:
23. Liz is a 40-year-old woman who presents to triage with fever and
productive cough. She says she is not short of breath and does not
complain of any pain. She is 18 weeks pregnant (G4P3) and is
normally well. Her respiratory rate is 24 breaths per minute, SpO2
is 96 per cent, and heart rate is 98 beats per minute. Her skin is
noted to be pale, warm and dry, Glasgow Coma Score is 15 out of
15, and her temperature is 38.2°C.
1 2 3 4 5
Comments:
24. David is a 40-year-old male who presents to triage complaining of
severe chest pain, saying he is having a ‘heart attack’. He says he
has no history of cardiac problems and his observations are within
normal range. He appears highly anxious and is hyperventilating.
Currently he says his pain is ‘ten out of ten’. His skin is warm and
moist.
1 2 3 4 5
Comments:
113
25. Lionel, 68 years, is transferred to your ED from a nursing home.
He has Alzheimer’s disease and for the past two days has refused
fluids.This morning his carer found him lying on the floor next to his
bed yet the cot-sides were up. She thought that he had probably
fallen because the blankets were also on the floor and he had been
incontinent of urine. Last week he was able to mobilise with a
frame and take himself to the toilet, but for the past two days he
has not had the energy to move at all and has needed assistance
going to the toilet. Since the fall he cannot stand up and he seems
to be guarding his right hip. On arrival, he is lying on the trolley
groaning. His heart rate is 122 beats per minute, respiratory rate
24 breaths per minute and blood pressure is 110/70.
1 2 3 4 5
Comments:
26. Nicholas is a three-year-old boy who presents with increasing
wheeze and shortness of breath. His mother indicates that he has a
history of asthma and has been in hospital before. He developed a
cold two days ago and he became increasingly wheezy yesterday.
His mother gave him Prednisolone this morning and he has had
hourly Ventolin at home. In the past two hours he has had three
doses of Ventolin; the last dose was 15 minutes ago. He has a tight
cough and a marked increase in work of breathing. Nicholas’s skin
is pale but warm; and he is distressed and restless.
1 2 3 4 5
Comments:
27. Frankie is an 18-month-old boy who presents to the ED with his
mother by ambulance. He has a barking cough and is having
difficulty breathing. His mother describes a recent cold. He woke
this morning with the cough and seemed distressed. His breathing
is fast and noisy. He does not have a stridor but does have a
barking (croup-like) cough and mild increase in work of breathing.
His skin is pink and warm and he remains settled while with his
mother. 114
1 2 3 4 5
Comments:
28. Parents present with their 13-month-old child, Oliver, who has
a history of diarrhoea and vomiting.They state that he has been
unwell for ‘about six days’. It started with vomiting, which
persisted for two to three days, but this has since stopped.
Oliver developed diarrhoea on the second day, which has
continued. He
is willing to drink and has passed two loose stools today. He shows
no shortness of breath, his skin is pink and warm and his mucous
membranes are not dry. He is grabbing at your ID badge.
1 2 3 4 5
Comments:
29. Mr Carver, an 87-year-old man, is brought to your ED in the early
hours of the morning with acute shortness of breath. He is sitting
upright on the ambulance trolley with a simple face mask in situ.
He is receiving eight litres of oxygen per minute. His heart rate is
116 beats per minute and irregular; blood pressure is 170/90;
jugular veins are visible and elevated. His skin is moist and pale.
He is unable to talk but he does nod when asked if he has chest
pain.
1 2 3 4 5
Comments:
30.A father presents at 6.30 pm with his 22-month-old son, Jackson, who has cut
his forehead after tripping and falling against the coffee table
when he was playing at home. He cried after the event and
received a large cut to his forehead.When you view Jackson he is
not distressed but he does squirm away when attempts are made
to examine his wound. He has a haematoma on the left side of his
forehead
and a full thickness laceration of one to two centimetres over his eye on the same
side.
1 2 3 4 5
115 Comments:
31. Adit is a 15-month-old boy with a two-hour onset of fever and
breathing difficulty. He presents via ambulance with an audible
stridor at rest.
1 2 3 4 5
Comments:
32. Tahlia is an 18-month-old girl who arrives at the ED with her mother
at midnight. About 24 hours prior she developed a ‘barking cough’
that became ‘much worse at night’. She is also febrile (temperature
is 38.4°C). Since becoming unwell,Tahlia has had two bottles of
water but refuses food and milk. Both mother and child appear very
anxious.
1 2 3 4 5
Comments:
33. Kerri, a 31-year-old female, presents to triage with her boyfriend.
She is complaining of a severe headache and has a history of
migraine. She said she saw her GP two days ago for ‘a sore throat’
and was prescribed penicillin, which she is currently taking. Today
she woke up with a headache and started to vomit. She is pale
with a washed-out appearance; her skin is cool and moist. Kerri’s
heart rate is 98 beats per minute, respiratory rate 18 breaths per
minute and her Glasgow coma score is 15 out of 15. She rates her
pain as ‘nine out of 10’.
1 2 3 4 5
Comments:
34. Antony, 56 years, was opening a tin of paint stripper with a
knife and some of the chemical splashed up into his right eye.
He ran water from the tap over his eye for fifteen minutes,
before his partner drove him to the ED. At triage he appears
very uncomfortable; the eye is closed and there is blistering to the
skin surrounding the right orbit.
1 2 3 4 5
Comments: 116
35. Rosemary is a 40-year-old woman who presents to triage
complaining of abdominal pain. She is 36 weeks pregnant (G5P3)
and is normally well. She tells you she has had pain ‘on and off’ for
one week but it has become more severe in the last day.
It is difficult for the patient to be precise about the location, but the
pain seems to be in the right upper quadrant. She says the pain is
‘worse after eating’ and today she has vomited twice. She says this
pain ‘does not feel like labour pains’. Her respiratory
rate is 22 breaths per minute, SpO2 is 98 per cent and heart rate
is 106 beats per minute. Her skin is pale, warm and dry, and her
Glasgow Coma Score is 15 out of 15.Temperature is measured at
37.8°C. Rosemary rates her pain as ‘seven out of ten’. She tells
you that she has had no PV loss.
1 2 3 4 5
Comments:
36. Mary-Jane is a 36-year-old woman who presents to triage via
ambulance following a fall from a ladder. She is 37 weeks
pregnant (G2P1) and is normally well. She
was hanging curtains in the nursery and standing the step second from the top
when
she overbalanced. She complains of a painful right wrist and pain in
her right hip. Her respiratory rate is 20 breaths per minute, SpO2 is
99 per cent, and her heart rate is 110 beats per minute. Her skin is
pale, warm and dry. Her Glasgow Coma Score is 15 out of 15 and
her blood pressure is 120/70. She rates her pain as ‘six out of ten’
and she reports no PV loss.
1 2 3 4 5
Comments:
37. Connie is a 74-year-old female who presents to the ED via
ambulance. Apparently she was an in-patient at your hospital five
days ago. At that time she was managed for an acute bowel
obstruction.Today the hospital-in-the-home nurse visited her and
then called an ambulance. According to the ambulance officers, she
has had increasing abdominal pain and vomiting during the night.
Her bowels have not been opened for three days.You note her to be
117
pale and distressed on the ambulance trolley. She complains that
her abdomen is ‘bloated’. Her blood pressure is 110/75, heart rate
is 112 beats per minute, respiratory rate is 26 breaths per minute,
and temperature is 37.2°C.
1 2 3 4 5
Comments:
38. Ted, a 78-year-old male, is brought to the ED via ambulance.The
patient attended the ED last night with a vague story of feeling
dizzy and unwell. He was diagnosed with a viral illness and sent
home.Throughout the night he was woken by heavy chest pains
that ‘came and went’. He took three of his Anginine, which did
not relieve the pain. He is now short of breath and his lips have a
frosted appearance. His heart rate is 92 beats per minute,
respiratory rate 24 breaths per minute and blood pressure
160/90.
1 2 3 4 5
Comments:
39. Macey is a 38-year-old female who presents to the ED with an injured right leg.
She is brought to the triage desk in a wheelchair by her father who
tells you she has multiple sclerosis.Today she was found by her
father after falling down four steps at the front of her home.
Normally she is able to walk using a walking stick, but since the fall
she has not been able to walk at all. On examination you note that
her right ankle is swollen and a right pedal pulse is palpable. She
tells you that she has ‘no pain’ at the moment and is happy to wait
to see a doctor.
1 2 3 4 5
Comments:
40.A solidly built male of about 40 years of age and smelling strongly
of alcohol starts shouting at another patient in the waiting room. He
says he wants to see a doctor, but before you can establish what is
wrong, he stands up and begins to threaten with a knife the other
patients who are waiting.
1 2 3 4 5
Comments: 118
41. Tomas is an eight-year-old boy presents to the ED with his
mother, who had been called to the school to pick him up
today.While playing at recess Tomas was involved in a fight, which
resulted in him being hit in the face with a cricket bat. His mother
says the school called her because the child was inconsolable after
the event and he didn’t want to go back to the classroom.There
was no loss of consciousness reported, but the child has a three
centimetre laceration to his left cheek.
1 2 3 4 5
Comments:
42. Harley is an 18-month-old boy who was brought to the ED via ambulance. He
was found face down in a swimming pool. His parents administered
cardiopulmonary resuscitation at the scene and called the
ambulance. On arrival the child is breathing spontaneously and
receiving 100 per cent oxygen via a bag-valve mask. His heart rate
is 140 beats per minute and his respiratory rate is 14 breaths per
minute.
The child’s eyes are closed and he is lying still on the trolley. He
is responding to painful stimuli.
1 2 3 4 5
Comments:
43. Phillip, 44 years, was bitten by an ant two days ago. The bite site,
which is located on his inner thigh, is red and itchy. There is a 15-
centimetre area of cellulitis surrounding the bite. He has a
temperature of 38.2°C.
1 2 3 4 5
Comments:
119
44. Laurie has been referred to the ED from his local doctor on a
Monday morning. He complains of increasing upper abdominal
pains, associated nausea and constipation over weekend. He had a
loose bowel action this morning. His appetite is normal, but his
pain is sharp in nature and he rates it as ‘eight out of ten’.
1 2 3 4 5
Comments:
45. Mr Smyth, 77 years of age, is brought to the ED by his daughter.
He is having difficulty passing urine and feels like his bladder ‘is
about to burst’. He tells you that he has had problems with ‘the
prostate’ before.When asked about his pain he says it is ‘about five
out of ten’.You notice that he is unable to sit still because of the
pain and he is sweating quite a bit.
1 2 3 4 5
Comments:
46. Ned, a 28-year-old jockey, attends the ED after being kicked in
the abdomen by a horse. He was assisting a colleague to guide
the animal into a float when it reared up and kicked him. Ned
was thrown some two metres and fell on the ground. He did not
lose consciousness, but was ‘winded from the kick’. At triage Ned
appears pale and distressed. He tells you that he has pain and
points to his left upper quadrant. His heart rate is 128 beats per
minute, his respiratory rate is 26 breaths per minute and his skin
is cool and moist.
1 2 3 4 5
Comments:
47. Homer, 28 years, twisted his right knee playing basketball.The
knee is very swollen and he is unable to weight-bear on it.The
injury occurred about two hours prior to his arrival in the ED and
an ice pack has been applied.
1 2 3 4 5
Comments:
120
48. Carmel, 59 years, woke this morning with pain in her left eye. She
then noticed a rash appearing above her brow and has developed
severe pain in the left side of her face and eye. She says there is ‘a
lump’ behind her ear. She has no past medical history but she did
have an episode of flu-like symptoms two days ago. She describes
the pain as ‘hot and sharp’. She rates it ‘eight out of ten’.
1 2 3 4 5
Comments:
49. Gillian presents to the ED with generalised abdominal pain. She
has been brought in by a work colleague.When questioned, she
complains of six days of constipation. She is booked in for a
colonoscopy at a private clinic tomorrow. She isn’t on medication
but she is bent over and crying in pain.
1 2 3 4 5
Comments:
50. Martin presents to the triage desk on his own. He tells you he has
pain in his left shoulder after he fell in a driveway. On examination
you note that his left shoulder is very swollen. He has very limited
range of movement, in fact he can not move the shoulder joint
itself, and he rates his pain as ‘seven out of ten’. His left radial
pulse is present, but he has some ‘numbness’ around the shoulder
area. His arm is in a sling and he smells of alcohol. He says the
accident occurred ‘a couple of hours ago’.
1 2 3 4 5
Comments:
51. Beverly is a 57-year-old female who was originally sent from her
local doctor to the outpatients department to make an
appointment to see an orthopaedic surgeon. She was referred for
an investigation of osteoarthritis in her right knee via an
arthroscope.Today, when she presents at the ED, she is in severe
pain and has difficulty weight–bearing on her right leg. She says
that the pain does settle somewhat at rest.The clerk at outpatients
1 2 3 4 5
121
Comments:
said she needed to be seen in the ED today because of her pain. An
orderly escorts her from outpatients to triage in a wheelchair.
52. Zane, 26 years of age, presents with an infected left arm. He has
a recent history of injecting drug misuse. He tells you that he has
been re-using and sharing needles. His cubical fossa is red and
cellulitic and there are several pus-filled sores on the arm. He is
afebrile. He looks around the waiting room nervously and asks you
how long it will be before he can get to see the doctor, as he ‘has
to be somewhere else in an hour’.
1 2 3 4 5
Comments:
53. Hamish is an 18-year-old male who is sent to the ED from his local
doctor with a sudden onset of right testicular pain. He has a history
of partial testicular torsion two weeks ago and states that pain is
the same as it was then. He is doubled over in pain at the triage
desk.
1 2 3 4 5
Comments:
54. Linda is a 35-year-old female with a past history of hepatitis
B.Today she presents with right side abdominal pain.The pain has
been getting worse over the past week and is currently ‘four out of
ten’. She has no vomiting; her skin is pink and warm.
1 2 3 4 5
Comments:
55. Marion, 76 years, presents to the ED from a nursing home. She
collapsed suddenly just before breakfast that morning.
Ambulance officers attended and found her semi-conscious. Her
blood glucose level was measured at 2.1 mmol and she was
given intravenous dextrose (50 mls of 50 per cent dextrose). She
is now sitting up on the ambulance trolley talking to staff.
1 2 3 4 5
Comments:
56. Cassandra, 15 years, was riding her horse in the bush some 60 122
km away from town when the animal was startled and threw her
about three metres. She was wearing
a helmet but it broke in half when her head struck a tree. Her
companions noted an initial loss of consciousness, after which she
was drowsy and vomiting, but she did not appear to have any
injuries elsewhere and she said she had no neck pain when asked.
Cassandra was transferred to your ED in the back of a utility. On
arrival
she has a Glasgow Coma Score of 8 out of 15. Her respiratory rate
is 24 breaths per minute and her heart rate is 62 beats per
minute.
1 2 3 4 5
Comments:
57. Lisa is an 18-year-old female who presents to the ED with her
friends who state that she ingested an unknown quantity of
tablets and drank a bottle of white wine about 40 minutes ago
following a fight with her boyfriend. On further questioning you
establish that the medication she took included 24 paracetamol
tablets. Lisa appears drowsy at triage, is disorientated to place
and time, and she smells strongly of alcohol. Her friends report
that in the past 10 minutes she has been ‘twitchy’.
1 2 3 4 5
Comments:
58. Iris is a 64-year-old woman who is brought to the ED by her
husband in private car. She states that she caught her leg on a
garden seat while carrying the washing in from the clothes line.
She was concerned that there was a fair amount of bleeding
occurring and she described the gash as three centimetres long.
She is not distressed.
1 2 3 4 5
Comments:
59. Silvia, 66 years, is brought to the ED by her husband. She is
complaining of a sudden onset of nausea and dizziness. She is
normally fit and well and has no relevant history. She has not
vomited and has no headache. Her blood pressure is 130/60, heart
rate 64 beats per minute and her respiratory rate is 22 breaths
per minute. She is afebrile. Her Glasgow Coma Score is 15 out of
15.
1 2 3 4 5
Comments:
123
60. Luke, a 27-year-old wants to travel to India next week. He attends
the ED for advice about the sorts of vaccinations he might need.
1 2 3 4 5
Comments:
61. Pete, aged 28 years, presents to the ED at 9 pm on a Sunday
night requesting a workers compensation certificate for a day he
had off work in the previous week. He was seen at the hospital five
days ago with a sprained wrist and had been given the certificate
for one day off work at that time. However, he states that he has
lost that certificate. He tells you that he is ‘prepared to wait’ as his
boss has told him to get a new certificate by Monday morning or he
would be ‘in big trouble’. His wrist is no longer painful and he says
he ‘feels fine’.
1 2 3 4 5
Comments:
62. Larry, 26 years, was in a fight last night. He attends the ED this
morning at 6.30 am with a five centimetre deep laceration to his
left ear. He says the injury was the result of a human bite which
occurred at about 3 am. He smells of alcohol.When asked if he was
knocked out he replies ‘no’. He has no pain elsewhere. His vital
signs are within normal limits and he is orientated to time, place
and person.
1 2 3 4 5
Comments:
63. Rudolf, 78 years, presents to triage via ambulance. He was at
church, and when he went to stand up during the service,
collapsed to the ground. He did not lose consciousness but did
become very pale and sweaty. Paramedics attended and noted
he was in heart block with a heart rate of 42 beats per minute
and blood
pressure of 80/60.They inserted an intravenous cannula and
administered atropine (600 mcg) with no effect. On arrival to the ED
he is conscious and states that he has no chest pain.
1 2 3 4 5
Comments: 124
64. Betty is a woman in her 20s. She presents to the triage desk with
her friend, who states that Betty has taken 25 Endep tablets. As
you begin talking to her friend, Betty collapses to the floor and
commences fitting.You summon help and staff arrive to lift her onto
a trolley and take her into the ED.
1 2 3 4 5
Comments:
65. Barry, a 43-year-old man, was using an angle-grinder today and
now has a foreign body in his left eye.The eye is red and painful.
He states that the pain is ‘seven out of ten’.
1 2 3 4 5
Comments:
66. Mario, a 67-year-old man, was putting some pesticide on his
vegetable patch and he accidentally spilt it on his clothing. He
had a shower at home immediately after
the accident but has come to the ED some two hours later because
he is nauseous, vomiting and has developed excessive sweating. His
heart rate is 122 beats per minute and his respiratory rate 28
breaths per minute. He says he is not sure of the name of the
chemical he was exposed to as he has had it in his shed for ‘many
years’.
1 2 3 4 5
Comments:
67. Mr F is a 66-year-old man who was brought to the triage desk by
his daughter. He states that he is confused and thinks that people
are talking about him. He tells you that he has a history of ‘heart
failure, high blood pressure, renal failure, urinary tract infection
and depression’. His skin is warm and moist, his respiratory rate is
20 breaths per minute, and his Glasgow Coma Score is 15 out of
15.
1 2 3 4 5
Comments:
125
68. Hugh is a 54-year-old male who was seen in the ED with a
fractured right radius and ulna four days prior. He presents again
because he says the cast is too loose and needs to be replaced. He
has no pain.
1 2 3 4 5
Comments:
69. Sue, a 36-year-old female, presents with a two-day history of
feeling generally unwell. She has an ache in her lower abdomen
and describes having to go to the toilet more frequently than
normal. On further questioning she states that she has had urinary
frequency for 12 hours, and rates her pain as ‘four out of ten’. She
has a heart rate of 98 beats per minute and a temperature of
37.8°C. She appears to be quite pale.
1 2 3 4 5
Comments:
70. Joanne is a 34-year-old female who walks to the triage desk at
10.50 am.When you ask her what is wrong she says ‘I can’t go to
the toilet and my backside is painful’. When questioned further
she says that she has not passed urine today but ‘did last night
and it was not painful’. She rates her current pain as ‘four out of
10’.
1 2 3 4 5
Comments:
71. Mrs W is assisted to the triage desk by her daughter around
midday. Mrs W doesn’t speak very good English so her daughter
tells you her history. Last night Mrs W had an episode of
palpitations and complained of nausea and feeling lethargic.Today
‘the palpitations are back’. She has a history of coronary artery
bypass grafts.When asked if she has chest pain, Mrs W says she is
‘very sick’. Her heart rate is 108 beats per minute and her skin is
cool and moist to touch.
1 2 3 4 5
Comments: 126
72. Maree is a 32-year-old woman who presents via ambulance
complaining of ‘palpitations’. She is 30 weeks pregnant (G3P1) and
is normally well. She was doing the vacuuming when her
palpitations started. She complains of mild chest pain that is dull in
nature and a mild shortness of breath.’ Her respiratory rate is 24
breaths per minute; SpO2 is 98 per cent; and heart rate is162 beats
per minute. Her skin is pale, cool and dry; blood pressure is 90/R;
and Glasgow Coma Score is 15 out of 15. Her temperature is
36.3°C.
1 2 3 4 5
Comments:
73. Kerry is a 36-year-old woman who presents to the ED with her
husband via ambulance with a sudden onset of a headache. She
tells you that she is 31 weeks pregnant (G3P1) and has been
‘keeping well’. Her husband tells you that Kerry was making
lunch when she suddenly complained of a severe occipital
headache.
Her respiratory rate is 20 breaths per minute, SpO2 is 98 per cent, and heart rate
is 124 beats per minute. Her skin is pale, cool and dry.The ambulance officers
report
that Kerry’s blood pressure is 160/100 and she has a Glasgow
Coma Score of 14 out of 15 (eyes open to voice). Her
temperature is 36.3°C.When asked to score her level of pain,
she tells you it is ‘nine out of 10’.
1 2 3 4 5
Comments:
74. Tricia, an 18-year-old female, is brought into the ED by a friend.
Her friend states that she has had vaginal bleeding since her ‘Depo
injection 15 days ago’. Her friend states that Tricia is suicidal and
wants to find ‘peace’. Her friend also tells you that Tricia took a
1 2 3 4 5
127
Comments:
large quantity of herbal sedative last night and now feels ‘weak and
tired’.
75. Josie, 39, walks to the triage desk and complains of pain in her
legs, stating;‘My feet and legs are swollen and sore’ She has a
history of intravenous drug use and heavy alcohol intake and she
has hepatitis C. Currently Josie is not on any medication and is
alert and orientated.
1 2 3 4 5
Comments:
76. Jake is 28 years old. He attends the ED with his partner at 5.30
pm. He has abdominal pain radiating to his right loin, urinary
frequency and dysuria. He saw his GP yesterday for the pain and
was told he ‘might have kidney stones’.The pain is worse now than
yesterday (‘seven out of 10’) and he has noticed some blood in his
urine the last time he voided.
1 2 3 4 5
Comments:
77. Isaac is an 85-year-old male who presents to triage with his son.
He has left loin pain and has recently undergone a lithotripsy for
renal calculi.Today he has had pain for one-and-a-half hours which
‘comes and goes’; the pain is now ‘eight out of ten’.
1 2 3 4 5
Comments:
78. Jess, 14 years, is brought to the ED by her mother. She is
complaining of severe period pains and is doubled over in a
wheelchair crying. Her mother tells you that Jess has not been
able to go to school for the past week because of her menstrual
problems and wants a referral to a specialist to ‘sort out the
problem’. When
you talk to Jess you establish that the blood loss is moderate and
the pain is in her abdomen, thighs and back. She seems to calm
down after you speak to her and appears more comfortable when
you wrap a blanket around her.
1 2 3 4 5
Comments:
128
79. A 5-year-old boy is rushed into your ED by his parents on a hot
summer day. He has been holidaying with his family in Far North
Queensland and was wading in the sea. He has a raised red welt on
his right leg and is crying in severe pain, He has a heart rate of 128
beats per minute and a blood pressure of 130/70.
1 2 3 4 5
Comments:
80. Reese, 31 years, suffers from migraines.Today she has come to
the ED with her sister. She has had an eight-hour history of a global
headache, vomiting and visual disturbance. She has taken her
usual medication (Imigran), but says it is ‘not working’. Her heart
rate is 96 beats per minute, respiratory rate 28 breaths per minute.
She is afebrile and rates her pain seven out of ten.
1 2 3 4 5
Comments:
81. India is a nine-year-old girl who arrives to the ED via a taxi
accompanied by her mother. She fell while playing netball, injuring
her right foot. She is transferred to the triage desk in a wheelchair
as it is painful for her to weight bear.
1 2 3 4 5
Comments:
82. Terry is a 53-year-old male who presents to the ED asking for a
review of his blood pressure medication. He describes having had a
‘headache’ during the past week. It is two years since he saw a
doctor about his medication. His Glasgow Coma Score is 15 out of
15 and his heart rate is 70 beats per minute; he has no nausea or
vomiting and is currently pain free.
1 2 3 4 5
Comments:
129 83. A mother presents with her six-month-old baby who she says
won’t wake up. The child is breathing, but is floppy, can not be
roused and has pin-point pupils.
1 2 3 4 5
Comments:
84. Paddy is a 32-year-old male who presents to triage stating that he
has vomited blood twice in the last six hours. He states that he has
had dark bowel motions for the last three days and he normally
drinks ‘12 stubbies of beer per day’. Paddy’s skin is pale, warm and
dry. His heart rate is 108 and his respiratory rate is 20 breaths per
minute. He doesn’t have any pain but does complain of nausea.
1 2 3 4 5
Comments:
85. Amber is a 22-year-old woman who presents to the ED at 11 pm
complaining of a 24-hour history of a sore throat and is feeling
generally unwell. She had been attending a party nearby and
decided to call into the hospital to get some antibiotics. She has no
other symptoms, looks well and is afebrile.
1 2 3 4 5
Comments:
86. You are called to assist a young woman getting her boyfriend
out of the car that is pulled up in the ambulance bay. She tells
you that Matt ‘shot up’ 30 minutes ago.
On examination Matt appears to have vomited and is centrally
cyanosed. He has irregular grunting respirations of 6 breaths per
minute and his heart rate is 42 beats per minute.
1 2 3 4 5
Comments:
130
87. Elliot is 27 years old. He injured his back yesterday lifting a
heavy box at work. He had been managing the pain at home,
however today it is ‘much worse’. He was unable to get an
appointment with his local doctor so he has come to the ED. He
rates his pain ‘five out of ten’, and has taken two Panadeine Forte
and two Nurofen tablets in the past hour.
1 2 3 4 5
Comments:
88. Ambulance officers arrive without prior notice with a female aged
26. She was a front-seat passenger in a single motor vehicle crash
that involved multiple rollovers.
The ambulance officers state that the patient was walking around
intoxicated at the scene and was abusive, complaining of
abdominal pain and reluctant to come to hospital. On examination
the patient is centrally cyanosed and not breathing.
1 2 3 4 5
Comments:
89. Ron, the 50-year-old coach of a visiting interstate football team,
presents to triage at 7 pm on Saturday night. His anti-
hypertensive medications have run out and
his GP had warned him that it would be dangerous for him to stop
his medications. The man says that he realises that it is ‘not
completely appropriate’ for him to attend the ED for a prescription,
but says he doesn’t know any GPs in the city and is quite prepared
to wait for a prescription. His Glasgow Coma Score is 15 out of 15
and his skin is pink, warm and dry. He has no headache or pain
elsewhere.
1 2 3 4 5
Comments:
90. Noel, 29 years, is driven to the ED by friends following a fight at
his cousin’s party. You are called to retrieve Noel from the
ambulance bay.While getting Noel out of the car, you learn that he
was stabbed in the left side of his chest with a carving knife and
see a two centimetre laceration below his left nipple. His skin is
cool, pale and moist. He has a weak carotid pulse and a Glasgow
Coma Score of 9 out of 15.
1 2 3 4 5
131 Comments:
91. Brett is 27. He presents to triage via a private car following a
fall from scaffolding at a construction site approximately 20
minutes prior to presentation. Brett fell more than 10 feet onto a
concrete slab. He was observed by his work mates to
be unresponsive for ‘about five minutes’ and then he regained
consciousness, but he has been drowsy. He has vomited four times
and has a large boggy haematoma on his occiput. Brett is
complaining of a generalised headache. His Glasgow Coma Score is
13 out of 15, heart rate is 74 beats per minute, and respiratory
rate is 14 breaths per minute.
1 2 3 4 5
Comments:
92. An obviously pregnant woman presents to triage stating that she is in labour
and that she thinks there is something hanging down between her
legs. On cursory examination you see under her dress what
appears to be an umbilical cord.
1 2 3 4 5
Comments:
APPENDIX A: ACEM POLICY DOCUMENT
Guidelines for Implementation of
the Australasian Triage Scale
General Principles
Function of triage
Triage is an essential function in Emergency Departments (EDs),
where many patients may present simultaneously. Triage aims to
ensure that patients are treated in the order of their clinical urgency
and that their treatment is appropriately timely. It also allows for
allocation of the patient to the most appropriate assessment and
treatment area, and contributes information that helps to describe the
departmental casemix. Urgency refers to the need for time-critical
intervention – it is not synonymous with severity.
Patients triaged to lower acuity categories may be safe to wait longer
for assessment and treatment but may still require hospital
admission.
The Triage Assessment
132
The features used to assess urgency are generally a combination of
the presenting problem and general appearance of the patient,
possibly combined with physiological observations. The triage
assessment should generally take no more than two to
five minutes, obtaining sufficient information to determine the
urgency and identify any immediate care needs. This does not
preclude the initiation of investigations or referrals at this point.
There must be a balance between speed and thoroughness. The
triage assessment is not necessarily intended to make a diagnosis,
although this may sometimes be possible. Vital signs should only be
measured at triage if required to estimate urgency, or if time
permits. Any patient identified as ATS Category 1 or 2 should be
taken immediately into the appropriate assessment and treatment
area.
A more complete nursing assessment should be done by the
treatment nurse receiving the patient.
In Australia, triage is carried out by emergency nurses. As triage is so
important to both the smooth running of an ED and the outcome of the
patients, it should be carried out by staff who are both specifically
trained and experienced.
Safety at triage
Triage is the first point of public contact with the ED. Patients with the
whole spectrum of acute illness, injury, mental health problems and
challenging behaviour may present there. Pain, anxiety and/or
intoxication in patients or their relatives may provoke or magnify
aggressive behaviour. These factors may create a risk of harm for the
Triage Nurse and other reception staff. It is essential that all EDs plan
for this potential risk
by providing a safe but non-threatening physical environment,
providing minimisation- of-aggression training to front-line staff, and
having safe protocols and procedures for dealing with challenging
behaviour. Where the safety of staff and/or other patients is under
threat, staff and patient safety should take priority and an
appropriate security response should take place prior to clinical
assessment and treatment.
Time-to-treatment
The time-to-treatment described for each ATS category refers to the
maximum time a patient in that category should wait for
assessment and treatment. In the more urgent categories,
assessment and treatment should occur simultaneously. Ideally,
patients should be seen well within the recommended maximum
times. Implicit in the descriptors of categories 1 to 4 is the
assumption that the clinical outcome may be affected by delays to
133 assessment and treatment beyond the recommended times.
Further research is still required to describe the precise relationship
between the time to treatment and the clinical outcome. The
maximum waiting time for ATS Category 5 represents a standard for
service provision.
The recommended performance thresholds represent realistic practice
constraints in the clinical environment. However, there is no implied
justification for prolonged delays for patients falling outside the
required performance standards – all attempts should be made to
minimise delays.
Document standards
The documentation of the triage assessment should include at
least the following essential details:
• Date and time of assessment
• Name of Triage Officer
• Chief presenting problem(s)
• Limited, relevant history
• Relevant assessment findings
• Initial triage category allocated
• Re-triage category with time and reason
• Assessment and treatment area allocated
• Any diagnostic, first aid or treatment measures initiated.
Re-triage
If a patient’s condition changes while they are waiting for treatment,
or if additional relevant information becomes available that impacts
on the patient’s urgency, the patient should be re-triaged. Both the
initial triage and any subsequent categorisations should be recorded,
and the reason for the re-triage documented.
Clinical descriptors
The listed clinical descriptors for each category are based on
available research data where possible, as well as consensus.
However, the list is not intended to be exhaustive or absolute and
must be regarded as indicative. Absolute physiological measurements
should not be taken as the sole criterion for allocation of an ATS
category. Senior clinicians should exercise their judgement and,
where there is doubt, err on the side
of caution.
Most urgent features determine category
The most urgent clinical feature identified determines the ATS
category. Once a high-risk feature is identified, a response
commensurate with the urgency of that feature should be
initiated.
134
Practicality and reproducibility
The primary and most important role of triage is clinical. Therefore
application of the ATS must occur in such a way that ensures patient
safety and maximises flow through the emergency department. While
it is desirable to attempt to maximise inter-rater reliability for reasons
of inter-departmental comparisons and for casemix purposes,
it must be recognised that no clinical coding system achieves perfect
reproducibility. Acceptable levels of inter-rater agreement have been
defined which allow for a realistic balance between clinical practicality
and coding precision.
Specific conventions
In order to maximise reproducibility of ATS allocation between
departments, the following conventions have been defined:
• Paediatrics
The same standards for triage categorisation should apply to all ED
settings where children are seen – whether purely Paediatric or
mixed departments. All five triage categories should be used in all
settings. This does not preclude children being seen well within the
recommended waiting time for the ATS category if departmental
policy and operational conditions provide for this. However, for the
sake of consistency and comparability, children should still be
triaged according to objective clinical urgency. Individual
departmental policies such as ‘fast-tracking’ of specific patient
populations should be separated from the objective allocation of a
triage category.
• Trauma
All victims of trauma should be allocated a triage category
according to their objective clinical urgency. As with other clinical
situations, this will include consideration of high-risk history as well
as brief physical assessment (general appearance +/- physiological
observations). Although individual departments may have policies
that provide for immediate team responses to patients meeting
certain criteria, these patients should still be allocated an objective
triage category according to their clinical presentation. Again,
departmental ‘fast-tracking’ policies or systems should occur
separately to the objective allocation of a triage category.
• Behavioural disturbance
Patients presenting with mental health or behavioural problems
should be triaged according to their clinical and situational
urgency, as with other ED patients. Where physical and behavioural
problems co-exist, the highest appropriate triage category should
be applied based on the combined presentation.
While some acutely-disturbed patients may require an immediate
clinical response (perhaps combined with a security response) to
ensure their safety, it is recognised that some individuals entering
an emergency department and posing an immediate threat to staff
135 (eg brandishing a dangerous weapon) should not receive a clinical
response until the safety of staff can be ensured. In this situation,
staff should act so as to protect themselves and other ED patients,
and obtain immediate intervention from security staff and/or the
police service. Once the situation is stabilised, a clinical response
can take place as (and if) required, and triage should then reflect
clinical and situational urgency.
ATS Category Response Description of Category Clinical Descriptors (indicative only)
AP
Category 1 Immediate Immediately Life- Cardiac arrest PE
simultaneous
assessment and
Threatening Conditions
that are threats to life (or
Respiratory
arrest ND
treatment imminent risk of
deterioration) and require
Immediate risk to airway – impending
arrest Respiratory rate <10/min
IX
D
immediate aggressive
intervention.
Extreme respiratory distress B:
BP <80 (adult) or severely shocked child/infant
ep
ar
Unresponsive or responds to pain only (GCS AU
t
<9) Ongoing/prolonged seizure
IV overdose and unresponsive or
ST
m
en
hypoventilation
Severe behavioural disorder with immediate
RA
t
of threat of dangerous violence LA
H
ea
Category 2 Assessment and
treatment within 10
Imminently Life-
Threatening The patient’s
Airway risk – severe stridor or drooling
with distress Severe respiratory distress SIA
lt
h
minutes (assessment
and treatment often
condition is serious enough
or deteriorating so rapidly
Circulatory compromise
– Clammy or mottled skin, poor perfusion
N
an
d
simultaneous) that there is the potential of
threat to life, or organ
– HR<50 or >150 (adult)
– Hypotension with haemodynamic effects
TRI
A
ge
system failure, if not treated
within ten minutes of arrival
– Severe blood loss AG
Chest pain of likely cardiac
in
g
OR
Important Time-Critical
nature Very severe pain - any E
cause
–
E
Treatment BSL <2 mmol/l SC
The potential for time-critical
m
er
treatment (e.g. Drowsy, decreased responsiveness any cause
(GCS <13) Acute hemiparesis/dysphasia
AL
thrombolysis, antidote) to
ge
nc
make a significant effect on Fever with signs of lethargy (any age) E:
clinical outcome depends Acid or alkali splash to eye – requiring DES
on treatment commencing irrigation
within a few minutes of the CRIP
patient’s arrival in the ED Major multi trauma (requiring rapid organised
team response) Severe localised trauma – major TOR
OR
Very Severe Pain
fracture, amputation S
Humane practice mandates High-risk history: FOR
the relief of very severe pain – Significant sedative or other toxic ingestion
or distress within 10 minutes – Significant/dangerous envenomation CAT
– Severe pain suggesting PE, AAA or ectopic
pregnancy
1 Behavioural/Psychiatric:
3
– violent or aggressive
– immediate threat to self or others
– requires or has required restraint
– severe agitation or aggression
1
3
ATS Category Response Description of Category Clinical Descriptors (indicative
only)
Category 3 Assessment and Potentially life- Severe hypertension
treatment start within 30 threatening The patient’s Moderately severe blood loss – any
mins condition may progress to
cause Moderate shortness of
D life or limb threatening, or
may lead to significant breath
ep
ar morbidity, if assessment and Sp02 90-95%
t treatment are not BSL >16 mmol/l
m
commenced within thirty
minutes of arrival Seizure (now
en
t OR alert)
of Situational urgency Any fever if immuno-supressed eg.
H There is potential for oncology patient, steroid Rx
ea adverse outcome if time- Persistent
lt critical treatment is not vomiting
h commenced within thirty Dehydration
an minutes
OR Head injury with short LOC – now alert
d
A Humane practice Moderately severe pain – any
ge mandates the relief of cause – requiring analgesia
in severe discomfort or Chest pain likely non-cardiac and mod
g distress within thirty severity Abdominal pain without high risk
minutes features – mod severe or patient age
–
>65 years
E
m Moderate limb injury – deformity,
er severe laceration, crush
ge Limb – altered sensation, acutely
nc absent pulse Trauma – high-risk
history with no other high-risk
features
Stable
neonate
Child at risk
Behavioural/Psychiatric:
– Very distressed, risk of self-harm
– Acutely psychotic or thought
disordered
– Situational crisis, deliberate self harm
– Agitated/withdrawn/potentially
aggressive
ATS Category Response Description of Category Clinical Descriptors (indicative
only)
Category 4 Assessment and Potentially Serious Mild haemorrhage
treatment start within The patient’s condition may Foreign body aspiration, no respiratory
60 mins deteriorate, or adverse distress Chest injury without rib pain or
outcome may result, if respiratory distress
assessment and treatment is
not commenced within one Difficulty swallowing, no respiratory
D
ep hour of arrival in ED. distress Minor head injury, no loss of
ar Symptoms moderate or consciousness Moderate pain, some risk
t prolonged.
features
m OR
Situational Urgency Vomiting or diarrhoea without
en
t There is potential for dehydration
of adverse outcome if time- Eye inflammation or foreign body –
H critical treatment is not normal vision
ea commenced within the
Minor limb trauma – sprained ankle,
lt hour. possible fracture, uncomplicated
h OR laceration requiring investigation or
an Significant complexity intervention – Normal vital signs,
d or Severity low/moderate pain
A Likely to require complex Tight cast, no neurovascular
ge work-up and consultation impairment Swollen “hot” joint
in and/or inpatient
management. Non-specific
g
– OR abdominal pain
E Humane practice Behavioural/Psychiatri
m mandates the relief of
c:
er discomfort or distress
– Semi-urgent mental health problem
ge within one hour.
– Under observation and/or no
nc immediate risk to self or others
Category 5 Assessment and Less Urgent Minimal pain with no high risk
treatment start within The patient’s condition is features Low-risk history and
120 minutes chronic or minor enough
that symptoms or clinical now asymptomatic Minor
outcome will not be symptoms of existing stable
significantly affected if illness Minor symptoms of low-
assessment and treatment
risk conditions
are delayed up to two
hours from arrival. Minor wounds - small abrasions, minor
lacerations (not requiring sutures)
Clinico-administrative Scheduled revisit eg wound review,
1
problems, such as: results complex dressings
3
review, medical certificates, Immunisation only
prescriptions only.
Behavioural/Psychiatric:
– Known patient with chronic
symptoms
– Social crisis, clinically well patient
APPENDIX C: CENA TRIAGE NURSE
POSITION STATEMENT
Position
Statement
Triage Nurse
Introduction
The purpose of this position statement is to define the role of
Triage Nurse and the minimum Triage Nurse practice standards in
accordance with the best available evidence, to promote national
triage consistency in the application of the Australasian Triage
Scale (ATS). It is acknowledged that although triage may be
performed in a number of settings other than an Emergency
Department, CENA
produces this position statement in the setting of the Triage Nurse
working within an Emergency Department.
Triage is the first instance of clinical contact for all people presenting
to the Emergency Department. It is the point at which emergency
139 care begins.Triage is a brief clinical assessment that determines the
urgency of treatment and the time and sequence in which patients
should be seen in the Emergency Department. Although primarily a
clinical tool for ensuring that patients are seen in a timely manner,
commensurate with their clinical urgency, the ATS is also a useful
casemix measure.The scale directly relates triage code with a range of
outcome measures (inpatient length of stay, ICU admission, mortality
rate) and resource consumption (staff time, cost). It provides an
opportunity for analysis of a number of performance parameters in
the Emergency Department, such as casemix, operational efficiency,
utilisation review, outcome effectiveness and cost (ACEM, 2006).
Position
Triage is an autonomous nursing role and essential to the efficient
delivery of emergency care. Clinical decisions made by Triage Nurses
require complex cognitive processes. The Triage Nurse must
demonstrate critical thinking skills and abilities in environments
where data available to inform such decisions is limited, incomplete or
ambiguous. The ability to formulate judgments and make decisions is
critical, and the quality and accuracy of triage judgments and
decision-making are central to appropriate clinical care. In some
models of care, triage may include a medical officer in a triage team.
CENA endorses the concept that Triage must be attended to by no
Department of Health and Ageing – Emergency Triage
less than a triage qualified Registered Nurse.
Department of Health and Ageing – Emergency Triage
The Role of the Triage Nurse is to:
a. Undertake patient assessment and allocate the ATS category
b. Initiate appropriate nursing interventions and organisational
guidelines (e.g. first aid and emergency interventions) to improve
patient outcomes and secure the safety of patients and staff of the
department
c. Ensure continuous reassessment and management of patients
who remain in the waiting room commensurate with their
condition and time frames determined by the ATS category
d. Provide patient and public education where appropriate to facilitate
a. health promotion and education
b. injury prevention
c. community resourcing and information
e. Act as the liaison for members of the public and other health care professionals
The ability to undertake effective and efficient triage is dependent
on extensive knowledge and experience with a wide range of illness
and injury patterns. As such, it is the position of CENA that triage
and the role of Triage Nurse be undertaken by specifically trained
and experienced Registered nurses.1
The decisions of Triage Nurses should, as far as possible, be
evidence-based and demonstrate best-practice standards. All triage
decisions are to be based on clinical urgency of the patients’ 140
presenting problems, and not determined by factors such as
departmental workload and funding.
CENA endorses a set of minimum standards for the Triage Nurse and
triage practice:
Standard 1: Education,Training and Professional Development
CENA endorses the Emergency Triage Education Kit as the minimum
standard for all Triage Nurses and nurses required to undertake triage
roles within the Emergency Department. The following theoretical and
practice elements are core components of Triage Nurse training:
a. history, science and practice of triage
b. the Australian health care system
c. the role of the Triage Nurse
d. the Australasian Triage Scale (ATS)
e. effective communication skills
f. legislative requirements and considerations
g. epidemiology and population health
h. assessment and triage decision-making by presentation type
i. primary and secondary surveys
ii. trauma
iii. medical and surgical emergencies
iv. paediatric emergencies
v. obstetric and gynaecological emergencies
vi. mental health emergencies
vii. rural and isolated triage practice
viii.environmental emergencies
i. quality and safety in health care
Standard 2: Clinical Practice
The Triage Nurse:
a. is a qualified and experienced Registered Nurse* who demonstrates
and maintains clinical expertise in emergency nursing prior to
undertaking the role of Triage Nurse
b. completes triage education based on the Emergency Triage
Education Kit prior to commencing the triage role
c. participates in research processes of audit and evaluation of triage practice
d. participates in annual education, training and professional
development in triage and related emergency nursing and
emergency care activities
Standard 3: Equipment and Environment
The triage environment must provide safety for the public, the
Triage Nurse, staff and patients of the Emergency Department and
the hospital. The environment must:
a. be immediately accessible and well sign posted
b. have access to an area for patient examination and primary treatment
c. be designed to maximise the safety of the Triage Nurse, staff and
patients (e.g. duress alarms, access to security personnel)
d. be equipped with emergency equipment and the Triage Nurse
deemed competent in its use
141 e. enable care to be provided with due regard to standard and
additional precautions for infection control and prevention
f. enable and facilitate patient privacy
Rationale
The role of the Triage Nurse is central to the effective and efficient
operation of the Emergency Department. Emergency Departments
are routinely unpredictable settings. The finite resources of the
Emergency Department emphasise the need for timely and accurate
triage decisions that ultimately underpin optimal health service
delivery.
References
Australasian College for Emergency Medicine (ACEM). (2006). P06
– Policy on the Australasian Triage Scale. Melbourne: Australasian
College for Emergency Medicine.
Date developed: January
2007 Endorsed: March
2007
Review: March 2009
*
Division 1 in Victoria
APPENDIX D: METHODOLOGY
Aims
The ETEK aims to provide a nationally consistent approach to the
educational preparation of emergency nurses for the triage role, and
promote the consistent application of the Australasian Triage Scale
(ATS).This project involves the validation of the educational tools
provided in the ETEK and was conducted to achieve the following
aims:
1. To develop a large set of paper-based triage scenarios for inclusion in the ETEK
2. To assess the above scenarios for content validity and determine inter-rater
reliability
3. To achieve a weighted kappa of at least 0.6 for the scenarios
set using an expert panel of emergency nurse raters
Method
The method used for this study was a postal survey comprising a
demographic questionnaire and a series of paper-based triage
scenarios. An expert panel of Triage Nurses rated each of the
scenarios into one of five ATS categories.This process was
undertaken to identify a scenario set suitable for teaching consistent 142
application of the ATS using established guidelines.2
Design
A descriptive correlational design was used to measure inter-rater
reliability for 237 triage scenarios among multiple expert raters.
Setting
Participants worked in publicly funded Emergency Departments
located in each of five Australian states and two Australian
territories. Data was collected for this study from 2 April to 14 May
2007.
Participants
Participants in this study comprised a convenience sample of 50
experienced Triage Nurses working in public Emergency
Departments. Inclusion and exclusion criteria for participants are
shown in Table A1. In order to obtain a sample with representation
from each state a stratified approach was used to select
participants.Table A2 shows the sampling frame used for this study.
Instrument
The consultant developed a questionnaire containing a total of 237
triage scenarios that related to the content covered in the following
chapters of the ETEK:
Chapter 4: Triage
Basics Chapter 5:
Mental
Health Chapter 7:
Pain
Chapter 8: Paediatrics
Chapter 9: Pregnancy
The fictitious scenarios were constructed in consultation with a
group of specialist Triage Nurses (paediatric, mental health,
obstetric).The types of scenarios included in the questionnaire
took into account common chief complaints to Australian EDs
with some additional scenarios to cover presentations more likely to
occur in regional and rural areas (e.g. snake, spider and marine
envenomation). Each of the 237 triage scenarios was written in a
narrative form, to resemble a triage note.The scenarios were one
paragraph long and provided a basic description of the patient
including; age gender, general appearance and observations.The
information provided in the scenarios was based on the original work
of Whitby et al131, who developed it from 11,169 actual episodes of
triage, a description of the features of emergency presentations
used by nurses when allocating a triage code.The narrative
approach to the scenario presentation was adopted to take into
143
account the primary purpose of the ETEK
as a teaching resource. Using this format, participants are
required to identify from a scenario, which is presented as text,
the clinically important components of the presentation and
allocate a triage code.
Procedure
Members of the National Education Framework for Emergency Triage
Working Party located individuals from their own professional
networks who met the section criteria according to the sampling
frame.Working Party members contacted these potential participants
by telephone to explain the study. This was done using the Plain
Language Statement (PLS).
Those participants who agreed to take part were sent the PLS, a
consent form, the questionnaire, ATS Guidelines1,50 and a reply paid
envelope.These were provided to those performing the recruitment
as a kit.Working Party members addressed the envelopes
containing the consent form, the questionnaire, the ATS Guidelines
and the PLS.They recorded the questionnaire number against the
participant name and informed Marie Gerdtz of the codes used.
Participants returned completed questionnaires directly to Marie
Gerdtz via the reply paid envelope.
Ethical approval
Ethical approval to conduct this low risk project was sought via
expedited review. Ethical and technical review of the research
protocol was undertaken via the School of Nursing Human Research
Ethics Advisory Group and approved by the University of Melbourne
Human Research and Ethics Committee.
Approval granted on 27 March 2007.
The research was performed in accordance with NHMRC Guidelines. 144
Participants were given a Plain Language Statement (PLS) and provided written
consent.
Analysis
Raw data was entered into SPSS (version 10.0). Descriptive analysis
was performed including calculation of frequencies, mean and
standard deviation for demographic variables.The data was also
explored descriptively by determining concurrence; that is, the
percentage of responses for each case scenario in the modal
category and spread. Raw percentage agreement was calculated for
the 237 scenarios for the modal response category.
144
Scenarios were analysed as an entire set and were also categorised
according to the chapters contained in the ETEK (Chapters 4, 5, 7, 8
and 9).
To explore the data, all scenarios in which the modal response
category was greater than 60 and 70 per cent respectively were
included in a model to calculate
chance-corrected agreement (unweighted kappa).This approach
was taken to identify the maximum number of scenarios
appropriate for testing according to the predetermined criteria
(kappa >0.6).
The formula for calculating kappa is by Fleiss et al.145:
Kj = P j – pj
1 – pj
= N nij2 – Nnpj[1+(n –1)pj)]
i=1
Nn(n –1)pjqj
Notation: N represents the total number of subjects, n is the number
of ratings per subject and k is the number of categories into which
assignments are made. Let the subscript I where i=1,… N, represent
the subjects and the subscript j, where j=1,…k, represent the
categories of the scale. Define nij the number of raters who assigned
the ith subject to the jth category.The quantity pj is the proportion of
all assignments that were to the jth category.8
The formula for calculating kappa variance was also from Fleiss et al.145:
Var(k)= 2
x( pjqj)2 – pjqj(qj – pj)
Nn(n–1)(
pjqj)2
Analysis of kappa statistics was done by programming the above
formula into a Microsoft Office Excel (2003). Frequencies were
entered by scenario and triage code.
Statistical advice and checking of analysis was provided by Marnie
Collins (Statistical Consulting Centre Department of Mathematics and
Statistics,The University of Melbourne).
Results
A total of 42 (84%) of questionnaires were returned, of which eight
145 individual items on the questionnaire were incomplete. A total of
9946 occasions of triage were
available for the analysis. In Table A3 the demographic characteristics
of the participants who completed the questionnaire are provided. All
participants were experienced in emergency nursing and in
performing the triage role, more than half (n=27; 64.3%)
held specific qualification in emergency nursing at a Graduate
Certificate or Diploma level.The majority designated appointment
level as Registered Nurse/Clinical Nurse Specialist (n=27; 64.2%).The
remainder reported working in combined management, teaching and
clinical roles.
Good distribution of participants was achieved nationwide with the
exception of South Australia. Recruitment of suitable participants in
South Australia was adversely influenced because the person who
was responsible became unwell during the project and was not able
to distribute the questionnaires as, originally planned. Raw
percentage agreement for each of the 237 scenarios was found to
vary widely with 92 (38%) showing raw percentage agreement at 70
per cent or more, and 152 (64%) showing raw percentage agreement
of 60 per cent or more. Certain types of scenarios were noted to have
poorer levels of agreement. In particular scenarios classified as
‘mental health’ and ‘pregnancy’ show lower levels of agreement than
other scenario types.
Excellent rates of interobserver agreement were achieved using
unweighted kappa in the scenario set that included all scenarios with
raw percentage agreement >70 per cent.
For this scenario set an overall unweighted kappa of 0.63 was
achieved, with higher levels of agreement noted in ATS categories 1
and 5. 95 per cent confidence intervals for this scenario set indicate
reasonable precision (0.629–0.636).Table A3 shows the distribution of
responses for each of the scenarios with a modal response of >70 per
cent. Good rates of interobserver agreement for all five ATS categories
were also achieved with raw percentage agreement >60 per cent. For
this scenario set an overall unweighted overall kappa was 0.5.Table A6
shows the distribution of responses for each of the scenarios with a
modal response of >60 per cent. Ninety Five per cent confidence
intervals for this scenario set also indicate reasonable precision
(0.508, 0.514).
Recommendations
The scenarios that were tested in this project fell naturally into three groups.
The first set of scenarios show excellent level of agreement and can
be used for testing purposes if required (n=92). Evidence of
agreement was determined by raw percentage agreement >70 per
cent and chance corrected agreement of >kappa 0.6. For this scenario
set an overall unweighted kappa of 0.63 was achieved, with higher
levels of agreement noted in categories 1 and 5.These 92 scenarios
are suitable for the purpose of testing the application of the ATS 146
guidelines.
The second set of scenarios show moderate levels of agreement and
could be used for teaching, but not testing, purposes within the ETEK
(n=61). For this scenario set an overall unweighted overall kappa
was 0.5.
The third group of scenarios shows lower levels of agreement (n=84).
It is not recommended that these scenarios be used for testing or
teaching in the context of the ETEK. However, future work could be
performed to assist in identifying aspects of the scenarios that
influence agreement levels.
We have identified from this work that, despite being provided with
guidelines, interobserver agreement among expert Triage Nurses for
scenario sets involving mental health, pregnancy and paediatrics
remain relatively poor when compared to those involving other types
of general ED presentations. Future work may be required to refine
current ATS guidelines to these specific patient presentations.
Table A1: Inclusion and exclusion criteria
Inclusion criteria Exclusion criteria
Registered Nurse NBV People with whom working party
Division 1 (or equivalent) members you may have a
currently practicing dependent relationship (i.e. junior
Five years of experience as staff member with line
an emergency nurse management responsibility).
Practicing triage at least once per
week in an A1, B1 or B2 hospital.
Table A2: Sampling frame
State/Territory n Hospital classification (n)
ACT 1 A1 (1)
New South Wales 1 A1(5), B1(5), B2(5)
5
Northern Territory 2 A1 (2)
Queensland 1 A1 (5), B1(3), B2 (2)
0
147 South Australia 5 A1 (3), B1 (2)
Tasmania 2 A1 (2)
Victoria 1 A1 (5), B1(2), B3 (3)
0
Key: A1 Principle referral hospital, B1 Large major city hospital, B2 large regional hospital
Table A3: Demographic description of participants
Variable Frequenc Per cent Mean SD
y SD
Gender
Male 4 9.5
Female 38 90.5
Age 38.7 7.9
Education
Certificate of 3 7.1
Nursing Bachelor of 7 16.7
Nursing Graduate 27 64.3
Diploma Masters 5 11.9
Experience
Years as a registered 15.9 8.
nurse Years as an 11.5 4
emergency nurse Years 9.9 6.
as a Triage Nurse 1
5.
7
148
Appointment level
Registered Nurse 13 31.0
Clinical Nurse 14 33.3
Specialist Associate 2 4.8
Charge Nurse 5 11.9
Nurse Unit Manager 42 14.3
Nurse Educator 1 4.8
Other
Shifts worked at triage 1.95 1.0
Location
New South 14 33.0
Wales Victoria 8 19.0
Australian Capital 1 2.4
Territory Queensland 9 21.4
Northern 2 4.8
Territory 2 4.8
Tasmania 5 11.9
Western Australia 1 2.4
South Australia
APPENDIX E: ANSWERS TO
CONSOLIDATION/SELF-TEST
CHAPTERS
Answers to Chapter 11 triage scenarios
1. Ebony is a four-month-old girl who is brought to the ED by her
mother at 4.00 pm. Her mother states that the child has had
difficulty breathing for two days and has been worse overnight.The
child has been coughing and feeding poorly. Her fluid intake has
been approximately half that of a normal day and she has had a
decrease in the number of wet nappies. She has a moist-sounding
cough and no
audible wheeze. She is tachypneic with a respiratory rate of 60
breaths per minute. Examination of her chest shows mild use of
accessory muscles. On auscultation she has an expiratory wheeze.
Her skin is pink and she has moist mucous membranes.
1 2 3 4 5
Airway is patent.This child has mild respiratory distress as
evidenced by mild use of accessory muscles. Poor feeding and
reduced wet nappies indicate that this child may become
149 dehydrated if not treated early.
2. Laura is a 10-year-old girl who presents to the ED at 11.00 pm with
her older sibling saying that she has had abdominal pain for the
past few hours. She indicates that the pain is across the centre of
her stomach and paracetamol has not helped. She complains of
nausea and says that she has vomited once since the onset of
pain. When asked, she states that she has had normal bowel
motions. She is able to give her own history while leaning over
onto the desk, holding her stomach. Her skin is pink and she is not
short of breath.
1 2 3 4 5
There is no compromise to the primary survey. Pain has not been
relieved by oral analgesia and there is a moderate level of
distress associated with the pain.
3. Graham is a 55-year-old male who presents to the ED accompanied
by his partner. He states that he has been ‘bleeding from the back
passage’ since the previous night. He is very anxious about the
bleeding and reports that it was ‘bright red’ in colour and ‘filled the
toilet bowl’ on two occasions. His blood pressure is 155/100; his
heart rate is 102 beats per minute; his respiratory rate is 20
breaths per minute.
1 2 3 4 5
There is no compromise to airway or breathing. From the history
it appears that the blood loss is moderate and that the HR is
mildly elevated but there is no haemodynamic instability.
4. Louisa is a 24-year-old female who presents to the ED with her
friend after ‘fainting’ in the toilet at home. She is complaining of
left-sided abdominal pains, which she has had ‘on and off’ for
several months. She previously attended the ED two weeks
ago for the same problem. An abdominal ultrasound was
performed at that time but identified no abnormalities. She rates
her pain as ‘six out of ten’. Her heart rate is 82 beats per minute
and her respiratory rate is 18 breaths per minute. Her
skin is cool and dry. She looks pale and uncomfortable.
1 2 3 4 5
There is no compromise to airway, breathing or circulation. Pain is
moderate to severe and is causing distress.
5. A mother presents to the ED at 9.20 pm with her nine-week-old
son, Christopher, stating that he has had a fever since 4.00 pm
that afternoon. She gave him paracetamol at 5.00 pm. She says
that he normally vomits after feeds but has vomited once this
evening between feeds. Christopher is breast-fed; he has fed less
frequently this evening. Christopher’s mother also informs you
that he had his first immunisation two days ago. He is in his
mother’s arms and is crying. He appears slightly pale. His hands
are warm but his feet are cold. Capillary refill is about two
seconds and he has moist mucous membranes and normal skin 150
turgor. His anterior fontanelle is not bulging.
1 2 3 4 5
Airway and breathing are not compromised. Skin is pale and
capillary refill indicates adequate peripheral perfusion. Dehydration
is not evident.The child is distressed despite being with his mother
and having been given analgesia.
6. Kimberley is 32 years of age. She was sent to the ED following an
accident at work. She was carrying a pot of hot oil and slipped,
spilling it on her upper legs. She immediately removed her
clothing and stood under a cool shower for 15 minutes.
On arrival in the ED she is in considerable pain (‘nine out of
ten’).You estimate that she has approximately eight per cent burns
to her anterior thighs. Her heart rate is 110 beats per minute and
her respiratory rate is 24 breaths per minute.
1 2 3 4 5
Burns of this nature cause severe pain. Analgesia should be given,
and patient should commence treatment within 10 minutes of
arrival.
7. Michaela is a three-week-old infant who is brought to the ED at
9.30 pm by her parents. She has been referred by her local doctor.
Her parents state that Michaela has been feeding poorly for
several days and that her weight gain has been poor. The infant
seems lethargic.The parents have not noticed a fever.The infant is
sleeping in her mother’s arms and her skin is pale. Her peripheries
are cool and her eyes slightly sunken. Painful stimulus is required
to wake the child, who then wriggles and cries vigorously.
1 2 3 4 5
The airway is clear and there is no compromise to breathing.The
child shows signs of dehydration including lethargy and
drowsiness, and should therefore commence treatment within 10
minutes of arrival.
8. Toby is an 18-month-old boy who presents to triage at 6.00 pm with his parents.
They state that he has been ‘unwell’ for two days; he started vomiting 48 hours
ago, developed diarrhoea yesterday and has had seven loose stools
today. He has had episodes of ‘crying and drawing up his legs’. He
is drinking small amounts. He appears lethargic and uninterested in
his surroundings. He is pale and his capillary refill is approximately
151 1 2 3 4 5
Airway and breathing are not compromised. Multiple signs and
symptoms of dehydration are evident including lethargy and poor
capillary refill.The child also appears to be in pain and very
distressed with his illness. He needs to commence treatment within
10 minutes of arrival.
three to four seconds.
9. Edward is a 36-year-old male with a past history of alcoholism. He
presents to triage at 5.30 pm. He has a referral letter from the
nearby drug and alcohol service
and an escort.The referral letter states that the patient has ‘suicidal
ideation and homicidal thoughts’.The letter requests a psychiatric
assessment and states that the patient is ‘possibly experiencing
alcohol withdrawal’. He states that his last drink was at 9.00 am.
1 2 3 4 5
This patient has no compromise to this airway, breathing or
circulation.There is no report of acute behavioural disturbance or
agitation at this stage.You will need to determine whether it was
9.00 am today or the day before that the drink was consumed
(possible alcohol withdrawal state).
This patient has an escort and will be under direct surveillance
from that person. He should not leave the waiting room without
this support person. He must also be under close observation of
the Triage Nurse. Security should be informed that this person is in
the ED.
The patient has suicidal ideations and is at risk of self-harm.
He should wait no longer than 30 minutes to commence
treatment. Re-triage may need to be performed if the patient
shows signs of behavioural disturbance.
10. Rae, a 24-year-old university student, comes to the ED with a
friend. She has a four-hour history of generalised abdominal
pain now localised to the right iliac fossa. She has vomited
twice and had one episode of diarrhoea about two hours ago.
Her heart rate is 92 beats per minute and her temperature is
38.2°C.
1 2 3 4 5
The airway, breathing and circulation are not compromised. She
has pain and likely intra-abdominal pathology as evident by
localising pain, fever and vomiting. She should wait no more than
30 minutes to commence treatment.
11. A father presents to the ED at 8.00 pm with his three-and-a-half-
year-old daughter, Savannah, stating that she has had a sore
throat for ‘a day or two’. It started with a runny nose and a fever,
and then yesterday she began complaining of a sore throat. She
has no cough or stridor, she demonstrates no shortness of breath
and her skin is pink and warm
1 2 3 4 5
Airway, breathing and circulation are intact. She is experiencing
some discomfort from her condition and should therefore
commence treatment within an hour.
12. Baz, 34 years old, was installing a ceiling fan with the 152
assistance of a friend in his own home. He received a 240 volt
charge to his right hand, and was thrown back against the roof.
His friend immediately switched the power off and called an
ambulance. Baz had a brief period of loss of consciousness, but
was alert when
the ambulance crew arrived. His heart rate is 80 beats per minute
and irregular; his respirations are 20 breaths per minute. He has a
five centimetre blackened area to his right hand. No exit wound is
seen.
1 2 3 4 5
Airway, breathing and circulation are intact. Likely full-thickness
burn from electrocution indicates severe localised trauma, with
possible systemic involvement. This patient should wait no more
than 10 minutes to commence treatment.
13. Hannah is a 41-year-old woman who presents via ambulance
with an altered conscious state following collapse. She is 30
weeks pregnant (G3P1) and is normally well. She was out
shopping with a friend when she suddenly collapsed.
Ambulance officers report a fluctuating conscious state. At the
scene she tolerated an oropharyngeal airway but spat it out en
route. She is in a lateral position on the ambulance trolley with
supplemental oxygen via a mask. Her respiratory rate is 10
breaths per minute. Her SpO2 is 93 per cent; her heart rate is 130
beats per minute. Her skin is pale, cool and moist. Her blood
pressure is 190/110. Her Glasgow Coma Score is 10 out of 15. Her
temperature is 36.3°C.
1 2 3 4 5
Airway may be compromised due to fluctuating level of
consciousness.There is additional risk to her airway and
breathing due to the pregnancy.There is
hypoventilation and severe hypertension.There is an imminent risk
of deterioration. Simultaneous assessment and treatment must
commence immediately.
14. Mr J is a 74-year-old man who is brought to the ED by
ambulance at 5.10 am. He has acute shortness of breath and a
history of left ventricular failure. His heart rate is 112 beats per
minute and irregular, his blood pressure is 180/100 and his
153 respiratory rate is 30 breaths per minute, with accessory muscle
use. His SpO2
is 89 per cent, but the pulse oximetry display is giving a poor trace. Oxygen is
being
administered at 100 per cent via bag-valve-mask. Mr J is trying to
remove the mask and is very agitated.
1 2 3 4 5
Airway is currently clear, however, there is severe respiratory
distress.There is acute shortness of breath and a mild
tachycardia.These signs and symptoms are possibly due to acute
left ventricular failure.
15. Bo is a 16-month-old boy who presents to triage at 11.00 am with
his mother. She states that he has had ‘a cold for over a week’
which ‘has not improved’. Since last night he has had a fever and a
cough and has seemed ‘more congested’. He was restless over
night, is tired today and is drinking less than usual. He is resting
against his mother and doesn’t protest when examined. No cough,
stridor or grunting is heard. He is tachypneic and demonstrates
mildly increased work of breathing. His skin is flushed and warm.
His capillary refill is less than two seconds and his mucous
membranes are moist.
1 2 3 4 5
Airway is patent.The child is tachypnoeic with mildly increased
work of breathing. Perfusion is not compromised.The child should
wait no longer than 30 minutes to commence treatment.
16. Luka is a nine-year-old boy who presents to triage with his
father at 3.00 pm. He has an injured elbow as a result of a fall
playing football. He is distressed and is clutching his arm, which
is in a sling. He tells you that his pain is ‘ten out of ten’.
His left elbow is markedly swollen and deformed. He has a strong
radial pulse, and sensation distal to the injury is intact. He is pale,
slightly diaphoretic and tachycardic.
1 2 3 4 5
Airway, breathing and circulation are intact.There is severe pain
and the child should wait no more than 10 minutes to commence
treatment.
17. Albert, 62 years old, often attends your ED.Today he says he is
constipated. His bowels have not opened for ‘at least two weeks’.
He says he has pain and feels bloated.When you ask him to score
his pain he is not sure what to say and just answers ‘it’s really
bad’. His vital signs are within normal limits and his skin is warm
and dry.
1 2 3 4 5
There is no compromise to airway, breathing or circulation.The pain
and discomfort have been present for more than two weeks.The
patient should wait no more than one hour before treatment is
commenced.
154
18. Sebastian is a 16-year-old boy who is brought to the ED by a
passer-by, who found him crying and banging his head against the
footpath in a small laneway. After bringing Sebastian to the triage
the accompanying adult leaves the ED. Sebastian has superficial
lacerations to both wrists, and is dishevelled and unkempt. He is
upset about having being brought to the ED, and is saying,‘just
leave me alone – why don’t you just piss off’. He admits trying to
hurt himself, and says that he will do so again as soon as he can.
1 2 3 4 5
Airway, breathing and circulation are intact.The patient has
attempted self-harm. His comments indicate that he may be at
high risk of absconding. He should commence treatment within 10
minutes of arrival in the ED.
19. Anne-Marie is a 22-year-old female who is brought to the ED by
her flatmates, who are concerned about her bizarre behaviour. She
had been talking to herself for several days, turning the television
off and on because it is sending her messages, yelling out at night
and not sleeping. Her flat mates are concerned that she will come
to some harm without help.
1 2 3 4 5
Airway, breathing and circulation are intact.There are signs of
thought disorder as well as bizarre and agitated behaviour.
20. Mohammed is a 24-year-old Somali man who is brought to the ED
by police. He is crying and lying on the floor, rocking. He smells of
alcohol, and police say he is a refugee who has recently been
released from a detention centre. He has committed no crime, but
was apprehended ‘directing traffic’ in the middle of a busy city
highway.
1 2 3 4 5
Airway, breathing and circulation are intact. A full medical
assessment will be required to identify any physical causes for the
behaviour. Psychotic symptoms are reported including delusions and
bizarre behaviour.This patient should commence treatment within
30 minutes of arrival.
21. Damien is a 36-year-old male who is brought to the ED by his
friend. He has had a recent marriage break-up, which involved a
lengthy custody and property court case. He has had symptoms
of depression for several weeks, including low mood,
ruminations, poor sleep and appetite, feelings of hopelessness
and agitation. Since
receiving the outcome of the Family Court hearing three days ago,
Damien has been using the amphetamine ‘ice’, and is now
‘obsessed with plotting revenge’ on his former spouse. He has been
awake for more than 48 hours, and presents as angry, rambling in
1 2 3 4 5
155
Airway, breathing and circulation are intact.There is extreme
agitation and possible threats of harm to others.This patient should
commence treatment within 10 minutes.
speech, volatile and disordered in his thinking.
22. Chloe is a 15-year-old girl who is brought to the ED from a friend’s
house after taking an overdose.The circumstances are unclear,
however, she admits to having taken 12 paracetamol tablets and
‘some other things’, including alcohol. She is known to the ED,
having presented 12 months ago following an episode of self-harm.
She is cooperative, coherent and not drowsy. Her breath smells of
alcohol.
1 2 3 4 5
Airway, breathing and circulation are intact. Actual self-harm has
occurred and the patient is at risk of the toxic effects of the
ingestion of paracetamol and other tablets which are as yet
unknown, thus full physical assessment is required in the ED.This
patient should commence treatment within 30 minutes of arrival.
23. Leonie is a 29-year-old woman who presents to triage with her
mother. She has had three days of abdominal pain and vomiting.
She tells you she is 32 weeks pregnant (G2P1) and is an insulin-
dependent diabetic. Her main reason for coming to the ED is that
she couldn’t get an appointment with her obstetrician and the pain
is ‘worrying’ her. She appears a little short of breath and her
respiratory rate is 28 breaths per minute. Her SpO2 is 98 per cent.
Her heart rate is 128 and her skin is
pale, warm and dry. She is alert and oriented and her Glasgow
Coma Score is 15 out of 15. Her temperature is 37.2°C (tympanic).
1 2 3 4 5
Airway is intact and there is slight shortness of breath and
tachypnoea.The heart rate is elevated.The patient is experiencing
abdominal pain. She also has a significant co-morbidity, being an
insulin-dependent diabetic. Notwithstanding these factors, she is
ventilating well and is alert and orientated. She should wait no
more than 30 minutes to commence treatment.
24. Paul is a 47-year-old male. He has a painful left shoulder, and
received treatment in the ED for the same problem two days
ago.There is no history of injury, but
Paul tells you that his shoulder is stiff and keeps ‘seizing up’. He
tells you that he was prescribed some pain killers that worked
initially, but that the pain is back and
156
is ‘much worse now’. He is crying in pain. His left hand is pale and
cool; a week radial pulse is noted. His right hand is pink and warm.
1 2 3 4 5
Airway, breathing and circulation are intact. Neurovascular
assessment indicates a limb-threatening condition.The patient is
experiencing severe pain and so should wait no longer than 30
minutes to commence treatment.
25. Gillian is a 26-year-old woman who presents via ambulance
with palpitations. She is 34 weeks pregnant (G1P0) and is
normally well. She tells you that she was out
shopping when her palpitations started. She does not have any associated chest
pain
or shortness of breath. Her respiratory rate is 20 breaths per
minute. Her SpO2 is 98 per cent. Her heart rate is 108 beats per
minute and her blood pressure is 120/80. Her skin is pale, warm
and dry. Her Glasgow Coma Score is 15 out of 15.
1 2 3 4 5
Airway, breathing and circulation are intact.The heart rate is mildly
elevated and the patient is experiencing palpitations. She should
wait no longer than 30 minutes to commence treatment.
26. Mal is a 28-year-old male who presents to triage saying
that he has been bitten by ‘some sort of insect’. He was
clearing rubble from a building site about two hours ago
when he felt a sudden burning sensation in his right hand.
He said ‘I flicked something off but I didn’t see what it
was’. Over a period of two hours his right arm has become
increasingly painful and he
is sweating. He is complaining of a frontal headache. He is
alert and oriented to time, place and person. His heart rate is
98 beats per minute and his respiratory rate is 22 breaths
per minute.
1 2 3 4 5
Airway, breathing and circulation are intact. Envenomation is likely
from the history, and the increasing pain warrants treatment within
30 minutes.
27. Thuy, a 44-year-old woman, presents to the ED with back pain.
She has had the problem on and off for many years.This current
episode was brought on after lifting a light shopping bag from her
car four hours ago. She has taken Nurofen with little improvement.
Currently she has no general practitioner so she ‘didn’t know
where else to go when the pain happened’. Her vital signs are
1 2 3 4 5
157
Airway, breathing and circulation are intact.The pain is due to an
acute back injury and the patient should wait no longer than one
hour to commence treatment.
within normal limits and she is not sure how to rate her pain but
says it is ‘very bad’.
28. Patty is a 53-year-old female who presents to triage complaining
of right-sided abdominal pain. She states that the pain has been
constant for two days now. She has not had any nausea or
vomiting. She tells you that the pain is worse when she
is sitting still. She states that she has had this pain before and that
her doctor thought it might be gall stones. Prior to coming to the ED
she took two paracetamol with minimal effect. She rates the pain
as ‘five out of ten’. Her blood pressure is 145/84, her heart rate is
96 beats per minute and her respiratory rate is 18 breaths per
minute. Her temperature is 36.4°C.
1 2 3 4 5
Airway, breathing and circulation are intact.The patient has
abdominal pain with no associated nausea or vomiting.
Symptoms are moderate, and treatment should commence within
one hour.
29. Emil is a five-year-old boy with a seven-day history of diarrhoea
and vomiting. He presents to the ED with his mother at 9.30 pm.
He has been unable to keep food or fluids down today. He is pale,
lethargic and drowsy. His heart rate is 124 beats per minute and
his respiratory rate is 20 breaths per minute.
1 2 3 4 5
Airway is intact and there is no respiratory distress.The patient is
mildly tachycardic. He should wait no longer than 30 minutes for
treatment.
30. Catherine is a four-year-old girl who is brought to the ED at 4.30 pm with a
12- hour onset of being unwell. In the past four hours she has
developed a petechial rash on her abdomen. She also has a runny
nose and a fever (her temperature is 37.8°C per axilla). She has
been tolerating sips of oral fluid but now seems drowsy.
1 2 3 4 5
Airway, breathing and circulation are intact. History
suggests suspected meningococcaemia.Treatment
should commence within 10 minutes.
31. Lee is a 20-year-old female who presents to the ED with her
mother. Her mother reports that she has had paranoid
hallucinations and that since yesterday she has not taken any
fluids. She states that her reason for not drinking is that she
believes that there are ‘spiders and poison around’.
1 2 3 4 5
Airway, breathing and circulation are intact.The presence of
psychotic symptoms (paranoid ideas) indicates that treatment
should commence within 30 minutes.
32. Candy, a three-month-old female, presents to the ED with her
mother. She has been referred by the maternal child and health
nurse. According to her mother, the infant has been ‘crying a lot’ 158
and has ‘bad colic’.The baby was born prematurely at 36 weeks,
and was delivered by emergency caesarean section due to
preeclampsia. Since birth, the baby has gained weight and her
mother says that apart from the colic she ‘is doing OK’.When you
examine the baby you note green/yellow bruising and red welts on
her upper arms.
1 2 3 4 5
Airway, breathing and circulation are intact. A number of risk
factors suggest this child is at risk of abuse; accordingly the child
should wait no longer than 30 minutes for assessment.
33. Nathan is a 45-year-old man who presents to the ED with his wife
and child. He asks to see a psychiatrist because he has been
having problems managing his anxiety about his work situation,
and he doesn’t know how to get a referral. He reports that he once
saw a psychiatrist, four years ago, and that it helped him sort out
his troubles, but that he can not remember the doctor’s name. He
is on no medication and has no active thoughts of harming himself;
he says that he ‘just needs to sort out his anxiety’.
1 2 3 4 5
Airway, breathing and circulation are intact.The patient has brought
himself in to
the ED to access help. He reports a pre-existing mental health
disorder (depression) and demonstrates that he is cooperative and
able to engage in developing his own management plan. He should
wait no longer than two hours before treatment is commenced.
34. Brian is a 39-year-old male who walks to the triage desk. He says
he fell in his driveway and now has left shoulder pain. On
examination his shoulder is very swollen and painful on
movement. His arm is already in a sling. His left hand is warm and
a radial pulse is present.
1 2 3 4 5
Airway, breathing and circulation are intact.There is an acute injury
causing pain, but there is no circulatory compromise to the
effected limb.The patient should wait no longer than one hour
before commencing treatment.
35. Bianca is 24 years old. She has a history of a perianal abscess,
which underwent drainage two days ago. She continues to have
pain (‘six out of ten’) and was seen by her local doctor today. She
has taken Panadeine Forte with no relief and is also on oral
antibiotics.
1 2 3 4 5
Airway, breathing and circulation are intact. Pain is due to acute
infection which is being treated.The patient has taken analgesia
but continues to experience a
moderate level of pain.The patient needs to undergo a review of
her condition by a medical officer within one hour of arrival.
159
36. Craig is an 18-year-old male patient who presents saying he
feels ‘suicidal’ and requesting admission. He makes a verbal
threat to ‘cut up’ if he is not admitted.
1 2 3 4 5
Airway, breathing and circulation are intact.The patient reports
suicidal ideation and wants to be admitted. He is seeking help for
his condition so there is no risk of absconding, from the
information available. He should receive treatment within
30 minutes.
37. Karen, a 36-year-old female, presents to triage accompanied by a social worker.
She has come from the plaster clinic. She has increasing pain in her
left foot from a fractured right fibula which she sustained yesterday
morning. A lower-leg plaster was applied in the ED last evening.The
social worker tells you that the patient has a history of depression
and has said that she wants to ‘end it all’.
1 2 3 4 5
Airway, breathing and circulation are intact. A pain assessment is
required at triage. The patient reports having suicidal ideation. She
is accompanied by a social worker in whom she has confided. She
should be under close observation in the ED waiting room and it is
desirable that the social worker wait with her in the waiting room
until she is assessed by a medical officer. She should receive
treatment within
30 minutes.
38. Ida is a 66-year-old female who presents to the ED alone. She
states that she is on Aropax and is having ‘suicidal’ ideation. She
tells you that she has two possible plans to harm herself. She says
she is having an anxiety attack and reports poor sleeping and
eating patterns for the past two weeks.
1 2 3 4 5
Airway breathing and circulation are intact.The patient reports
having suicidal ideation, and is independently seeking help. She
should be under close observation in the ED waiting room, and
should receive treatment within 30 minutes.
39. A 52-year-old male presents to triage. He has a history of schizophrenia. He
is currently on medication for his condition but can not recall the
name of the medication, or the name of his case manager. He
says that he has been having suicidal thoughts and that ‘there
are voices’ urging him to ‘step in front of a train’.
1 2 3 4 5
Airway, breathing and circulation are intact.There is evidence of
psychotic symptoms (command hallucinations) and suicidal
ideation.The patient should be under close observation in the ED
waiting room, and should receive treatment within 30 minutes.
40. Rohan, a 50-year-old male, has been brought to the ED by the 160
district nurse.The nurse states that he has a history of alcohol
abuse and that he is feeling ‘suicidal’. She notes also that over the
past week he has been neglecting his general care.The patient
has a history of an intracerebral bleed (two years ago) and he is
deaf.
1 2 3 4 5
Airway, breathing and circulation are intact. Extra help is required
for communication and thus an interpreter should be involved for
signing.The main risk is suicidal ideation.The patient should be
under close observation in the ED waiting room; ideally the nurse
should wait with him until he is seen by a medical officer, because of
his communication needs and cognitive impairment. He will need to
be re-triaged if he develops signs of agitation. He should receive
treatment within 30 minutes.
41. While playing volley ball, Gary, 47, hurt his left wrist. He
has a good range of movement but reports pain when asked
to rotate his left hand.
1 2 3 4 5
Airway, breathing and circulation are intact.The main problem is
pain.The patient should receive treatment within one hour.
42. Janine is a 56-year-old woman who presents to the ED with her
partner at 2.30 am. She has pain in the epigastric region which has
been increasing since yesterday.The pain radiates to her lower
abdomen and she says that she has been vomiting clear fluid
tonight. Her bowels last opened two days ago. She is on Oridus and
has a history of hypertension.
1 2 3 4 5
Airway, breathing and circulation are intact.There is localised
abdominal pain and a risk of intra-abdominal bleeding due to
medication. Persistent vomiting will add to the patient’s
discomfort. She should be treated within 30 minutes.
43. Mr D, 84, has a chronic leg ulcer.The district nurse has sent him
to the ED because she believes the wound is infected. Mr D has a
history of hypertension and ischemic heart disease. He lives with
his daughter, who normally helps him out with his daily living, but
she has gone to Queensland for a holiday.The wound is covered
when you see him, but the bandage is soiled with what appears to
be haemo-serous ooze. His temperature is 35.9°C and his vital
signs are within normal limits.
1 2 3 4 5
Airway, breathing and circulation are intact.The main problem is
161 suspected infection. The patient has co-morbid factors
(hypertension and ischemic heart disease). He should commence
treatment within one hour.
44. Nic, a 38-year-old aborist, has cut his left arm with a chain saw. He
was brought to the ED by a workmate. He has a deep laceration of
about ten centimetres to the inner aspect of his arm.The wound
was bleeding ‘quite a bit’, but the blood loss has been controlled
with a firm bandage. He tells you that the wound is ‘not that
painful’, but he looks pale and is sweating. His heart rate is 84
beats per minute and his respiratory rate is 20 breaths per minute.
His workmate reports that the dressing was changed once, half an
hour ago, because it was soaked with blood.
1 2 3 4 5
Airway and breathing are intact and there is no haemodynamic
compromise despite moderate blood loss.This patient should wait
no longer than 30 minutes before treatment is commenced. A
clean dressing and firm bandage should be applied to the wound. A
sling should be applied and elevation of the wound should also
occur. Observation of ongoing blood loss needs to occur. Re-triage
will be required if the patient develops signs of haemodynamic
compromise or if the blood loss is not stemmed with basic pressure
immobilisation.
45. Liam is a 23-year-old male who presents to triage after being
seen by a locum doctor. He is backpacking around Australia and
has been staying in a boarding house near the hospital. His partner
has brought him to the ED. He has a six-hour history of fever and
lethargy. He has been vomiting, and complains of a headache.The
doctor gave him intramuscular Maxalon, with some effect. His
temperature is 38.4°C, and his partner points out a fine petechial
looking rash on his torso. He is drowsy but oriented to time, place
and person
1 2 3 4 5
Airway, breathing and circulation are intact.The patient has signs
of meningococcaemia and needs to commence treatment within
10 minutes.
46. Ashley, a 23-year-old university student, fell off her bicycle two
days ago and was seen in another ED. She is complaining of
stiffness and pain to her left wrist. Her left hand is swollen but she
has full range of movement; her left hand is pink and warm.
1 2 3 4 5
Airway, breathing and circulation are intact.This injury occurred 48
hours ago and was treated at that time. Function of the limb is not
impaired and there are no high risk features to this presentation.This
patient should receive treatment within two hours.
162
47. Remo is a 43-year-old male who presents with a two-week history
of right renal stones. He now has pain, which he describes as
‘colicky’ in nature. He rates the pain as ‘four out of ten’. He has
had no pain relief today.
1 2 3 4 5
Airway, breathing and circulation are intact.The pain is likely to be
due to renal calculi. Pain is mild to moderate and there are no high-
risk factors.This patient should be seen within one hour.
48. Angie is a 27-year-old woman who presents via ambulance
following a high-impact motor vehicle accident. She is 38 weeks
pregnant (G2P1) and is normally well. She was a passenger in a car
that collided head-on with another vehicle in an 80 kph zone.The
ambulance officers report significant damage to both vehicles.
Angie was wearing a seatbelt and the passenger airbag was
deployed. She has good recall of events but complains of a painful
chest and abdomen and has visible seatbelt marks. She also has a
facial abrasion and lacerations to both her knees. She has a
cervical collar on; oxygen is at 10 litres per minute via mask and
500mls crystalloid fluid in
progress intravenously. Her respiratory rate is 28 breaths per
minute, her SpO2 is 93 per cent and her heart rate is 134 beats per
minute. Her skin is pale, cool and dry. Her Glasgow Coma Score is
15 out of 15. Her blood pressure is100/R. Her pain is ‘six out of
ten’. She has no PV loss.
1 2 3 4 5
Airway is intact, the patient has mild tachypnoea and a lower than
expected SpO2 and is tachycardic. She is also relatively
hypotensive despite volume replacement.The mechanism of injury
indicates a significant force and she has moderate pain. Due to
these factors she should receive treatment within 10 minutes.
49. Norm is a 60-year-old man who arrives at triage at 9.20 am. He is
ambulating using a walking stick.When asked what is wrong he
163 points to his abdomen and chest and says,‘This is as tight as billy-
o. I got stirred up yesterday – I had a barney with a bloke up home,
and then the tightness got worse, like a vice’. On examination you
find that his heart rate is within normal limits and is regular. His
skin is warm and dry. He is not short of breath. His SpO2 is 95 per
cent on room air.
1 2 3 4 5
Airway, breathing and circulation are intact.The patient has chest pain
which is likely to be cardiac in origin.Accordingly, he should wait no
longer than 10 minutes for treatment.
50. Ann is a 16-year-old female who walks to triage with her mother.
She reports that she injured her left wrist while playing volley ball.
On examination you note good range of movement but she still has
some pain. She says the pain is ‘three out of ten’.
1 2 3 4 5
Airway, breathing and circulation are intact.The pain is mild but
exists in the context of an acute injury.The patient should wait no
longer than one hour for treatment.
51. Mr A is a 54-year-old man who has been sent to the ED by his
local doctor. He is unsteady on his feet and requires the
assistance of his son to walk. His referral letter reads:
Dear Doctor,
Please assess this man who was recently admitted to your
hospital with left renal calculi. He has been complaining of
dizziness and headache for several days. No focal weakness,
visual disturbance or confusion. Seen for same 2/7 ago no
improvement with Stemitil. PMx, IHD, NIDDM, renal calculi,
hypertension. Blood pressure: 215/130. Please assess.
Via translation through his son, Mr A tells you that he is ‘very dizzy’,
feels ‘weak all over’, has pain in his back and his abdomen and has
vomited twice today.
1 2 3 4 5
Airway and breathing are intact.The patient has severe hypertension
and has a number of co-morbid conditions. He is also experiencing
pain in the abdomen and discomfort from vomiting. He should wait
no longer than 30 minutes for treatment.
52. Jake, 46 years of age, presents to triage with his carer. He is
crying because he has abdominal pain and has a recent history of
a small bowel obstruction (six months ago). Jake has an intellectual
disability, and lives in a community residential unit with three
164
other adults and supervisory staff. His carer says that he is
‘normally able to attend to his activities of daily living under
supervision’, and that he ‘usually tolerates a lot of pain before he
will let staff know he is unwell’. In fact, his carer says that ‘last time
he was hospitalised he had been ill for quite a while before staff
actually realised that there was a problem with his health’. His
heart rate is 120 beats per minute and his respirations are 26
breaths per minute. His skin is pale, cool and moist.
1 2 3 4 5
Airway is intact.The patient is mildly tachypnoeic and tachycardic.
He reports pain and although the severity is unclear his behaviour
indicates at least a moderate level of distress. He should wait no
longer than 30 minutes for treatment.
53. Jane is a 17-year-old girl who was sent to the ED by her local
doctor. On her way home from school her boyfriend noticed that
she had become drowsy,‘she kept asking where she was, and
appeared disoriented’. She was seen by her local doctor who told
her to ‘go straight to the ED’. He did not provide her with a letter
of referral. Her Glasgow Coma Score is 14 out of 15.
1 2 3 4 5
Airway, breathing and circulation are intact.The history is unclear and
there is an altered conscious state.The patient should wait no longer
than 30 minutes for treatment.
54. Jonny, 34 years of age, has an abscess under his tooth. He presents to the ED at
1.30 am. He is in pain (‘six out of ten’) despite having taken
Panadeine and Nurofen regularly. He has an appointment with his
dentist tomorrow, but has not been able to sleep because of the
pain. He is afebrile.
1 2 3 4 5
Airway, breathing and circulation are intact.The main problem is pain,
possibly due to infection. Pain is at a moderate level and treatment
should commence within one hour.
55. Rose is a 47-year-old female who presents to triage with a
letter from her local doctor. She makes no eye contact when
you speak to her. The letter reads:
Dear Doctor,
Please assess Rose, a 47-year-old woman who lives alone.
She has a history of cholelithiasis and schizophrenia. She has
some burns on her inner thigh which require your attention.
On questioning, Rose tells you that her burns occurred two days
ago, and that they are red and itchy.When you ask her how she
sustained the burns she says she isn’t sure.
1 2 3 4 5
165 Airway, breathing and circulation are intact.The patient has a pre-
existing mental health disorder. No agitation is noted in the scenario,
and the patient has independently sought help.The mechanism of her
injuries requires investigation – they may be the result of abuse or self-
harm.Treatment should commence within 30 minutes.
56. Adrian is a 13-year-old boy who presents to the ED via ambulance at 10.00 am.
The ambulance officer states that he was hit by a car with a bull
bar, and was thrown several metres. He is complaining of pain in his
neck and legs. He has a cervical collar in place. He looks pale. He is
tachycardic and tachypneic. He answers questions appropriately
and is able to move all limbs on request.
1 2 3 4 5
Airway is clear.There is possible cervical spine injury, tachypnoeic
and tachycardic. The mechanism of injury indicates significant
force.The patient should wait no more than 10 minutes.
57. Edna is 93 years old. She has been transferred to the ED via
ambulance from a nearby aged care facility. For the past two
weeks she has not been eating much, and yesterday was only
taking small amounts of oral fluids.Today she was found to be semi
conscious and with a fever (39.8°C). She has a history of ischemic
heart disease, heart block and hypertension. She has a dual-
chamber pace maker. She also has a three-year history of
dementia. On arrival her respiratory rate is 28 breaths per minute;
her heart rate is 68 beats per minute. Her skin is hot and moist,
and her Glasgow Coma Scale is 9 out of 15.
1 2 3 4 5
Airway is intact.The patient is tachypnoeic, tachycardic and has an
altered level of consciousness. She is possibly septic and treatment
should commence within 10 minutes.
58. Rodney is 43 years old. He was escorted to the ED by police,
having been apprehended climbing out of a window of an
abandoned warehouse.While trying to escape he cut his right hand
on some broken glass. He has a deep, six centimetre laceration to
the palm of his right hand.There has been minimal blood loss, but
he says he can not feel his right index or second finger at all.
1 2 3 4 5
166
Airway, breathing and circulation are intact.The injury has caused
neurovascular impairment.Treatment should begin within 30
minutes.
59. Mr G is a 53-year-old male who walks to the triage desk
unassisted. He is short of breath. He states that he was recently a
patient of this hospital. He has cancer of the liver and had a
peritoneal tap 10 days ago for acities. He also tells you that he
‘needs draining again’. His respiratory rate is 24 breaths per
minute and his heart rate is 92 beats per minute.
1 2 3 4 5
Airway, breathing and circulation are intact.The patient is
experiencing discomfort due to his acities.
60. Heidi, a 17-year-old female, presents to the ED complaining of a sore throat. She
has a hoarse voice and her friend states that she also has muscular
pain to her neck, shoulders and back. She has been unwell for a
few days, but has come to the ED today because she is having
trouble swallowing.You ask her to open her mouth
and note that her breath is foul-smelling. Her tonsils appear to be
covered in pus. Her temperature is 39.4°C.
1 2 3 4 5
Airway, breathing and circulation are intact.The patient is septic and
treatment should commence within 30 minutes.
61. Over the past four weeks Gregory, 56, has attended your ED 14
times.Today he says he has a problem with a tattoo that was
applied ‘by a mate’ two weeks ago.The
wound looks red and is oozing pus. Gregory has a history of alcohol
and intravenous drug use, hepatitis C and type 2 diabetes. His vital
signs are within normal limits.
1 2 3 4 5
Airway, breathing and circulation are intact.The patient has an
infection to the skin and has a number of co-morbid conditions. He
should commence treatment within one hour.
62. Larry, 62, stubbed his right big toe on the corner of a
fireplace.The nail has lifted right off and the toe is now covered
with a blood-soaked tea-towel. Larry walks into the ED assisted by
his son. He tells you that he takes Warfarin, so ‘thought it best to
come to hospital rather than see the local doc’. His son tells you
that the tea-towel has not been changed since the injury, but that
there was ‘blood all over the floor’.
1 2 3 4 5
Airway breathing and circulation are intact.There is moderate blood
loss and a clean, firm dressing needs to be applied to the
wound.The patient should wait no longer than one hour for
treatment, however, close observation of the dressing needs to
occur and re-triage should be performed if bleeding can not be
adequately controlled
167
63. Carole, 48 years, is brought to the ED by her husband. She is
vomiting and has severe epigastric pain. She ate at a local
restaurant and tells you she thinks that she has ‘food poisoning’.
Her heart rate is 98 beats per minute; her respiratory rate is 26
breaths per minute. Her skin is pale and moist to touch. She says
that the pain comes and goes: she rates it as ‘eight out of ten’ at
the worst point and ‘two out of ten’ at the lowest point’. She has
vomited semi-digested food more than six times in the past hour.
Now the vomit is clear fluid.
1 2 3 4 5
Airway, breathing and circulation are intact. Pain is the problem, and
it is reported to be severe.Accordingly, the patient should wait no
longer than 10 minutes for treatment.
Answers to Chapter 12 triage scenarios
1. Albert, 44 years, was mowing the lawn on Sunday morning when a
foreign body flicked up out of the mower and into his eye. His wife
drove him to the hospital. On presentation at triage, his eye is
tearing quite a bit, and he still has the sensation that ‘something is
there’. He says he has ‘no pain’ but the eye is ‘uncomfortable’. A
quick check of vision reveals that he has no problems with visual
acuity.
1 2 3 4 5
Airway breathing and circulation intact. Foreign body in eye with no
change to visual acuity. He should wait no longer than 60 minutes.
2. Shane is a 30-year-old male who presents to the ED complaining of
having a frontal headache.The patient states he has been thinking
of harming himself and wants to ‘get help’. If he can not sort out
his problems, he says he will ‘go and jump off a bridge’.
1 2 3 4 5
Airway breathing and circulation are intact.The main problem is the
pain and this must be investigated thoroughly before assuming the
problem is psychiatric. Suicidal ideation but is seeking help.This 168
patient should be under close observation in the waiting room. If he
develops signs of agitation, he should be re-triaged. He should wait
no longer than 30 minutes.
3. Violet is a 91-year-old female of non-English speaking background.
She is brought to the ED in her son’s car after seeing her local
doctor.You are called to assist her
to get out of the car. Her son tells you she is ‘very sick’.You note that she is able
to transfer to a wheelchair with minimal assistance. According to
her son,Violet fell three days earlier and has bruised her right hip.
She is able to walk, though the hip is very painful. She is not
distressed when seated. Her blood pressure is 150/90, her heart
rate is 88 beats per minute, and her respiratory rate is 20 breaths
per minute. You are unable to ascertain her exact level of pain,
though she tells you she is ‘alright’.
1 2 3 4 5
Airway breathing and circulation intact. Her hip is causing pain on
movement but the patient is able to weight-bear. She should wait
no longer than 60 minutes.
4. Glen, 52 years, presents to the ED with ‘bleeding haemorrhoids’.
He has had this problem ‘on and off for the past few months’, but
now it is ‘getting worse’. He says he has considerable pain when
he opens his bowels and bleeds ‘quite a bit’ (about half a cup at a
time for the last two days). He states that he needs to be seen by
a doctor ‘as soon as possible’ as he considers his problem is ‘an
emergency’.
1 2 3 4 5
Airway breathing and circulation intact. Mild haemorrhage. Pain
only occurs when bowels are opened. He should wait for no more
than 60 minutes.
5. Rebecca is a 17-year-old female who is brought to the ED by her
friends following an all-night party, where she took two tablets of
ecstasy. She can not stop crying, and says she wants to die. She
has had two previous minor overdoses in the past year.
1 2 3 4 5
Airway breathing and circulation intact. Severe symptoms of
depression.The patient’s friend should stay with her in the waiting
room. She should also be under close observation from the Triage
Nurse. She should wait no more that 30 minutes.
6. Charlie is a 15-month-old boy who presents to the ED via
ambulance at 2 am following an episode of ‘shaking and jerking’
with a loss of consciousness.The episode lasted approximately
four minutes.The ambulance officers state that he stopped fitting
when they arrived but he had been ‘very drowsy’ during
transport.
His mother states that Charlie has never had an episode like this before. During
the night, he had a fever and a runny nose. He has been sleeping
poorly and is a little irritable.The child is flushed and his skin is very
warm. He is tachypneic, but has no use of accessory muscles or
retraction. He is crying and clinging to his mother.
1 2 3 4 5
Airway breathing and circulation intact.There is no increase work of
169 breathing. History of fever and possible febrile convulsion.This child
should wait no longer than 30 minutes
7. Dianne is a 67-year-old lady who was out shopping with her
daughter when she slipped and fell on her outstretched hand
injuring the left wrist. She is not distressed by the pain and rates it
as ‘three out of ten’. Her wrist is tender, but not deformed. Radial
pulse is present at 72 beats per minute.
1 2 3 4 5
Airway breathing and circulation intact. Mild pain with no
circulatory compromise. This patient should wait no longer than 60
minutes.
8. Kate is 18 years old. She attends triage at 12.30 pm with a work
colleague. Her hand is wrapped in a tea towel and she appears pale
and anxious. She tells you she has cut her hand with a carving
knife. On examination you see a four centimetre laceration across
her left palm.Tendons are on view and the wound is bleeding
slowly. Kate tells you she is feeling quite nauseous and her pain is
‘seven out of ten’. Movement and sensation to her fingers are
intact.
1 2 3 4 5
Airway breathing and circulation intact. Blood loss is mild but
pain is severe. This patient should wait no longer than 30
minutes.
9. Denise is a 34-year-old female who is transferred to the ED on a
40°C day via ambulance. According to bystanders, she was
attending an outdoor barbeque and ‘collapsed in the
heat’.Witnesses helped her to an upright position, and she was
then observed having a ‘fit’ that lasted approximately two minutes.
She was not incontinent and regained consciousness when she was
placed in the supine position. Paramedics attended and inserted an
intravenous cannula. Her Glasgow Coma Score is 15 out of 15;
heart rate 112 beats per minute and respiratory rate 22 breaths
per minute. Her skin is hot and moist.
1 2 3 4 5
Airway breathing and circulation intact. History suggests heat
stroke and seizure. Patient has normal neurological functioning
now.The patient should wait no longer than 30 minutes.
10. Justin is a 22-year-old male who comes to the ED concerned
about a mole on his back. He says that his girlfriend advised him to
see a doctor and he is ‘worried that it might be a melanoma’.The
mole is large and irregular in shape; he says it is sometimes itchy.
1 2 3 4 5
Airway breathing and circulation intact.This is not an urgent
problem, however the lesion needs to be checked for melanoma
170
and this should occur within two hours.
11. A young man is brought in to the ED by ambulance after having
been dragged unconscious and not breathing from the sea in Far
North Australia. He arrives with full CPR in progress and you note
that he has red welts across his chest.
1 2 3 4 5
Cardiopulmonary arrest. Possible jelly fish envenomation.The
patient should commence treatment immediately.
12. Fred, an 84-year-old man, presents to triage complaining of
palpitations and central chest pains. He has a history of ischemic
heart disease, coronary artery by-pass grafts and atrial fibrillation.
He takes his anti-arrhythmic medications regularly and normally
manages well at home.Today his skin is pale, cool and moist, and
his heart rate is 142 beats per minute and irregular.
1 2 3 4 5
Airway intact. Chest pain is likely to be cardiac in origin.This
patient should be treated within 10 minutes.
13. A man states that his three-week-old baby grandson, Kyle, is ‘not
breathing properly’. The baby is wrapped in a bunny rug held by his
grandfather. On closer examination, you note the baby’s eyes are
open and his face is white/grey. Respiratory effort is poor.
1 2 3 4 5
Cardiopulmonary arrest is imminent.The child should be treated
immediately.
14. Robyn is a 38-year-old woman with a history of asthma. She has
required two admissions to the intensive care unit for her asthma
in the past 18 months. She presents to triage at 8.30 pm following
a 22-hour history of wheeze and shortness of breath. She has been
self-administering Ventolin at home but has had a minimal
response despite the use of three nebulisers in the past hour. On
arrival to triage, her respiratory rate is 26 breaths per minute; she
is speaking three-word sentences and has an audible wheeze.
1 2 3 4 5
Airway is intact. Significant level of respiratory distress in the
context of history of previous severe asthma requiring ICU
admissions.This patient should be treated within 10 minutes.
171 15. Caroline is a 45-year-old female who presents to triage
complaining of a ‘cold’ for the past four days. In the past two
days, she has pain in her right upper quadrant. The pain is now
increasing and she describes right thoracic ‘back pain’. Caroline
states that she has no diarrhoea, vomiting or urinary symptoms
but has had ‘difficulty breathing since yesterday’. Her skin is
pale, hot and moist, and she has normal respiratory effort.
Caroline says she has a fever and her heart rate is
112 beats per minute. Her respiratory rate is 26 breaths per
minute and she says that her pain is currently ‘seven out of
ten’.The pain is worse on deep inspiration and movement.
1 2 3 4 5
Airway is intact. Patient is mildly tachypnoeic febrile and is
experiencing pleuritic and upper abdominal pain. She should
receive treatment within 30 minutes.
16. Neil is a 74-year-old male who presents to triage following trauma
to his left arm after slipping on a wet floor. He describes tenderness
at his wrist, elbow and shoulder. He rates his pain as ‘three out of
ten’. No obvious deformity of the wrist is noted, but he has a
decreased range of movement. His heart rate is 92 beats per
minute.
1 2 3 4 5
Airway breathing and circulation intact. No circulatory compromise
to injured limb. Pain is reported to be mild.
17. Harry is a 48-year-old man who regularly attends your ED for
various complications associated with his poorly controlled type 2
diabetes. He has no GP, lives in a special accommodation house
but frequently sleeps out in a local park.Today he presents to triage
complaining of a two-hour history of intermittent left-sided chest
pain that is ‘heavy’ in nature. On further questioning you establish
that his pain came on at
rest and radiates down both arms. His heart rate is 66 beats per
minute, respirations are 20 breaths per minute, and skin is cool and
dry.
1 2 3 4 5
Airway breathing and circulation intact. Chest pain is likely to be
cardiac in nature. Diabetes is a co-morbid factor.The patient should
receive treatment within 10 minutes.
18. Mr Wallace, 57 years, works for an energy company reading
gas meters. On his rounds today he was attacked by a dog and
bitten on the upper left leg. On inspection you note six to seven
square centimetres of skin loss.The wound is irregular, fat tissue
is exposed and it looks dirty.There is a small amount of blood
loss. Mr Wallace says the injury is ‘a bit painful’ but he is not
overtly distressed.
1 2 3 4 5
172
Airway breathing and circulation intact. Blood loss is mild. Pain is
reported as mild. This patient needs to receive treatment for his
injury within 60 minutes.
19. Kira is a seven-year-old girl who presents with a school teacher
having fallen from play equipment. Her mother is on her way to the
ED. She fell onto her right arm and has been complaining of pain
around her wrist. She ‘did not hit her head’ and does not complain
of pain anywhere else. Her arm has been placed in a sling but she
has not received any analgesia. Kira is tearful but states that her
arm is only ‘a little bit sore’.There is a small amount of swelling
around her distal forearm; there is no deformity and no
neurovascular impairment. She demonstrates tenderness over her
distal radius and has a limited range of movement of her wrist. She
has no other signs of injury.
1 2 3 4 5
Airway breathing and circulation intact. Injured wrist/forearm with
no circulatory impairment. Pain is mild.This child should commence
treatment within 60 minutes.
20. A mother presents at midday with her nine-month-old son,
Connor. She describes three days of fever, poor oral intake and a
‘runny nose’. On the first evening of
the episode he vomited once and had two loose stools overnight, but this has
not reoccurred. Connor had 180 ml of fluid this morning (his usual
intake is about 320 ml) and he had a normal number of wet
nappies. He was previously well and
he is fully immunised. He has signs of a runny nose, no cough is
heard and he has no signs of increased work of breathing. His skin
is slightly pale but warm and his mucus membranes are not dry. He
is quiet but looking around at his surroundings.
1 2 3 4 5
Airway breathing and circulation intact. Child is alert with mildly
decreased oral intake.This child should receive treatment within
60 minutes.
21. Alanna is a ten-week-old infant who presents with her parents.
She has a two-day history of increasing lethargy and poor feeding.
Her mother indicates that she has become unsettled and less keen
to feed over the past two days. She developed a fever yesterday
and had to be woken for feeds overnight, which is unusual. She was
born at term, has had her first immunisation and has no other
health problems. She does not demonstrate increased work of
breathing but is slightly tachypneic. Her skin is pale and her legs
173
are mottled, a little cool and demonstrate a capillary refill of three
to four seconds. She is lethargic but responds to painful stimuli.
1 2 3 4 5
Airway is intact. Respiratory rate is mildly evaluated and there is
poor peripheral perfusion.This child should wait no longer than 10
minutes to commence treatment.
22.A 76-year-old woman, Rita, is brought to the ED by her daughter
who found her wandering in a ‘dazed state’ outside her house.The
patient presents as agitated and confused, is picking at imaginary
things on her cardigan and is unable to give an account of herself.
1 2 3 4 5
Airway breathing and circulation are intact.The patient is acutely
confused and agitated. She will need to be under close observation
and should wait no longer than 30 minutes to commence treatment.
Her daughter should wait with her.
23. Liz is a 40-year-old woman who presents to triage with fever and
productive cough. She says she is not short of breath and does not
complain of any pain. She is 18 weeks pregnant (G4P3) and is
normally well. Her respiratory rate is 24 breaths per
minute, SpO2 is 96 per cent, and heart rate is 98 beats per minute.
Her skin is noted to be pale, warm and dry, Glasgow Coma Score is
15 out of 15, and her temperature
is 38.2°C.
1 2 3 4 5
Airway breathing and circulation are intact.The patient is febrile and
is likely to have an infection. She should wait no longer than 60
minutes.
24. David is a 40-year-old male who presents to triage complaining of
severe chest pain, saying he is having a ‘heart attack’. He says he
has no history of cardiac problems and his observations are within
normal range. He appears highly anxious and is hyperventilating.
Currently he says his pain is ‘ten out of ten’. His skin is warm and
moist.
1 2 3 4 5
Airway is intact.This patient reports severe pain and has some risk
factors for heart disease. Investigations must be conducted to rule
out cardiac causes for his pain and he should wait no longer than
10 minutes before treatment is commenced.
25. Lionel, 68 years, is transferred to your ED from a nursing home.
He has Alzheimer’s disease and for the past two days has refused
fluids.This morning his carer found him lying on the floor next to his
bed yet the cot-sides were up. She thought that he had probably
fallen because the blankets were also on the floor and he had been
incontinent of urine. Last week he was able to mobilise with a
frame and take himself to the toilet, but for the past two days he
has not had the energy to move at all and has needed assistance
going to the toilet. Since the fall he can not stand up and he seems
to be guarding his right hip. On arrival, he is lying on the trolley
174
groaning. His heart rate is 122 beats per minute, respiratory rate
24 breaths per minute and blood pressure is 110/70.
1 2 3 4 5
Airway is intact heart rate is elevated as is respiratory rate.There
is a possibility of a fractured hip and given cognitive impairment
pain difficult to assess.This patient is likely to be in pain and
discomfort from the injury. He should wait no longer than 30
minutes for treatment.
26. Nicholas is a three-year-old boy who presents with increasing
wheeze and shortness of breath. His mother indicates that he has a
history of asthma and has been in hospital before. He developed a
cold two days ago and he became increasingly wheezy yesterday.
His mother gave him Prednisolone this morning and he has had
hourly Ventolin at home. In the past two hours he has had three
doses of Ventolin; the last dose was 15 minutes ago. He has a tight
cough and a marked increase in work of breathing. Nicholas’s skin
is pale but warm; and he is distressed and restless.
1 2 3 4 5
Airway is clear, though there is marked increased work of
breathing.The child is distressed and restless. He should wait no
longer than 10 minutes.
27. Frankie is an 18-month-old boy who presents to the ED with his
mother by ambulance. He has a barking cough and is having
difficulty breathing. His mother describes a recent cold. He woke
this morning with the cough and seemed distressed. His breathing
is fast and noisy. He does not have a stridor but does have a
barking (croup-like) cough and mild increase in work of breathing.
His skin is pink and warm and he remains settled while with his
mother.
1 2 3 4 5
Airway is clear though there is increased work of breathing.This child
should wait no longer than 10 minutes for treatment.
28. Parents present with their 13-month-old child, Oliver, who has
a history of diarrhoea and vomiting.They state that he has been
unwell for ‘about six days’. It started with vomiting, which
persisted for two to three days, but this has since stopped.
Oliver developed diarrhoea on the second day, which has
continued. He
is willing to drink and has passed two loose stools today. He shows
no shortness of breath, his skin is pink and warm and his mucus
membranes are not dry. He is grabbing at your ID badge.
1 2 3 4 5
175 Airway breathing and circulation are intact.The child is alert and
active. He should wait no more than 60 minutes.
29. Mr Carver, an 87-year-old man, is brought to your ED in the early
hours of the morning with acute shortness of breath. He is sitting
upright on the ambulance trolley with a simple face mask in situ.
He is receiving eight litres of oxygen per minute. His heart rate is
116 beats per minute and irregular; blood pressure is 170/90;
jugular veins are visible and elevated. His skin is moist and pale.
He is unable to talk but he does nod when asked if he has chest
pain.
1 2 3 4 5
Airway is clear.The patient has marked increased work of breathing
and shows signs of acute left ventricular failure. He also has pain in
the chest. He should wait no more than 10 minutes for treatment.
30. A father presents at 6.30 pm with his 22-month-old son, Jackson, who has cut
his forehead after tripping and falling against the coffee table
when he was playing at home. He cried after the event and
received a large cut to his forehead.When you view Jackson he is
not distressed but he does squirm away when attempts are made
to examine his wound. He has a haematoma on the left side of his
forehead
and a full thickness laceration of one to two centimetres over his eye on the same
side.
1 2 3 4 5
Airway breathing and circulation are intact.The child is not
distressed and he did not lose consciousness. He should wait no
longer than 60 minutes for treatment.
31. Adit is a 15-month-old boy with a two-hour onset of fever and
breathing difficulty. He presents via ambulance with an audible
stridor at rest.
1 2 3 4 5
Partially obstructed airway.This child should wait no longer than
10 minutes for treatment.
32. Tahlia is an 18-month-old girl who arrives at the ED with her mother
at midnight. About 24 hours prior she developed a ‘barking cough’
that became ‘much worse at night’. She is also febrile (temperature
is 38.4°C). Since becoming unwell,Tahlia has had two bottles of
water but refuses food and milk. Both mother and child appear very
anxious.
1 2 3 4 5
Airway is clear and the child is ventilating adequately.The child is
anxious but is still able to take oral fluids. She should wait no
longer than 30 minutes for treatment.
33. Kerri, a 31-year-old female, presents to triage with her boyfriend.
She is complaining of a severe headache and has a history of
migraine. She said she saw her GP two days ago for ‘a sore throat’
and was prescribed penicillin, which she is currently taking. Today
she woke up with a headache and started to vomit. She is pale 176
with a washed-out appearance; her skin is cool and moist. Kerri’s
heart rate is 98 beats per minute, respiratory rate 18 breaths per
minute and her Glasgow coma score is 15 out of 15. She rates her
pain as ‘nine out of 10’.
1 2 3 4 5
Airway and breathing are intact. Periphery is pale and cool,
indicating poor perfusion. This patient has severe pain and should
wait no longer than 30 minutes for treatment.
34. Antony, 56 years, was opening a tin of paint stripper with a
knife and some of the chemical splashed up into his right eye.
He ran water from the tap over his eye for fifteen minutes,
before his partner drove him to the ED. At triage he appears
very uncomfortable; the eye is closed and there is blistering to the
skin surrounding the right orbit.
1 2 3 4 5
Airway breathing and circulation are clear.The chemical burn to the
eye with changed visual acuity.This patient should receive treatment
within 10 minutes.
35. Rosemary is a 40-year-old woman who presents to triage
complaining of abdominal pain. She is 36 weeks pregnant (G5P3)
and is normally well. She tells you she has had pain ‘on and off’ for
one week but it has become more severe in the last day.
It is difficult for the patient to be precise about the location, but the
pain seems to be in the right upper quadrant. She says the pain is
‘worse after eating’ and today she has vomited twice. She says this
pain ‘does not feel like labour pains’. Her respiratory
rate is 22 breaths per minute, SpO2 is 98 per cent and heart rate
is 106 beats per minute. Her skin is pale, warm and dry, and her
Glasgow Coma Score is 15 out of 15.Temperature is measured at
37.8°C. Rosemary rates her pain as ‘seven out of ten’. She tells
you that she has had no PV loss.
1 2 3 4 5
Airway, breathing and circulation are intact.The patient has severe
pain and should wait no longer than 30 minutes for treatment.
36. Mary-Jane is a 36-year-old woman who presents to triage via
ambulance following a fall from a ladder. She is 37 weeks
pregnant (G2P1) and is normally well. She
was hanging curtains in the nursery and standing the step second from the top
when
she overbalanced. She complains of a painful right wrist and pain in
her right hip. Her respiratory rate is 20 breaths per minute, SpO2 is
99 per cent, and her heart rate is 110 beats per minute. Her skin is
177
7 pale, warm and dry. Her Glasgow Coma Score is 15 out of 15 and
7 her blood pressure is 120/70. She rates her pain as ‘six out of ten’
and she reports no PV loss.
1 2 3 4 5
Airway and breathing are intact.There is a slight tachycardia and
the patient has moderate pain.The patient should wait no longer
than 30 minutes.
37. Connie is a 74-year-old female who presents to the ED via
ambulance. Apparently she was an in-patient at your hospital five
days ago. At that time she was managed for an acute bowel
obstruction.Today the hospital-in-the-home nurse visited her and
then called an ambulance. According to the ambulance officers, she
has had increasing abdominal pain and vomiting during the night.
Her bowels have not been opened for three days.You note her to be
pale and distressed on the ambulance trolley. She complains that
her abdomen is ‘bloated’. Her blood pressure is 110/75, heart rate
is 112 beats per minute, respiratory rate is 26 breaths per minute,
and temperature is 37.2°C.
1 2 3 4 5
Airway breathing and circulation are intact.There is mild tachypnoea
and tachycardia. Significant pain is experienced due to a possible
recurrence of bowel obstruction.
This patient should wait no longer than 30 minutes.
38. Ted, a 78-year-old male, is brought to the ED via ambulance.The
patient attended the ED last night with a vague story of feeling
dizzy and unwell. He was diagnosed with a viral illness and sent
home.Throughout the night he was woken by heavy chest pains
that ‘came and went’. He took three of his Anginine, which did
not relieve the pain. He is now short of breath and his lips have a
frosted appearance. His heart rate is 92 beats per minute,
respiratory rate 24 breaths per minute and blood pressure
160/90.
1 2 3 4 5
Airway is intact.The patient may be hypoxic as evidenced by
‘frosted lips’. Haemodynamics are stable.There is also chest pain
which may be cardiac in nature. This patient should wait no longer
than 10 minutes.
39. Macey is a 38-year-old female who presents to the ED with an injured right leg.
She is brought to the triage desk in a wheelchair by her father who
tells you she has multiple sclerosis.Today she was found by her
father after falling down four steps at the front of her home.
Normally she is able to walk using a walking stick, but since the fall
she has not been able to walk at all. On examination you note that
her right ankle is swollen and a right pedal pulse is palpable. She
tells you that she has ‘no pain’ at the moment and is happy to wait
to see a doctor.
178
1 2 3 4 5
Airway breathing and circulation are intact.There is an injury to the
ankle and possible fracture.The patient should wait no longer than
60 minutes.
40.A solidly built male of about 40 years of age and smelling strongly
of alcohol starts shouting at another patient in the waiting room. He
says he wants to see a doctor, but before you can establish what is
wrong, he stands up and begins to threaten with a knife the other
patients who are waiting.
1 2 3 4 5
Immediate threat of physical violence using a weapon.
Simultaneous security and clinical response to aggression is
required immediately.
41. Tomas is an eight-year-old boy presents to the ED with his
mother, who had been called to the school to pick him up
today.While playing at recess Tomas was involved in a fight, which
resulted in him being hit in the face with a cricket bat. His mother
says the school called her because the child was inconsolable after
the event and he didn’t want to go back to the classroom.There
was no loss of consciousness reported, but the child has a three
centimetre laceration to his left cheek.
1 2 3 4 5
Airway breathing and circulation are intact.There has been no loss
of consciousness, nonetheless the child is distressed and should
receive treatment within 60 minutes.
42. Harley is an 18-month-old boy who was brought to the ED via ambulance. He
was found face down in a swimming pool. His parents administered
cardiopulmonary resuscitation at the scene and called the
ambulance. On arrival the child is breathing spontaneously and
receiving 100 per cent oxygen via a bag-valve mask. His heart rate
is 140 beats per minute and his respiratory rate is 14 breaths per
minute.
The child’s eyes are closed and he is lying still on the trolley. He
is responding to painful stimuli.
1 2 3 4 5
Near drowning with respiratory arrest. Respiration and
circulation now re-established.The child should
commence treatment immediately.
43. Phillip, 44 years, was bitten by an ant two days ago. The bite site,
which is located on his inner thigh, is red and itchy. There is a 15-
centimetre area of cellulitis surrounding the bite. He has a
temperature of 38.2°C.
1 2 3 4 5
Airway breathing circulation is intact but there is cellulitis with
fever.This patient should commence treatment within 60
minutes.
179 44. Laurie has been referred to the ED from his local doctor on a
Monday morning. He complains of increasing upper abdominal
pains, associated nausea and constipation over weekend. He had a
loose bowel action this morning. His appetite is normal, but his
pain is sharp in nature and he rates it as ‘eight out of ten’.
1 2 3 4 5
Airway breathing circulation is intact.The patient has severe
pain and abdominal symptoms. He should commence
treatment within 30 minutes.
45. Mr Smyth, 77 years of age, is brought to the ED by his daughter.
He is having difficulty passing urine and feels like his bladder ‘is
about to burst’. He tells you that he has had problems with ‘the
prostate’ before.When asked about his pain he says it is ‘about five
out of ten’.You notice that he is unable to sit still because of the
pain and he is sweating quite a bit.
1 2 3 4 5
Airway breathing circulation is intact. Moderate pain due to acute
urinary retention. This patient should wait no longer than 30
minutes to commence treatment.
46. Ned, a 28-year-old jockey, attends the ED after being kicked in
the abdomen by a horse. He was assisting a colleague to guide
the animal into a float when it reared up and kicked him. Ned
was thrown some two metres and fell on the ground. He did not
lose consciousness, but was ‘winded from the kick’. At triage Ned
appears pale and distressed. He tells you that he has pain and
points to his left upper quadrant. His heart rate is 128 beats per
minute, his respiratory rate is 26 breaths per minute and his skin
is cool and moist.
1 2 3 4 5
Airway clear, tachypnoeic and tachycardic. Significant blunt
trauma to the abdomen (possible liver injury) and the patient is
showing signs of shock.This patient should commence treatment
within 10 minutes.
47. Homer, 28 years, twisted his right knee playing basketball.The
knee is very swollen and he is unable to weight-bear on it.The
injury occurred about two hours prior to his arrival in the ED and
an ice pack has been applied.
1 2 3 4 5
Airway breathing and circulation are intact.The patient has pain due
to an acute limb injury. He should wait no more than 60 minutes for
treatment.
180
48. Carmel, 59 years, woke this morning with pain in her left eye. She
then noticed a rash appearing above her brow and has developed
severe pain in the left side of her face and eye. She says there is ‘a
lump’ behind her ear. She has no past medical history but she did
have an episode of flu-like symptoms two days ago. She describes
the pain as ‘hot and sharp’. She rates it ‘eight out of ten’.
1 2 3 4 5
Airway breathing and circulation are intact. Severe pain due to
infection with possible trigeminal nerve distribution.This patient
should wait no longer than 30 minutes.
49. Gillian presents to the ED with generalised abdominal pain. She
has been brought in by a work colleague.When questioned, she
complains of six days of constipation. She is booked in for a
colonoscopy at a private clinic tomorrow. She isn’t on medication
but she is bent over and crying in pain.
1 2 3 4 5
Airway breathing and circulation are intact.Abdominal pain is causing
significant discomfort.This patient should wait no longer than 30
minutes to commence treatment.
50. Martin presents to the triage desk on his own. He tells you he has
pain in his left shoulder after he fell in a driveway. On examination
you note that his left shoulder is very swollen. He has very limited
range of movement, in fact he can not move the shoulder joint
itself, and he rates his pain as ‘seven out of ten’. His left radial
pulse is present, but he has some ‘numbness’ around the shoulder
area. His arm is in a sling and he smells of alcohol. He says the
accident occurred ‘a couple of hours ago’.
Airway breathing and circulation are intact.The patient is in severe
pain and the neurological involvement suggests dislocation +
fracture.The patient should wait no longer than 30 minutes.
51. Beverly is a 57-year-old female who was originally sent from her
local doctor to the outpatients department to make an
appointment to see an orthopaedic surgeon. She was referred for
an investigation of osteoarthritis in her right knee via an
arthroscope.Today, when she presents at the ED, she is in severe
pain and has
difficulty weight–bearing on her right leg. She says that the pain
does settle somewhat at rest.The clerk at outpatients said she
needed to be seen in the ED today because of her pain.An orderly
1 2 3 4 5
181
Airway breathing and circulation are intact. Her pain requires
attention, though it does settle at rest.This patient should
commence treatment within 60 minutes.
escorts her from outpatients to triage in a wheelchair.
52. Zane, 26 years of age, presents with an infected left arm. He has
a recent history of intravenous drug misuse. He tells you that he
has been re-using and sharing needles. His cubical fossa is red and
cellulitic and there are several pus-filled sores on the arm. He is
afebrile. He looks around the waiting room nervously and asks you
how long it will be before he can get to see the doctor, as he ‘has
to be somewhere else in an hour’.
1 2 3 4 5
Airway breathing and circulation are intact. Infection requires
treatment and this should commence within 60 minutes. Re-
assess if there are increasing signs of agitation while waiting as
this may indicate drug withdrawal and re-triage may be
required.
53. Hamish is an 18-year-old male who is sent to the ED from his local
doctor with a sudden onset of right testicular pain. He has a history
of partial testicular torsion two weeks ago and states that pain is
the same as it was then. He is doubled over in pain at the triage
desk.
1 2 3 4 5
Airway breathing and circulation are intact. Severe pain with
possible torsion of testes.The patient should receive treatment
within 10 minutes.
54. Linda is a 35-year-old female with a past history of hepatitis
B.Today she presents with right side abdominal pain.The pain has
been getting worse over the past week and is currently ‘four out of
ten’. She has no vomiting; her skin is pink and warm.
1 2 3 4 5
Airway breathing and circulation are intact. Moderate pain is the
main problem for this patient and she should commence
treatment within 60 minutes.
55. Marion, 76 years, presents to the ED from a nursing home. She
collapsed suddenly just before breakfast that morning.
Ambulance officers attended and found her semi-conscious. Her
blood glucose level was measured at 2.1 mmol and she was
given intravenous dextrose (50 mls of 50 per cent dextrose). She
is now sitting up on the ambulance trolley talking to staff.
1 2 3 4 5
Airway breathing and circulation are now intact. Hypoglycaemia
has been treated. Though, given loss of consciousness this
patient should wait no longer than 30 minutes to commence
treatment.
56. Cassandra, 15 years, was riding her horse in the bush some 60
km away from town when the animal was startled and threw her 182
about three metres. She was wearing
a helmet but it broke in half when her head struck a tree. Her
companions noted an initial loss of consciousness, after which she
was drowsy and vomiting, but she did not appear to have any
injuries elsewhere and she said she had no neck pain when asked.
Cassandra was transferred to your ED in the back of a utility. On
arrival
she has a Glasgow Coma Score of 8 out of 15. Her respiratory rate
is 24 breaths per minute and her heart rate is 62 beats per
minute.
1 2 3 4 5
Airway management and cervical spine precautions are
required as the patient is unconscious and is likely to have a
significant head injury + c-spine injury. With a Glasgow Coma
Scale of 8/15, treatment should commence immediately.
57. Lisa is an 18-year-old female who presents to the ED with her
friends who state that she ingested an unknown quantity of
tablets and drank a bottle of white wine about 40 minutes ago
following a fight with her boyfriend. On further questioning you
establish that the medication she took included 24 paracetamol
tablets. Lisa appears drowsy at triage, is disorientated to place
and time, and she smells strongly of alcohol. Her friends report
that in the past 10 minutes she has been ‘twitchy’.
1 2 3 4 5
Airway breathing and circulation are currently intact, although
‘twitching’ suggests significant toxic ingestion of unknown
substances.This patient should commence treatment within 10
minutes.
58. Iris is a 64-year-old woman who is brought to the ED by her husband in a private
car.
She states that she caught her leg on a garden seat while carrying
the washing in from the clothes line. She was concerned that there
was a fair amount of bleeding occurring and she described the gash
as three centimetres long. She is not distressed.
1 2 3 4 5
Airway breathing and circulation are intact.The laceration needs to
be dressed and observed for further bleeding.This patient should
wait no longer than 60 minutes.
59. Silvia, 66 years, is brought to the ED by her husband. She is
complaining of a sudden onset of nausea and dizziness. She is
normally fit and well and has no relevant history. She has not
vomited and has no headache. Her blood pressure is 130/60, heart
rate 64 beats per minute and her respiratory rate is 22 breaths
per minute. She is afebrile. Her Glasgow Coma Score is 15 out of
15.
1 2 3 4 5
Airway breathing and circulation are intact. Collapse of unknown
now alert and orientated.This woman should wait no longer than
60 minutes.
60. Luke, a 27-year-old wants to travel to India next week. He attends
183 the ED for advice about the sorts of vaccinations he might need.
1 2 3 4 5
This is a non-urgent problem; however the patient still requires
advice about vaccinations given he is travelling overseas in the
next week. He should wait no longer than two hours.
61. Pete, aged 28 years, presents to the ED at 9 pm on a Sunday
night requesting a workers compensation certificate for a day he
had off work in the previous week. He was seen at the hospital five
days ago with a sprained wrist and had been given the certificate
for one day off work at that time. However, he states that he has
lost that certificate. He tells you that he is ‘prepared to wait’ as his
boss has told him to get a new certificate by Monday morning or he
would be ‘in big trouble’. His wrist is no longer painful and he says
he ‘feels fine’.
1 2 3 4 5
This is a non-urgent problem and the patient can wait no longer
than two hours.
62. Larry, 26 years, was in a fight last night. He attends the ED this
morning at 6.30 am with a five centimetre deep laceration to his
left ear. He says the injury was the result of a human bite which
occurred at about 3 am. He smells of alcohol.When asked if he was
knocked out he replies ‘no’. He has no pain elsewhere. His vital
signs are within normal limits and he is orientated to time, place
and person.
1 2 3 4 5
Airway breathing and circulation are intact.There is a risk of
infection and the laceration needs to be closed.The patient
should wait no longer than 60 minutes.
63. Rudolf, 78 years, presents to triage via ambulance. He was at
church, and when he went to stand up during the service,
collapsed to the ground. He did not lose consciousness but did
become very pale and sweaty. Paramedics attended and noted
he was in heart block with a heart rate of 42 beats per minute
and blood
pressure of 80/60.They inserted an intravenous cannula and
administered atropine (600 mcg) with no effect. On arrival to the ED
he is conscious and states that he has no chest pain.
1 2 3 4 5
Airway and breathing intact. Haemodynamic compromise in the
setting of heart block.This patient should wait no longer than 10 184
minutes.
64. Betty is a woman in her 20s. She presents to the triage desk with
her friend, who states that Betty has taken 25 Endep tablets. As
you begin talking to her friend, Betty collapses to the floor and
commences fitting.You summon help and staff arrive to lift her onto
a trolley and take her into the ED.
1 2 3 4 5
Collapse with seizure due to toxic effects of drugs.This patient
should receive simultaneous assessment and treatment.
65. Barry, a 43-year-old man, was using an angle-grinder today and
now has a foreign body in his left eye.The eye is red and painful.
He states that the pain is ‘seven out of ten’.
1 2 3 4 5
Airway breathing circulation intact. Foreign body in eye with severe
pain.This patient should receive treatment within 30 minutes.
66. Mario, a 67-year-old man, was putting some pesticide on his
vegetable patch and he accidentally spilt it on his clothing. He
had a shower at home immediately after
the accident but has come to the ED some two hours later because
he is nauseous, vomiting and has developed excessive sweating. His
heart rate is 122 beats per minute and his respiratory rate 28
breaths per minute. He says he is not sure of the name of the
chemical he was exposed to as he has had it in his shed for ‘many
years’.
1 2 3 4 5
Airway intact, tachypnoeic with tachycardia and excessive
sweating, nausea and vomiting, possibly due to toxic exposure to
organophosphate.This patient should wait no more than 10
minutes.
67. Mr F is a 66-year-old man who was brought to the triage desk by
his daughter. He states that he is confused and thinks that people
are talking about him. He tells you that he has a history of ‘heart
failure, high blood pressure, renal failure, urinary tract infection
and depression’. His skin is warm and moist, his respiratory rate is
20 breaths per minute, and his Glasgow Coma Score is 15 out of
15.
1 2 3 4 5
Airway breathing and circulation intact. Normal level of
185 consciousness. Paranoid thoughts with no immediate risk of harm
to self or to others.This patient should wait no longer than 60
minutes. His daughter should wait with him in the waiting room.
68. Hugh is a 54-year-old male who was seen in the ED with a
fractured right radius and ulna four days prior. He presents again
because he says the cast is too loose and needs to be replaced. He
has no pain.
1 2 3 4 5
This is a non–urgent problem. However, the plaster needs to be
assessed as it was applied in the ED and if it is loose will not
effectively immobilise the fracture.
69. Sue, a 36-year-old female, presents with a two-day history of
feeling generally unwell. She has an ache in her lower abdomen
and describes having to go to the toilet more frequently than
normal. On further questioning she states that she has had urinary
frequency for 12 hours, and rates her pain as ‘four out of ten’. She
has a heart rate of 98 beats per minute and a temperature of
37.8°C. She appears to be quite pale.
1 2 3 4 5
Airway breathing and circulation intact. Acute urinary symptoms
and discomfort with mild-moderate pain.This patient should wait no
longer than 60 minutes.
70. Joanne is a 34-year-old female who walks to the triage desk at
10.50 am.When you ask her what is wrong she says ‘I can’t go to
the toilet and my backside is painful’. When questioned further
she says that she has not passed urine today but ‘did last night
and it was not painful’. She rates her current pain as ‘four out of
10’.
1 2 3 4 5
Airway breathing and circulation intact. Acute urinary symptoms
and discomfort with mild-moderate pain.The history is a little
unclear, however this patient should wait no longer than 60
minutes.
71. Mrs W is assisted to the triage desk by her daughter around
midday. Mrs W doesn’t speak very good English so her daughter
tells you her history. Last night Mrs W had an episode of
palpitations and complained of nausea and feeling lethargic.Today
‘the palpitations are back’. She has a history of coronary artery
bypass grafts.When asked if she has chest pain, Mrs W says she is
‘very sick’. Her heart rate is 108 beats per minute and her skin is
cool and moist to touch.
1 2 3 4 5
Airway intact but has palpitations/tachycardia with possible chest 186
pains. History suggestive of cardiac event with some signs of
increase sympathetic activity (pallor and diaphoresis).This patient
should wait no longer than 10 minutes.
72. Maree is a 32-year-old woman who presents via ambulance
complaining of ‘palpitations’. She is 30 weeks pregnant (G3P1) and
is normally well. She was doing the vacuuming when her
palpitations started. She complains of mild chest pain that
is dull in nature and a mild shortness of breath.’ Her respiratory rate
is 24 breaths per minute; SpO2 is 98 per cent; and heart rate is162
beats per minute. Her skin is pale, cool and dry; blood pressure is
90/R; and Glasgow Coma Score is 15 out of 15. Her temperature is
36.3°C.
1 2 3 4 5
Airway intact, mild tachypnoea and haemodynamic
compromise.This patient should wait no longer than 10 minutes.
73. Kerry is a 36-year-old woman who presents to the ED with her
husband via ambulance with a sudden onset of a headache. She
tells you that she is 31 weeks pregnant (G3P1) and has been
‘keeping well’. Her husband tells you that Kerry was making
lunch when she suddenly complained of a severe occipital
headache.
Her respiratory rate is 20 breaths per minute, SpO2 is 98 per cent, and heart rate
is 124 beats per minute. Her skin is pale, cool and dry.The ambulance officers
report
that Kerry’s blood pressure is 160/100 and she has a Glasgow
Coma Score of 14 out of 15 (eyes open to voice). Her
temperature is 36.3°C.When asked to score her level of pain,
she tells you it is ‘nine out of 10’.
1 2 3 4 5
Airway and breathing intact; hypertensive in the context of
pregnancy with sudden severe onset of headache and altered
conscious state.This patient should wait no longer than 10
minutes.
74. Tricia, an 18-year-old female, is brought into the ED by a friend.
Her friend states that she has had vaginal bleeding since her ‘Depo
injection 15 days ago’. Her friend states that Tricia is suicidal and
wants to find ‘peace’. Her friend also tells you that Tricia took a
187 1 2 3 4 5
Airway breathing and circulation intact. Suicidal ideation.This
patient should be under close observation and be treated within
30 minutes. Her friend should be encouraged to sit with her for
support.The patient may need to be re-triaged if she attempts to
leave without being seen.The priority is the physical assessment in
respect to drug toxicity, as there is the additional risk of
absconding which requires close monitoring.
large quantity of herbal sedative last night and now feels ‘weak and
tired’.
75. Josie, 39, walks to the triage desk and complains of pain in her
legs, stating;‘My feet and legs are swollen and sore’ She has a
history of intravenous drug use and heavy alcohol intake and she
has hepatitis C. Currently Josie is not on any medication and is
alert and orientated.
1 2 3 4 5
Airway breathing and circulation are intact.The patient has pain
and no history of injury. She has significant co-morbid factors and
should be seen within 60 minutes.
76. Jake is 28 years old. He attends the ED with his partner at 5.30
pm. He has abdominal pain radiating to his right loin, urinary
frequency and dysuria. He saw his GP yesterday for the pain and
was told he ‘might have kidney stones’.The pain is worse now than
yesterday (‘seven out of 10’) and he has noticed some blood in his
urine the last time he voided.
1 2 3 4 5
Airway breathing and circulation are intact. Pain is severe and
the patient should receive treatment within 30 minutes.
77. Isaac is an 85-year-old male who presents to triage with his son.
He has left loin pain and has recently undergone a lithotripsy for
renal calculi.Today he has had pain for one-and-a-half hours which
‘comes and goes’; the pain is now ‘eight out of ten’.
1 2 3 4 5
Airway breathing and circulation are intact. Pain is severe.The
patient should receive treatment within 30 minutes.
78. Jess, 14 years, is brought to the ED by her mother. She is
complaining of severe period pains and is doubled over in a
wheelchair crying. Her mother tells you that Jess has not been
able to go to school for the past week because of her menstrual
problems and wants a referral to a specialist to ‘sort out the
problem’.When you talk to Jess you establish that the blood loss
is moderate and the pain is in her abdomen thighs and back. She
seems to calm down after you speak to her and appears more
comfortable when you wrap a blanket around her.
1 2 3 4 5
Airway and breathing are intact. Blood loss is within normal
limits. Discomfort alleviated by local measures.This patient
should wait no longer than 60 minutes.
79. A 5-year-old boy is rushed into your ED by his parents on a hot 188
summer day. He has been holidaying with his family in Far North
Queensland and was wading in the sea. He has a raised red welt on
his right leg and is crying in severe pain, He has a heart rate of 128
beats per minute and a blood pressure of 130/70.
1 2 3 4 5
Possible marine envenomation. Rapid heart rate and elevated
blood pressure associated with pain indicate that treatment should
commence within 10 minutes.
80. Reese, 31 years, suffers from migranes.Today she came to the ED
with her sister. She has had an eight-hour history of global
headache, vomiting and visual disturbance. She has taken her
usual medication (Imigran), but says it is ‘not working’. Her heart
rate is 96 beats per minute, respiratory rate 28 breaths per minute.
She is afebrile and rates her pain seven out of ten..
1 2 3 4 5
Airway, breathing, and circulation are intact.The patient is
experiencing severe pain and should wait no longer than 30
minutes for treatment.
81. India is a nine-year-old girl who arrives to the ED via a taxi
accompanied by her mother. She fell while playing netball, injuring
her right foot. She is transferred to the triage desk in a wheelchair
as it is painful for her to weight bear.
1 2 3 4 5
Airway, breathing and circulation are intact. Moderate pain will
require investigation and treatment should commence within 60
minutes.
82. Terry is a 53-year-old male who presents to the ED asking for a
review of his blood pressure medication. He describes having had a
‘headache’ during the past week. It is two years since he saw a
doctor about his medication. His Glasgow Coma Score is 15 out of
15 and his heart rate is 70 beats per minute; he has no nausea or
vomiting and is currently pain free.
1 2 3 4 5
Airway, breathing, and circulation are intact.This condition does
not currently warrant urgent treatment.The patient is able to wait
two hours to see a doctor, but vital signs and pain level should
definitely be re-assessed if he is not seen within this time.
189 83. A mother presents with her six-month-old baby who she says
won’t wake up. The child is breathing, but is floppy, can not be
roused and has pin-point pupils.
1 2 3 4 5
Airway is unstable and cardiopulmonary arrest imminent.The
child requires immediate simultaneous assessment treatment.
84. Paddy is a 32-year-old male who presents to triage stating that he
has vomited blood twice in the last six hours. He states that he has
had dark bowel motions for the last three days and he normally
drinks ‘12 stubbies of beer per day’. Paddy’s skin is pale, warm and
dry. His heart rate is 108 and his respiratory rate is 20 breaths per
minute. He doesn’t have any pain but does complain of nausea.
1 2 3 4 5
While airway, breathing and circulation are currently within normal
parameters, this patient is at significant risk of a sudden and large
gastrointestinal blood loss. He should not wait longer than 10
minutes to commence treatment.
85. Amber is a 22-year-old woman who presents to the ED at 11 pm
complaining of a 24-hour history of a sore throat and is feeling
generally unwell. She had been attending a party nearby and
decided to call into the hospital to get some antibiotics. She has no
other symptoms, looks well and is afebrile.
1 2 3 4 5
This is a non-urgent problem and the patient can wait two hours to
see a doctor.
86. You are called to assist a young woman getting her boyfriend
out of the car that is pulled up in the ambulance bay. She tells
you that Matt ‘shot up’ 30 minutes ago.
On examination Matt appears to have vomited and is centrally
cyanosed. He has irregular grunting respirations of 6 breaths per
minute and his heart rate is 42 beats per minute.
1 2 3 4 5
Grunting respirations and central cyanosis indicate that this
patient has an airway obstruction and requires immediate
treatment.
87. Elliot is 27 years old. He injured his back yesterday lifting a
heavy box at work. He had been managing the pain at home,
however today it is ‘much worse’. He was unable to get an
appointment with his local doctor so he has come to the ED. He
rates his pain ‘five out of ten’, and has taken two Panadeine Forte
and two Nurofen tablets in the past hour.
1 2 3 4 5
Airway, breathing and circulation are intact. Adequate analgesia
has been administered prior to arrival.This patient should wait no
longer than 60 minutes to see a doctor.
190
88. Ambulance officers arrive without prior notice with a female aged
26. She was a front-seat passenger in a single motor vehicle crash
that involved multiple rollovers.
The ambulance officers state that the patient was walking around
intoxicated at the scene and was abusive, complaining of
abdominal pain and reluctant to come to hospital. On examination
the patient is centrally cyanosed and not breathing.
1 2 3 4 5
Respiratory arrest immediate simultaneous assessment and
treatment is required.
89. Ron, the 50-year-old coach of a visiting interstate football team,
presents to triage at 7 pm on Saturday night. His anti-
hypertensive medications have run out and
his GP had warned him that it would be dangerous for him to stop
his medications. The man says that he realises that it is ‘not
completely appropriate’ for him to attend the ED for a prescription,
but says he doesn’t know any GPs in the city and is quite prepared
to wait for a prescription. His Glasgow Coma Score is 15 out of 15
and his skin is pink, warm and dry. He has no headache or pain
elsewhere.
1 2 3 4 5
This is a non-urgent presentation and this patient can wait up to two
hours to see a doctor.
90. Noel, 29 years, is driven to the ED by friends following a fight at
his cousin’s party. You are called to retrieve Noel from the
ambulance bay.While getting Noel out of the car, you learn that he
was stabbed in the left side of his chest with a carving knife and
see a two centimetre laceration below his left nipple. His skin is
cool, pale and moist. He has a weak carotid pulse and a Glasgow
Coma Score of 9 out of 15.
1 2 3 4 5
Cardiopulmonary arrest is imminent. Immediate assessment and
resuscitation is required.
91. Brett is 27. He presents to triage via a private car following a
fall from scaffolding at a construction site approximately 20
minutes prior to presentation. Brett fell more than 10 feet onto a
concrete slab. He was observed by his work mates to
be unresponsive for ‘about five minutes’ and then he regained
consciousness, but he has been drowsy. He has vomited four times
and has a large boggy haematoma on his occiput. Brett is
complaining of a generalised headache. His Glasgow Coma Score is
13 out of 15, heart rate is 74 beats per minute, and respiratory
rate is 14 breaths per minute.
1 2 3 4 5
191 Airway, breathing and circulation are intact.The mechanism of
injury and history of loss of consciousness for several minutes
indicate that this patient should be seen within 10 minutes.
92. An obviously pregnant woman presents to triage stating that she is in labour
and that she thinks there is something hanging down between her
legs. On cursory examination you see under her dress what
appears to be an umbilical cord.
1 2 3 4 5
Birth is imminent.The patient should receive immediate
simultaneous assessment and treatment.
APPENDIX F: ABBREVIATIONS
AAA Abdominal aortic aneurysm
AAEN Australian Association of Emergency
Nurses ACEM Australasian College of
Emergency Medicine ATS Australasian
Triage Scale
BP Blood pressure
BSL Blood sugar level
CTAS The Canadian Triage and Acuity Scale
DRGs Diagnosis related groups
ED Emergency Department
ENA Emergency Nurses’ Association
ESI Emergency Severity Index
GCS Glasgow Coma Scale
HR Heart rate 192
ICD-10 International Classification of Diseases – version 10
ICU Intensive Care Unit
ITS Ipswich Triage Scale
K Kappa
LOC Loss of consciousness
O/A On arrival
MTS Manchester Triage System
NTS National Triage Scale
SAO2 Oxygen saturation
SOB Shortness of Breath
PE Pulmonary embolism
RR Respiratory rate
APPENDIX G: GLOSSARY
access block of emergency medicine and certified by the
The situation where patients in the Australasian College of Emergency Medicine
ED requiring inpatient care are with the qualification of Fellow of the
unable to gain access to appropriate Australasian College of Emergency Medicine
hospital beds within a reasonable (FACEM).
timeframe.
admission delay time
The difference between the ready-for-
departure time and the actual
departure time for patients who are
admitted to hospital, die in the ED,
or are transferred to another hospital
for admission.
arrival time
The first recorded time of contact
between the patient and the ED staff.
assessment and treatment time
The difference between the time
of initial medical assessment and
treatment and the ready-for-
departure time.
193
departure time
The time the patient physically leaves
the ED.
emergency department (ED)
The dedicated area in a hospital that is
organised and administered to provide
a high standard of emergency care to
those in the community who are in
need of acute or urgent care.
ED overcrowding
The situation where ED function is
impeded because the number of
patients waiting to be seen,
undergoing assessment and
treatment, or waiting for departure
exceeds either the physical or staffing
capacity of the ED.
emergency medicine
A field of practice based on the
knowledge and skills required for the
prevention, diagnosis and
management of acute and urgent
aspects of illness and injury affecting
patients of all age groups.
emergency physician
A registered medical practitioner who
is trained and qualified in the specialty
re-triage wait for an emergency assessment.
Clinical status is a dynamic state
for all patients. If clinical status waiting time
changes in a way that will The difference between arrival time
impact upon the triage category, and time of initial medical
or if additional information assessment and treatment. A
becomes available that will recording accuracy to within the
influence urgency, then re-triage nearest minute is appropriate.
must occur.
When a patient is re-triaged,
the initial triage code and any
subsequent triage code must
be documented.The reason for
re-triaging must also be
documented.
time of medical assessment
& treatment The time at which
the patient is initially reviewed
and assessed by the medical
officer; represents the start of
the care for which the patient
presented.
total ED time
The difference between the
arrival time and the departure
time.
triage
The basic process whereby all
incoming patients are categorised
into numerical groups dependant
upon their urgency rating scale.
Triage Nurse
A qualified and experienced
registered nurse who
demonstrates and maintains
clinical expertise in emergency
nursing prior to commencing the
triage role. A Triage Nurse
undertakes patient assessment
and allocates the ATS category
in an ED.
triage system
The process by which a
clinician applies a patient’s
clinical urgency.
urgency
Determined according to the
patient’s clinical condition and is
used to ‘determine the speed of
intervention that is necessary to
achieve an optimal outcome’.
Urgency is independent of the
severity or complexity of an
illness or injury. Patients may be
triaged to a lower urgency rating
because it is safe for them to
Emergency Medicine Journal 2004;21:681–
REFERENCES 4.
10.Gilboy N,Travers D,Wuerz R. Re-evaluating
1. Australasian College for Emergency triage in the new millennieum: a
Medicine. Policy on the Australasian Triage comprehensive look at the need for
Scale. ACEM [Online] 2006 [cited 2007 Feb standardisation and quality. Journal of
2]. Available from: Emergency Nursing 1999;25:468–73.
URL:
http://www.acem.org.au/media/policies_ 11.Australasian College for Emergency
and_ guidelines/P06_Aust_Triage_Scale_- Medicine. National Triage Scale.
_Nov_2000.pdf Emergency Medicine 1994;6:145–6.
2. Australasian College for Emergency
Medicine. Guidelines for
implementation of the Australasian
Triage Scale in Emergency
Departments. ACEM [Online] 2005
[cited 2007 Feb 2]. Available from: URL:
http://www.acem.org.au/media/policies_a
nd_ guidelines/G24_Implementation
ATS.pdf
3. Zimmermann PG.The case for a
universal, valid, reliable 5-tier triage
acuity scale for US emergency
departments. Journal of Emergency
Nursing 2001;27(3):246–54.
4. FitzGerald G.Triage. In: Cameron P, Jelinek
G, Kelly AM, Murray L, Heyworth J,
editors.Textbook of Adult Emergency
Medicine. Edinburgh: Churchill Livingstone;
2000. p. 584–8.
5. Richardson D.Triage. In: Cameron P,
Jelinek G, Kelly AM, Murray L, Brown AFT,
Heyworth J, editors. Textbook of Adult
Emergency Medicine. 2nd edn. Sydney:
Churchill Livingstone; 2004. p. 702–5.
6. Charles A.The challenge of triage.
Coronial Communiqué 2003 Dec [cited
2007 Jan 2]; 1(1): 3. Available from:
URL: http://www.vifm.org/cgi-
bin/getObject. cgi?id=o249
7. Kennedy K, Aghababian RV, Gana L,
Lewis PC.Triage: Techniques and
Applications in Decision Making. Annals
of Emergency Medicine 1996;28(2):136–
44.
8. Schom A. Napoleon Bonaparte. New
York: Harper Collins; 1997.
9. Taylor DM, Bennett DM, Cameron PA. A
paradigm shift in the nature of care
provision in emergency departments.
five-level triage instrument. Academic
Emergency Medicine 2000;7(3):236–
12. Beveridge R, Clarke B, Janes L, 42.
Savage N,Thompson J, Dodd G, et al.
Implementation Guidelines for The 22. Wuerz, R,Travers DA, Gilboy N, Eitel N,
Canadian Emergency Department Rosenau A,Yazahari R. Implementation
Triage and Acuity and refinement of the Emergency
Scale (CTAS). Clinical Journal of Severity Index. Academic Emergency
Emergency Medicine 1999;1(3 suppl). Medicine 2001;8:61–4.
13. Beveridge R. CAEP issues.The 23. Tanabe P, Gimbel R,Yarnold PR, Adams
Canadian Triage and Acuity Scale: a JG.The Emergency Severity Index
new and critical element in health care (version 3) 5-level triage system scores
reform. Canadian Association of predict ED resource consumption.
Emergency Physicians. Journal of Journal of Emergency Nursing
Emergency Medicine. 1998;16(3):507– 2004;30(1):22–9.
11.
14. Beveridge R, Clarke B, Janes L,
Savage N,Thompson J, Dodd G, et al.
Canadian Emergency Department
Triage and Acuity Scale;
Implementation Guidelines. Canadian
Journal of Emergency Medicine 1999;
1(3 (special supplement)): s3–s23.
15. Beveridge R, Ducharme J, Janes l, Beaulieu
S,Walter
S. Reliability of the Canadian
Emergency Department Triage and
Acuity Scale: Interrater Agreement.
Annals of Emergency Medicine
1999;34(2):155–9.
16. Considine J, Ung L,Thomas S.Triage
nurses’ decisions using the National
Triage Scale for Australian emergency 194
departments. Accident and Emergency
Nursing 2000;8(4):201–9.
17. FitzGerald GJ. Emergency
Department Triage. Doctoral
Dissertation. Brisbane:
University of Queensland; 1989.
18. FitzGerald G.The National Triage
Scale. Emergency Medicine
1996;8:205–6.
19. Hollis G, Sprivulis P. Reliability of the
National Triage Scale with Changes in
Emergency Department Acuity Level.
Emergency Medicine 1996;8(4):131–5.
20. Jelinek GA, Little M. Inter-rater
reliability of the National Triage
Scale over 11,500 simulated cases.
Emergency Medicine 1996;8:226–
30.
21. Wuerz R, Milne LW, Eitel DR,Travers D, Gilboy
N. Reliability and validity of a new
24. Fernandes C,Tanabe P, Gilboy N, Johnson 34. Doherty S. Application of the National Triage
L, McNair. R, Rosenau M, et al. Five-Level Scale
Triage. A report from the ACEP/ENA Five- is not uniform. Australian Emergency Nursing
level Triage Task Force. Journal of Journal 1996;1(1):26.
Emergency Nursing 2005;31(1):39–50.
35. Considine J, Le Vasseur SA, Charles A.
25. Australian Council on Healthcare Consistency of Triage in Victoria’s Emergency
Standards. Clinical Indicators – A Users’ Departments. Education and Quality Report.
Manual. Zetland NSW; 1996. Melbourne: Monash Insitute of Health
Research. Report to the Victorian Department
26. Wuerz R, Milne LW, Eitel DR,Wiencek J, of Health and Community Services, July 2001.
Simonds
W. Outcomes are predicted by a new five
level triage algorithm. Academic
Emergency Medicine 1999;6:389.
27. Considine J, Le Vasseur SA, Charles A.
Consistency of Triage in Victoria’s
Emergency Departments:
Literature Review. Monash Institute of
Health Services Research. Report to the
Victorian Department of Human Services,
2001.
28. Gerdtz M, Bucknall T. Australian Triage
Nurses’ decision-making and scope of
practice. Australian Journal of
Advanced Nursing 2000;18(1).
29. Gerdtz M, Bucknall T.Triage nurses’
clinical decision-making: An
observational study of urgency
195 assessment. Journal of Advanced
Nursing 2001;35(4):550–61.
30. Tchernomoroff R, Knight K.Telephone
Triage Program. Bendigo: Bendigo Health
Care Group; 2002.
31. Whitby S, Leraci S, Johnson D, Mohsin
M. Analysis of the Process of Triage:The
Use and Outcome of
the National Triage Scale. Report to
Commonwealth department of Health and
Family Services. Liverpool, NSW: Liverpool
Health Service; August 1997.
32. McNair R. It takes more than string to
fly a kite: 5-level acuity scales are
effective, but education,
clinical expertise and compassion are
still essential. Journal of Emergency
Nursing 2005;31(6):600–3.
33. Dilley S, Standen P. Victorian Nurses
Demonstrate Concordance in the
Application of the National Triage Scale.
Emergency Medicine 1998;10:12–18.
36. Considine J, Botti M.Who, when and where?
Identification of patients at risk of an 46. Tobin M, Chen L, Scott E. Development
in-hospital adverse event: and implementation of mental health
implications for nursing practice. triage guidelines for emergency
International Journal of Nursing departments. South Eastern Sydney
Practice 2004;10(1):21–31. Area Mental Health; 1999.
37. Crispin C, Daffurn K. Nurses’ 47. Broadbent M, Jarmen H, Berk M.
responses to acute severe illness. Improving competence in emergency
Australian Critical Care mental health triage. Accident and
1998;1(4):131–3. Emergency Nursing 2000;10:155–62.
38. Tippins E. How emergency 48. Broadbent M.The Mental Health Triage
department nurses identify and Project Final Report. Unpublished.Victoria:
respond to critical illness. Barwon Health Mental Health Services;
Emergency Nurse 2005;13(3):24– 2001.
32.
39. Schein R, Hazday N, Pena M, et al.
Clinical antecedents to in-hospital
cardiopulmonary arrest. Chest
1990;98(6):1388–92.
40. Bedell SE, Deitz D, et al. Incidence
and characteristics of preventable
iatrogenic cardiac arrests. JAMA 2
1991;65(21):2815–20.
41. McQuillan P, Pilkington S, et al.
Confidential inquiry into quality of care
before admission to intensive care.
British Medical Journal
1998;316(7148):1853–8.
42. Sutherland Hospital. Mental health triage
guidelines
– a Sutherland Hospital
collaborative approach to quality
patient care. Unpublished paper;
1998.
43. Smart D, Pollard C,Walpole B. Mental
health triage in emergency medicine.
Australian and New Zealand Journal of
Psychiatry 1999;33:57–66.
44. Happell B, Summers M, Pinikahana J.
Measuring the effectiveness of the
National Mental Health Triage Scale in
an emergency department.
International Journal of Mental Health
Nursing 2003;12(4): 288–92.
45. New South Wales Health
Department, Centre for Mental
Health,Working Group for Mental
Health Care in Emergency
Departments. Mental
Health Care in Emergency Departments.
Final Report and Recommendations;
1998.
49.Broadbent M, Jarman H, Berk M.
Emergency department mental health 59.Loveridge N. Ethical implications of
triage scales improve outcomes. Journal achieving pain management. Emergency
of Evaluation in Clinical Practice Nurse 2000;8(3):16–21.
2004;10(1):57–62.
60.Trautman DE. Pain Management. In:
50.National Institute of Clinical Newberry L, ed. Sheehy’s emergency nursing
Studies.Victorian emergency principles and practice. 5th edn. New York:
department mental health triage Mosby; 2003. p. 156–68.
tool. NICS [Online] 2006 [cited 2007
Feb 2]. Available from:
URL:
http://www.health.vic.gov.au/emergency/
mhtriagetool.pdf
51.Boyce P, Carter G, Penrose-Wall
J,Wilhelm, Goldney R. Summary
Australian and New Zealand clinical
practice guidelines for the management
of adult deliberate self harm. Australasian
Psychiatry 2003;11(2).
52.Mullen P.The dangerousness of the
mentally ill and the clinical assessment
of risk. In: Brookbanks W, ed. Psychiatry
and the law.Wellington: Brooker; 1996.
53.New Zealand Ministry of Health.
Guidelines for clinical risk assessment
and management in mental health
services.Wellington: Ministry of Health
and Funding Authority; 1998.
54.Hillard R, Zitek R. Emergency
Psychiatry. New York: McGraw-Hill; 2004.
55.De Guio A.Training manual for non-
mental health trained staff to work with
mental health patients in hospital
emergency departments. South Eastern
Sydney Area Health Service, 1999.
56.Happell B, Summers M, Pinikahana
J.The triage of psychiatric patients in
the emergency department:
A comparison between emergency
department nurses and psychiatric nurse
consultants. Accident and Emergency
Nursing 2003;10(2):65–71.
57.Lee J. Pain measurement: understanding
existing tools and their application in the
emergency department. Emergency
Medicine Australia 2001;13(3):279–87.
58.Rupp T, Delaney KA. Inadequate
analgesia in emergency medicine. Annals
of Emergency Medicine 2004;43(4):494–
503.
61. Fatovich DM, Brown A, Pain Relief in 2004;12(3):136–40.
Emergency Medicine. In: Cameron P,
Jelinek G, Kelly AM, Murray L, Brown, A, 71. Nelson B, Cohen D, Lander O, Crawford
Heyworth J, eds.Textbook of Adult N,Viccellio A, Singer A. Mandated pain
Emergency Medicine. Sydney: Churchill scales improve frequency of ED analgesic
Livingstone; 2000. p. 533–34. administration. American Journal of
Emergency Medicine 2004;22(7):582–5.
62. Wong DL, Hockenberry-Eaton
M,Wilson D, Winkelstein, ML, 72. Puntillo K, Neighbour M, O’Neil N, Nixon
Schwartz P.Wong’s Essentials of R. Accuracy of emergency nurses in
Paediatric Nursing. 6th edn. St assessment of patients’ pain. Pain
Louis: Mosby; 2001. p. 1301. Management in Nursing 2003; 4(4):171–5.
63. Australian and New Zealand College
of Anaesthetists. Acute pain in
emergency departments; Managing
acute pain in opioid tolerant patients.
In: Acute pain management: Scientific
evidence. 2nd edn. Melbourne; 2005.
p. 178–82, 258–9.
64. Lyon F.The convergent validity of the
Manchester pain scale. Emergency
Nurse 2005; 13(1): 34–8.
65. Abbey J, Piller A, De Bellis A,
Easterman A, Paker, D, Giles L, et
al.The Abbey pain scale: a 1-
minute numerical indicator for
people with end-stage
dementia. International Journal of
Palliative Nursing 2004;10(1):6, 8–
13. 196
66. The Royal College for General
Practitioners. Medical care of older
persons in residential aged care
facilities. 4th ed. RCGP [Online] 2006
[viewed Feb 2 2007]. Available from:
URL: http://www.racgp.org.au/silverbookonline/4-
6.asp
67. Hall J. Paediatric pain assessment.
Emergency Nurse 2002;10(6):31–3.
68. Boyd RJ, Stuart P.The efficacy of
structured assessment and analgesia
provision in the paediatric emergency
department. Emergency Medicine
Journal 2005;22(1):30–2.
69. Dann E, Jackson R, Mackway-Jones K.
Appropriate categorisation of mild pain
at triage: a diagnostic study.
Emergency Nurse 2005;13(1):28–32.
70. Fry M, Ryan J, Alexander N. A
prospective study of nurse initiated
panadeine forte: expanding pain
management in the ED. Accident
and Emergency Nursing
73. Todd K. Influence of ethnicity on observation variables in assessing febrile
emergency department pain children. Pediatrics 1980;65(6):1090–5.
management. Emergency Medicine
Australia 2001;13(3):274–8. 84. McCarthy PL, Jekel JF, Stashwick CA, Spiesel
SZ, Dolan TF, Sharpe MR, et al. Further
74. Seguin D. A nurse-initiated pain definition of history and observation
management advanced triage protocol for variables in assessing febrile children.
ED patients with an extremity injury at a Pediatrics 1981;67(5):687–93.
level I trauma centre. Journal of
Emergency Nursing 2004;30(4):330–5.
75. Teanby S. A literature review into pain
assessment at triage in accident and
emergency departments. Accident and
Emergency Nursing 2003;11(1):12–17.
76. Durojaiye L, O’Meara M. A study of
triage of paediatric patients in
Australia. Emerg Medicine
2002;14(1):67–96.
77. Crellin DJ, Johnston L. Poor agreement in
application of the Australasian Triage Scale
to paediatric emergency department
presentations. Contemporary Nurse
2003;15(1-2):48–60.
78. George S, Read S,Westlake L, Fraser-
Moodie A, Pritty P, Williams B. Differences
in priorities assigned to patients by Triage
197 Nurses and by consultant physicians in
accident and emergency departments.
Journal of Epidemiology & Community
Health 1993;47(4):312–5.
79. Davis J. Children in Accident and
Emergency: parental perceptions of the
quality of care. Part 1. Accident and
Emergency Nursing 1995;3(1):14–18.
80. Selekman J, Malloy E. Difficulties in
symptom recognition in infants. Journal of
Pediatric Nursing: Nursing Care of Children
and Families 1995;10(2):89–92.
81. Cole TJ, Gilbert RE, Fleming PJ, Morley CJ,
Rudd PT, Berry PJ. Baby Check and the Avon
infant mortality study. Archives of Disease in
Childhood 1991;66(9):1077–8.
82. Hewson PH, Humphries SM, Roberton
DM, McNamara JM, Robinson MJ.
Markers of serious illness in infants
under 6 months old presenting to
a children’s hospital. Archives of Disease in
Childhood 1990;65:750–6.
83. McCarthy PL, Jekel JF, Stashwick CA,
Spiesel SZ, Dolan TF, Jr. History and
85. McCarthy PL, Lembo RM, Baron MA, 24 2007]. Available from:
Fink HD, Cicchetti DV. Predictive URL: http://www.fpnotebook.com/ID468.htm
value of abnormal physical
examination findings in ill-appearing 95. Hewson P, Poulakis Z, Jarman F, Kerr J, McMaster
and D, Goodge J, et al. Clinical markers of
well-appearing febrile children. serious illness in young infants: a
Pediatrics 1985;76(2):167–71. multicentre follow-up study. Journal of
Paediatrics & Child Health 2000;36(3):221–
86. McCarthy PL, Lembo RM, Fink HD, 5.
Baron MA, Cicchetti DV. Observation,
history, and physical examination in
diagnosis of serious illnesses in febrile
children less than or equal to 24
months. Journal of Pediatrics
1987;110(1):26–30.
87. Stanton AN, Downham MA,
Oakley JR.Terminal symptoms in
children dying suddenly and
unexpectantly at home. British
Medical Journal 1978; 2:1249–51.
88. Thornton AJ, Morley CJ, Cole TJ,
Green SJ,Walker KA, Rennie JM. Field
trials of the Baby Check score card in
hospital. Archives of Disease in
Childhood 1991;66(1):115–20.
89. Waskerwitz S, Berkelhamer JE.
Outpatient bacteremia: Clinical findings
in children under two years with intial
temperatures of 39.5oC or higher.
Journal of Pediatrics 1981;99(2):231–3.
90. Hewson P, Humphries S, Roberton D,
McNamara J, Robinson M. Markers of
serious illness in infants under 6 months
old presenting to a children’s hospital.
Archives of Disease in Childhood
1990;65:750–6.
91. Browne GJ, Gaudry PL, Lam L. A
triage observation scale improves the
reliability of the National Triage Scale.
Emergency Medicine 1997;9:283–8.
92. Wiebe R, Rosen L.Triage in the
emergency department. Emergency
Medicine Clinics of North America
1991;9(3):491–505.
93. McCarthy P, Sharpe M, Spiesel S,
Dolan T, Forsyth B, DeWitt T, et al.
Observation scales to identify serious
illness in febrile children. Paediatrics
1982;70(5):802–9.
94. McCarthy P, Sharpe M, Spiesel S,
Dolan T, Forsyth B, DeWitt T, et al.Yale
Observation Scale. In: Family Practice
Note Book [Online] 1982 [cited March
96. Gorelick MH, Stevens MW, Schultz computed tomography scans in
TR, Scribano paediatric head injury. Journal of
PV. Performance of a novel clinical score, Paediatrics & Child Health
the Pediatric Asthma Severity Score 2002;38(4):388–92.
(PASS), in the evaluation
of acute asthma. Academic Emergency 106. Gruskin KD, Schutzman SA.
Medicine 2004;11(1):10–18. Head trauma in children younger than
2 years: are there predictors for
97. Asthma Strategy Group. complications? Archives of Pediatrics
Asthma Best Practice Guidelines. & Adolescent Medicine
Melbourne: Royal Children’s 1999;153(1):15–20.
Hospital; 2001.
98. Lodha R, Bhadauria PS, Kuttikat
AV, Puranik M, Gupta S, Pandey RM, et al.
Can clinical symptoms or signs accurately
predict hypoxemia in children with acute
lower respiratory tract infections? Indian
Pediatrics 2004;41(2):129–35.
99. Mower WR, Sachs C, Nicklin EL,
Baraff LJ. Pulse Oximetry as a Fifth
Pediatric Vital Sign. Pediatrics
1997;99(5):681–6.
100. Advanced Life Support Group
Staff. Advanced paediatric life
support:The practical approach. 4th
edn. London: Blackwell BMJ
Publishing Group; 2005.
101. Otieno H,Were E, Ahmed I,
Charo E, Brent A, Maitland K. Are
bedside features of shock
reproducible between different
observers? Archives of Disease in
Childhood 2004;89(10):977–9.
102. Leonard PA, Beattie TF. Is
measurement of capillary refill time
useful as part of the initial
assessment of children? European
Journal of Emergency Medicine
2004;11(3):158–63.
103. Gorelick MH, Shaw KN, Murphy
KO.Validity and reliability of clinical
signs in the diagnosis of dehydration in
children. Pediatrics 1997;99(5):E6.
104. Holmes JF, Palchak MJ, MacFarlane
T, Kuppermann
N. Performance of the pediatric glasgow
coma scale in children with blunt head
trauma. Academic Emergency Medicine
2005;12(9):814–9.
105. Ng SM,Toh EM, Sherrington CA.
Clinical predictors of abnormal
107. Batchelor J, McGuiness A. A 1990;41(2):139–50.
meta-analysis of GCS 15 head
injured patients with loss of 115. Department of Human Services
consciousness or post-traumatic (Victoria). Review of Trauma and
amnesia. Emergency Medicine Emergency Services 1999: Final Report.
Journal 2002;19(6):515–9. Melbourne: DoHS (Victoria); 1999.
108. Dunning J, Batchelor J, Stratford-Smith 116. Hewson P, Poulakis Z, Jarman F,
P,Teece S, Kerr J, McMaster D, Goodge J. Clinical
Browne J, Sharpin C, et al. A meta-analysis markers of serious illness in young
of variables that predict significant infants: a multicentre follow-up study.
intracranial injury in minor head Journal of Paediatrics & Child Health
trauma. Archives of Disease in 2000;36(3):221–5.
Childhood 2004;89(7):653–9.
109. Culotta VP, Sementilli ME,
Gerold K,Watts CC.
Clinicopathological heterogeneity
in the classification of mild head
injury. Neurosurgery
1996;38(2):245–50.
110. Palchak MJ, Holmes
JF,Vance CW, Gelber RE, Schauer
BA, Harrison MJ, et al. A decision
rule for identifying children at low
risk for brain injuries after blunt
head trauma. Annals of
Emergency Medicine
2003;42(4):492–506.
111. Palchak MJ, Holmes JF, Vance
CW, Gelber RE, Schauer BA,
Harrison MJ, et al. Does an isolated
history of loss of consciousness or 198
amnesia predict brain injuries in
children after blunt head trauma?
Pediatrics 2004;113(6):e507–13.
112. Stiell IG,Wells
GA,Vandemheen K, Clement C,
Lesiuk H, Laupacis A, et al.The
Canadian CT Head Rule for
patients with minor head injury.
Lancet 2001;357(9266):1391–6.
113. Merkel SI,Voepel-Lewis T,
Shayevitz JR, Malviya S. The FLACC:
a behavioral scale for scoring
postoperative pain in young
children. Pediatric Nursing
1997;23(3):293–7.
114. Bieri D, Reeve RA, Champion GD, Addicoat
L,
Ziegler JB.The Faces Pain Scale for the
self-assessment of the severity of
pain experienced by children:
development, initial validation, and
preliminary investigation for ratio
scale properties. Pain
117. Emergency Nurses Association V. survey of 138 ectopic pregnancies. Archives
Position Statements:Triage and of Gynecology and Obstetrics
Educational Preparation of Triage 1998;261(2):83–7.
Nurses. In: Emergency Nurses
Association, Victoria; 2000. 128. Paterson Brown S. Placenta praevia
and placenta praevia accrete: diagnosis and
118. Considine J, LeVasseur SA, management. London: Royal College of
Charles A. Consistency of Triage in Obstetrics and Gynaecology; 2005.
Victoria’s Emergency Departments:
Guidelines for Triage Education and
Practice. In: Monash Institute of Health
Services Research Report to the
Victorian Department of Health
Services, 8 July 2001;2001.
119. Wise RA, Polito AJ. Respiratory
physiologic changes in pregnancy.
Immunology and Allergy Clinics of
North America 2000;20(4):663–8.
120. Murphy VE, Gibson P, et al.
Severe asthma exacerbations during
pregnancy. Obstetrics and
Gynecology 2005;106(5):1046–54.
121. Turner LA, Kramer, MS. Cause-
specific mortality during and after
pregnancy and the definition of
maternal death. Chronic Diseases in
Canada 2002;23(1).
199
122. Flik K, Kloen P, et al. Orthopaedic
trauma in the pregnant patient. Journal
of the American Academy of
Orthopaedic Surgeons 2006;14(3):175–
82.
123. Kearney M, Haggerty l, et al. Birth
outcomes and maternal morbidity in
abused pregnant women with public
versus private health insurance. Journal
of Nursing Scholarship 2003;35(4):345–
9.
124. Crochetiere C. Obstetric
Emergencies. Anesthesiology
Clinics of North America
2003;21(1):111–25.
125. Higgins S. Obstetric
haemorrhage. Emergency Medicine
Australasia 2003;15(3):227–31.
126. Coppola P, Coppola M.Vaginal
bleeding in the first 20 weeks of
pregnancy. Emergency Medicine Clinics
of North America 2003;21(3):667.
127. Aboud E, Chaliha C. Nine year
129. Cousins L. Fetal Health.Triage in NSW rural and remote
oxygenation, assessment of fetal Emergency Departments with no
wellbeing, and obstetric on-site doctors. [Online] 2004 [cited
management of the pregnant 2006 April 12]. Available from:
patient with asthma. Journal of URL:
Allergy And Clinical Immunology http://www.health.nsw.gov.au/pubs/200
1999;103(2(2)):S343–9. 4/pdf/ triage_rural_remote.pdf
130. Laurant L. Encounter at 140. Society of Rural Physicians of
triage results in legal liability. Canada Emergency Committee (SRPC-
Journal of Emergency Nursing ER) Working Group. CAEP and SRPC
2003;29(1):55–7. Position Statement – Rural
Implementation of CTAS [Online] 2002
131. Newberry L, editor. Sheehy’s [cited 2006 April 12]. Available from:
Emergency Nursing: Principles and URL:
practice. 5th ed. St.Louis: Mosby; http://www.caep.ca/002.policies/002
2003. -01. guidelines/CTAS-rural.htm
132. Verderber R.
Communicate! 10th ed.
Belmont: Wadsworth; 2002.
133. DeVito JA. Human
Communication:The basic
course. 8th ed. New York:
Longman; 2000.
134. Adler RB, Rodman G.
Understanding Human
Communication. 8th ed. New
York: Oxford University Press;
2003.
135. Martin WB. Quality
Customer Service. 4th ed.
Menlo Park: Crisp Publications;
2001.
136. Hegney D. Dealing with
Distance: rural and remote area
nursing. In Daly J, Speedy S,
Jackson
D. editors. Contexts of Nursing.
2nd ed. Sydney: Elsevier Australia;
2006. p. 213–28.
137. Bushy A, Bushy A. Critical
access hospitals: rural nursing
issues. JONA 2001;31(6):301–
10.
138. South Australian
Emergency Nurses Association.
Position Statement: triage in
rural and remote hospitals.
[Online] 2003 [cited 2006 April
12]. Available from:
URL: http://www.cena.org.au/pdfs/
sa_position_rural.pdf
139. NSW Department of
141. Rural Doctors Workforce
Agency. Rural after hours triage
education resource.
Adelaide: RDWA; 2005.
142. Health Information for
International Travel. Chapter
8:Traveling Safely with Infants and
Children. USA: Centers for Disease
Control and Prevention [Online]
2005 [Cited 2007 March 24.
Available from:
URL: http://www2.ncid.cdc.gov/travel/yb/utils/ybGet.
asp?section=children&obj=children_gen_info.
htm&cssNav=browsecyb
143. Clinical Practice Guidelines.
Melbourne: Royal Children’s Hospital
[Online] 2006
[viewed 2007 April 17]. Available
from: URL:
http://www.rch.org.au/clinicalguide/
cpg. cfm?doc_ id=5177
144. National Health and Medical
Research Council. National Statement
on Ethical Conduct in Research
Involving Humans. 1999 [cited 2001
18 September]; Available from:
URL:
http://www.health.gov.au.nhm
rc/ publications/synopses 200
145. Fleiss JL, Nee JCM, Landis JR.
Large sample Variance of Kappa in the
Case of Different sets of Raters.
Psychological Bulletin. 1979;86(5):974–
7.
INDEX C
Canadian Triage and Acuity Scale,
A 6 capillary refill time, paediatric
Abbey Pain Rating Scale, 59 triage, 65, 68 cardiac output,
abbreviations, 192 pregnant patients, 73
ABCs of mental health assessment, cardiotocograph, 75
38–41 abdominal pain, pregnant carers, information provided by, 64, 66
patients, 76 accountability, 80 cerebral haemorrhage, 74
ACEM, see Australasian College for cervical spine management, paediatric
Emergency Medicine acuity, measuring, 6 triage, 65 challenging communications,
acute behavioural disturbance, 20
42 acute pain, 58 children, see paediatric
affect, 39 triage chronic pain, 58
age factors, 28, see also paediatric circulation problems, 28
triage extremes of age, 32 paediatric triage, 65, 68
gestational age, 74–6 pregnant patients, 73
young patients, 42 clinical descriptors, 134
airway obstruction, 28, 30 clinical practices, 141
paediatric triage, 65, 68 clinical urgency, see
pregnant patients, 73 urgency co-morbid
altered consciousness, 31, 66, 82 factors, 67
analgesia, 60 cognitive impairments, see
answers to test questions, conscious-state abnormalities;
149–91 antepartum mental health triage
haemorrhage, 75 collegial ramifications, 52
aorta, 74 comfort, need for, 20
appearance criteria, 30, communication issues,
39, 65 application 16–26
procedures community knowledge, 52
Australasian Triage Scale, 11 conditions, criteria based on, 28, 39, 65–
pain assessment, 60 9 confidentiality issues, 64, 83, see also 201
assessment privacy issues conscious-state
techniques, 30–3 abnormalities, 31, 66, 82
mental health, 38–41 consent, 80–1
pain, 58–60 consistency of
assumptions, 19 triage, 12
asthma, 65, 73 consolidation exercises, 87–105
ATS, see Australasian Triage Scale conversation criteria, 39
Australasian College for Emergency coronial
Medicine, 82 policy document, 132–5 investigations, 33
Australasian Triage Scale, vii, 5– correct triage
6, 8–9, 80 descriptors for decisions, 7 critical
categories, 136–8 thinking skills, 139
implementation of, 132–5 CTAS, 6
AVPU scale, 66 cultural diversity, 19
B D
behaviour data analysis, 144–5
acute behavioural decision-making, factors
disturbance, 42 disturbed, in, 13 definitions, 3
135 dehydration, paediatric triage, 65–6, 68
in mental health assessment, 39 delirium, 40
blood pressure, see hypertension; dementia, 40
hypotension Box Hill Hospital System, demography of testing
11 participants, 148 detention of
breathing patients, 81
paediatric triage, 65, 68 disability rating, 28, 31–2, 66, 68–9
respiratory distress, 28, 31 district nursing, 51
documentation
requirements, 82–3, 133
duty of care, 81–2
E hyperthermia, 32
eclampsia, 75–6 hypotension, 31
ectopic pregnancy, 74–5 paediatric triage, 65
education pregnant patients, 73, 76
for patients and hypothermia, 32
public, 140 for Triage hypoxaemia, 31
Nurses, 3, 7, 140
recommended training I
scenarios, 146 elderly patients, Immediate treatment criteria, 42,
28, 32 136 mental health
emergency assessment, 43 paediatric
departments triage, 68–9
activity in, 17 implementation, 132–5
rural and remote, 50 implied consent, 81
triage in, 4 importance, need to
emergency nurses, see Triage feel, 20 infectious
Nurses Emergency Severity diseases, 67
Index, 6 Emergency injury patterns, 67
treatment criteria, 136 Ispwich Triage Scale (ITS), 11
mental health assessment, 43
paediatric triage, 68–9 L
Emergency Triage Education Kit, language use,
methodology, 142–8 emergency triage 19 left lateral
scales, 5–6 tilt, 74 lesson
emotions, 19 plans, 3
environmental criteria, 32 life support equipment,
environmental hazards, 30, 141 30 life-threatening
equipment, 30, 141 conditions, 28
ESI, 6
202
ETEK, methodology, 142–8 M
ethical approval, 144 management of pain,
evidence-based care, 140 60 Manchester Triage
expectations, 19 Scale, 6
expected triage mandatory reporting
decision, 7 eye injuries, responsibilities, 83 mass casualty
31–2 incident triage, 4
mastitis, 76
F maternal oxygenation, 76
fast-tracking policies, 135 MCI triage, 4
financial issues, rural and remote medico–legal issues,
nursing, 51–2 foetal wellbeing, 74–6 79–86 Mental Health
‘footprints’, 12 Act, 40
forensic evidence, preservation mental health triage, 37–48, 82
of, 83 function of triage, 5, 132 mood factors, in mental health
assessment, 39–40 MTS, 6
G multiple jobs, 51
GCS, see disability rating musculoskeletal pain, 60
general appearance criteria, 30, 39, 65
gestational age, 74–6 N
glossary, 193 nasal congestion, 73
National Education Framework for
H Emergency Triage Working Party, 143
haemodynamic compromise, 31 National Health Workforce Strategic
high-risk Framework, vii National Specialisation
conditions, 32 Framework for Nursing and
history of triage, 4 Midwifery, vii
history-taking in paediatric triage, 64, 67 National Triage Scale, 5–6, 11, see also
homicide risk, 39–40 Australasian Triage Scale
human needs, 20 needs, human, 20
hypertension in pregnant patients, 75–6 negligence laws, 82
‘noise’, 17 Non-urgent treatment criteria, 138
Department of Health and Ageing – Emergency Triage
Education Kit
mental health progesterone, respiratory effects, 73 program
assessment, 45 structure, 2
paediatric triage, 68–9 protocols, 80
non-verbal behaviours, 19 psychostimulant drugs, 42
NTS, 5–6, 11, see also Australasian pulmonary embolus, 74
Triage Scale numerical pain rating PV loss, 74
scales, 59–60
nurses, see Triage Nurses
O
obstetrics, 72–8
occluded airways, see airway
obstruction on-site medical staff,
50
ophthalmologic injuries, 31–2
organic illnesses, mental conditions
caused by, 39 outcomes, Australasian
Triage Scale, 11
over-triage, 7
oxygenation and foetal wellbeing, 76
P
paediatric triage, 63–71,
134 children at risk, 83
pain assessment, 57–
62 pallor, paediatric
triage, 65
palpitations, 74
parents of patients, 64,
66 participants in
program, 3
past history, paediatric
triage, 67 patients,
prioritising, 33
per vagina loss, 74, 76
performance indicators, time-to-
treatment, 12 philosophy of ATS, 11
physical environment, 19
physiological abnormalities, 28, 39,
65–9
placental conditions, 75
Plain Language Statement (PLS),
143 position statements, 80,
139–41
postnatal patients, 76
postpartum cardiomyopathy, 76
practicality, 134
predictors of
outcome, 28
preeclampsia, 75
pregnancy and
triage, 72–8
premature labour, 75
primary-survey approach,
38, 73–4 primary triage
decisions, 7 prioritisation of
patients, 33 privacy issues,
64, 83
professional development, 140
R gn, 64
secondary postpartum
rashes, 32
haemorrhage, 76 secondary
rationale, 141 203
triage decisions, 7
re-
self-report of pain,
triage,
58–9 self-test, 106–
3, 32,
31
82, 134
Semi-urgent treatment criteria,
reaction
138 mental health
s of
assessment, 44 paediatric
patient,
triage, 68–9
39
South Eastern Sydney Area Health
reasona
Service (SESAHS) triage tool, 38
ble
splenic artery, 74
standar
spontaneous arterial dissection,
d of
74 stridor, 30, 65
care, 81
sub-arachnoid haemorrhage, 74
recommended
subclavian artery, 74
training
suicide risk, 39–40
scenarios, 146
Sutherland Hospital guidelines, 38
recommended
triage method,
29 references,
T
194–200 tachycardia, paediatric
Registe triage, 68 Tasmanian
red triage rating, 38 telephone
Nurses, triage, 51
139 time constraints, 19, 51
time-to-treatment, 4, 12,
reliabilit
133 tissue turgor,
y of
paediatric triage, 68
scale, 5
training, see education
remote
trauma patients, 32, 135
location
triaging triage, 3, 7, 27–36
, 49–56 Triage Education Resource
reportin Book, vii Triage Nurses
g, 83 communication skills, 17
reprod
ucibilit
y, 134
respira
tion,
see
breath
ing
rest, ice,
compression
and elevation
(RICE)
treatment, 60
risk assessments, 40, 69
rural and remote triaging, 49–56
S
safety
issues,
5, 30,
51, 133,
141
SAVE A
CHILD
campai
Department of Health and Ageing – Emergency Triage
medico–legal issues, 80
position statements, 80,
139–41 rural and remote, 51
training for, 132
Triage Observation Tool, 64, 70
U
under-triage, 7
understanding, need
for, 20 urgency
defined, 3
determines category, 134
paediatric triage, 64
Urgent treatment
criteria, 137
mental health assessment,
44 paediatric triage, 68–9
urine output, paediatric
triage, 68 utility of scale, 5
V
vaginal bleeding, 74,
76 validity of scale, 5
verbal communication, 17
verbal consent, 81
verbal pain rating scales,
204 59–60 visual analogue pain
rating, 59–60
W
Wong-Baker FACES Rating
Scale, 59 written consent, 81
Y
Yale Observation Scale, 64, 70
young patients, 42, see also age factors
Department of Health and Ageing – Emergency Triage Education Kit
www.health.gov.au